Download as pdf or txt
Download as pdf or txt
You are on page 1of 117

ELPT_Level 2

ELPT LEVEL 2

Warning
For internal use only. Please do not copy, reproduce, or distribute
without permission from Pusat Bahasa dan Multibudaya

Peringatan
Hanya untuk penggunaan internal. Mohon tidak menyalin, memperbanyak
atau membagikan tanpa izin dari Pusat Bahasa dan Multibudaya

i
ELPT_Level 2

Third Edition : 2019

Pusat Bahasa dan Multibudaya


Universitas Airlangga
Jl. Dharmawangsa Dalam Kampus B - Surabaya
Ph. 031-5029046
Fax. 031-5053607
www.pinlabs.unair.ac.id
Email: pusatbahasa@fib.unair.ac.id

ii
ELPT_Level 2

CONTENTS
Introduction .........................................................................................................................................v
General Strategies ............................................................................................................................. vi
Score Conversion Table .................................................................................................................... vii

LISTENING SECTION
Part A ................................................................................................................................................. 1
Part B & C .......................................................................................................................................... 7
Exercise Part A .................................................................................................................................. 9
Review Part A .................................................................................................................................. 13
Exercise Part B & C ......................................................................................................................... 14
Practice Test 1 ................................................................................................................................. 16
Practice Test 2 ................................................................................................................................. 21
Practice Test 3 ................................................................................................................................. 25

STRUCTURE SECTION
Subject, Object and Noun Complement .......................................................................................... 30
Subjects and Verbs ......................................................................................................................... 31
Expressions of Quantity .......................................................................................... 31
Articles ............................................................................................................................................ 32
Personal, Possessive, and Reflexive Pronouns .............................................................................. 33
Adjectives and Nouns ...................................................................................................................... 33
Verb Tense Form ............................................................................................................................. 34
Verb Tense Meaning ....................................................................................................................... 35
Passive and Active sentences ......................................................................................................... 36
Modals and Modal-Like Verbs ......................................................................................................... 37
Subject – Verb Agreement .............................................................................................................. 38
Preposition ....................................................................................................................................... 39
Main and Subordinate Clause Markers ........................................................................................... 39
Prepositional Phrases and Subordinate Clauses ............................................................................ 40
Appositive and the Noun Structures They Rename ........................................................................ 41
One-Word –ing and –ed Adjectives ................................................................................................. 41
–ing and –ed Modifying Phrases ..................................................................................................... 42
Gerunds and Infinitives .................................................................................................................... 42
It and There Sentences ................................................................................................................... 43
Word Form and Function ................................................................................................................. 44
Words that Don’t Exist in English .................................................................................................... 45
Equative, Comparative and Superlative Degree ............................................................................. 45
Standard Word Order ...................................................................................................................... 46

iii
ELPT_Level 2

Word Order in Subordinate Clauses Beginning with Question Words ............................................ 47


Inverted Subject – Verb Word Order ............................................................................................... 47
Parallel Structure in Comparisons ................................................................................................... 48
Paired Expression ........................................................................................................................... 50
Confusing Words and Expressions ................................................................................................. 51
Review Exercise .............................................................................................................................. 52
Practice Test 1 .................................................................................................................................. 58
Practice Test 2 .................................................................................................................................. 61
Practice Test 3 .................................................................................................................................. 65
Practice Test 4 .................................................................................................................................. 69
Practice Test 5 ................................................................................................................................. 72
Practice Test 6 ................................................................................................................................. 75

READING SECTION
Skimming ......................................................................................................................................... 79
Scanning .......................................................................................................................................... 79
Topic and Main Ideas ...................................................................................................................... 79
Vocabulary in Context ..................................................................................................................... 80
Purpose and Organizational Pattern ............................................................................................... 81
Reference Words ............................................................................................................................. 82
Detail and Factual Information ........................................................................................................ 83
Making Inference ............................................................................................................................. 84
Attitude of the Author and Tone of the Passage ............................................................................. 84
Practice Test 1 ................................................................................................................................. 85
Practice Test 2 ................................................................................................................................. 91
Practice Test 3 ................................................................................................................................. 97

Bibliography ................................................................................................................................... 105

iv
ELPT_Level 2

INTRODUCTION
ABOUT THE COURSE

ELPT Level 2 is a course program provided by Pusat Bahasa dan Multibudaya – Universitas
Airlangga to prepare students for UA’s ELPT test. This program is intended for students whose score
is between 453 – 500. This program consists of 10 meetings (which is equal to 20 hour sessions), 7
of which are the materials and the others are for the tests. During the program, the program takers will
be intensively trained and thus finally better equipped with strategies and skills to answer questions in
UA’s ELPT test.

ABOUT THE MODULE AND WHAT IS IN THE MODUL

This module is specifically designed to meet the above objectives. This module contains a variety of
materials which together provide a comprehensive and intensive program:

 ELPT General Strategies. This will provide a broad picture of ELPT test and things to prepare
before sitting for the test
 Score Conversion Table. This will allow the program takers to estimate their ELPT score after
taking the test. There is also a clear example of how to use the table.
 Skill Based General Strategies. For each section of the test, Listening Section, Structure Section,
and Reading Section, there will be general strategies to help them to cope with technical difficulties
in each section of the test.
 Skill Based Specific Strategies. In each section of the test, there will be clearly defined skills to
improve performance on the test. .
 Exercises. In each section of the test, the program takers will be provided with exercises to
practice their skills and eventually to improve their skills on the test.

v
ELPT_Level 2

GENERAL STRATEGIES
1. Use your time wisely. The ELPT is a timed test. You must work very quickly and efficiently to
finish all of the questions in the time you are allowed. The skill strategy practice taught in this
course will help you learn to budget your time on each section of the ELPT test. In addition,
you should bring a watch to the test so that you can keep track of your time.

2. Don’t read the instructions to each ELPT test section. The test instructions used in exercises
during the course are exactly the same as the ones used on the ELPT test. You should
become familiar with these instructions. If you are familiar with them before you take the ELPT
test, you do not have to read them when you begin the test. Instead, when permitted, you can
move immediately to the test questions and begin working.

3. If you do not know the answer to a question, GUESS. This is a very important strategy to use
when taking the ELPT test. There is no penalty for guessing. You have a twenty-five percent
chance of guessing the correct answer to each question. Unmarked answers will be
counted as wrong and will lower your score.

4. Mark your answer sheet very carefully. To prevent marking answers in the wrong order, you
should follow your place on your answer sheet with one finger. Check to see that the number
next to this finger is the same as the number of the question you are looking at in your test
booklet. When you choose your answer, fill in the circle completely. If you need to change
the answer, erase it completely, and mark your new answer.

5. Do not write in your test booklet. You are not allowed to make any marks on your ELPT test
booklet during the test.

6. The night before the test, relax. Do not try to do any serious studying the night before the test.
This will only make you nervous and tired. The night before the ELPT test, it is a good idea to
relax and go to bed early. Then, you will be at your best for the test.

7. It is a good idea to eat something substantial before the ELPT test. This test is a long exam,
and having something to eat beforehand can help you to focus and concentrate on the test.

8. Arrive at the test center ahead of time. If you are late for the ELPT test, you will not be
allowed to take it.

vi
ELPT_Level 2

SCORE CONVERSION TABLE


Use the following table to estimate your ELPT scores
Number Correct (Cs) Converted Scores
Section 1 Section 2 Section 3
50 68 - 67
49 67 - 66
48 66 - 65
47 65 - 63
46 63 - 61
45 62 - 60
44 61 - 59
43 60 - 58
42 59 - 57
41 58 - 56
40 57 68 55
39 57 67 54
38 56 65 54
37 55 63 53
36 54 61 52
35 54 60 52
34 53 58 51
33 52 57 50
32 52 56 49
31 51 55 48
30 51 54 48
29 50 53 47
28 49 52 46
27 49 51 46
26 48 50 45
25 48 49 44
24 47 48 43
23 47 47 43
22 46 46 42
21 45 45 41
20 45 44 40
19 44 43 39
18 43 42 38
17 42 41 37
16 41 40 36
15 41 40 35
14 39 38 34
13 38 37 32
12 37 36 31
11 35 36 30
10 33 33 29
9 32 31 28
8 32 29 28
7 31 27 27
6 30 26 26
5 29 25 25
4 28 24 24
3 27 23 24
2 26 22 23
1 25 21 22
0 24 20 21

vii
ELPT_Level 2

Use the following formula to estimate your total ELPT score

Score Conversion

Your total score is equal to:


Section 1 + Section 2 + Section 3 X 10 = Your Score
3

Following is an example of how the Score Conversion Tables works


Sample Score Conversion

Andi, a student who has just finished taking Practice Test One, had the following scores

Section 1,
Number Correct = 27 Converted Score = 49
Section 2,
Number Correct = 29 Converted Score = 53
Section 3,
Number Correct = 37 Converted Score = 53

Using Score Conversion table, Andi can estimate his total ELPT Score for Practice Test One,
49 + 53 + 53 X 10 = 517
3

viii
ELPT_Level 2
ELPT_ Level 2

The purpose of listening section is to test your understanding of spoken English. Vocabulary, spoken
structures, and English sounds are tested. For the most part, topics used in this section are informal
and conversational. Some general academic topics are used in the short lecture segments of this
section. Even in these topics, however, the language is not as formal as that used in written English.

Section One contains 50 questions. The questions and the information you need to answer them are
played on a tape. Only the answer choices are printed in the test booklet. All Section One questions
are spoken just one time. You are allowed 12 seconds to answer each question. It takes
approximately 40 minutes to complete this section of the test.

There are three parts in this section.

Part A: Short-Dialogues
You choose the correct answer to a spoken question based on a very short spoken dialogue.

Part B: Longer Conversation


You choose the correct answers to a series of questions about a longer spoken dialogue between two
people.

Part C: Talks
You choose the correct answers to a series of questions about a talk given by one speaker.

PART A: SHORT DIALOGUE


The part A questions in Section One of the test measure your ability to understands short dialogues
between two people as well as to understand and respond to spoken questions about each dialogue.
Each short dialogue in this section is two lines long and involves two speakers. A third person asks a
question about what was said. You hear each dialogue and the question about it just one time.
After you hear the question, you have 12 seconds to read the four answer choices and decide which
one is the best answer to the question you heard. There are 30 questions in this section.

1. KEY WORDS AND KEY WORD SYNONYMS


All short dialogues in Part A contain key words. Key words are important words in the sentence that
give you an overall understanding of the statement. Listen carefully for the key words in Part A.
In addition, look for repeated words and their synonyms in the answer choices for part A questions.
These words are also key words because they usually relate to the key meaning in the short-
dialogues.

Example

You will hear


W: What is this?
M: It’s a composition Peter wrote for his business class
Q: What does the man mean?

You will read:


A. This class was for Peter
B. This is an essay Peter composed for a course
C. Peter’s taking this composition to class.
D. Peter is not interested in his composition course

Explanation
In this question, the key words Peter, wrote, composition, and class give you important information
about the overall meaning of the sentence. All four answer choices for this question contain the key
word Peter. However, only answer (B) contains key word synonyms for wrote, composition, and class.
These words are composed

Listening Section | 1
ELPT_ Level 2

2. IDIOMS
In Part A questions, idioms are often included in the sentences you hear. When you hear an idiom
used in a sentence, you must look for the best restatement of the meaning in the answer choices.
Incorrect answer choices often contain the literal or non-idiomatic meaning of idioms.

Example

You will hear


W: Look what you’ve done!
M: Wait a minute! Jane had a hand in this mess, too!
Q: What does the woman say about Jane?

You will read:


A. Jane’s job is messy.
B. Jane is partly responsible for this situation.
C. Jane put her hand in the dirt.
D. Jane applauded.

Explanation
In this question, the meaning of the idiom to have a hand in is being tested. Answer (A) contains a
form of the key word mess, but it does not contain a restatement of the idiom. Answer (C) contains a
literal (non-idiomatic) restatement of the meaning of the individual parts of the idiom. However, the
meaning of an idiom cannot be understood by understanding its parts. Answer (D) restates the
meaning of a different idiom, to give a hand, which sometimes means to applaud. Answer (B) is the
correct answer to the question. It is the only answer that contains an accurate restatement of the
meaning of the idiom to have a hand in. This idiom means to be partly responsible for.

3. WORDS WITH MANY MEANINGS


Some English words have only one meaning and one function. Many, however, have several
functions and many meanings. Look at the following examples:

I would like to buy a new winter coat.


This wall needs a second coat of paint.
She coated her ice cream with chocolate syrup.

In Part A questions, you will often hear a key word having one meaning. Then, in the answer choices,
you might see the same word again, but it will have a different meaning. Answer choices containing
words with many meanings are usually not the correct answers. You should be careful not to be
tricked by words with many meanings.

Example

You will hear


W: Jack’s been very busy lately, hasn’t he?
M: Yes. He’s running his father’s office.
Q: What does the woman mean?

You will read:


A. Jack runs to his father’s office.
B. Jack’s father is running for a public office.
C. Jack manages his father’s office.
D. Jack is running away from his father’s office.

Explanation
In this question, the many meaning of run and office are tested. In the sentence you hear runs means
manages and office means place of work. In answer (A) runs means goes faster than a walk. In
answer (B) is running means is trying to win a political race, and office means political position. In
answer (D), running is combined with away to mean leaving without permission. Answer (C) is the
correct answer to the question. It replaces runs with manages, and office has the same meaning as it
does in the statement you hear.

Listening Section | 2
ELPT_ Level 2

4. SOUND-ALIKES
There are many words in English that sound almost the same. These sound-alikes include minimal
pairs, numbers, and longer sound-alikes. Some sound-alikes are separated by only one difference in
sound. Words which differ by only one sound are called minimal pairs.
tall - ball bit - beat pick - pit
The different sound in a minimal pair can occur at the beginning of a word (as in a tall/ball). It can also
occur in the middle of a word (as in bit/beat) or at the end of a word (as in pick/pit). Although minimal
pairs differ by only one sound, they usually differ a great deal in meaning. If you cannot distinguish
minimal pair, you will have difficulty in understanding English sentences.

Longer English phrases also can be sound-alikes


make a mistake - bake a cake clearing out the store - cleaning up the floor
The two phrases above contain minimal pair (make/cake) as well as other similar sounding words and
phrases (clearing/cleaning, a cake/mistake, and the floor/the store). If you cannot distinguish these
longer sound-alikes, you will have difficulty in understanding English sentences.

Example

You will hear


W: What are Jane Smith’s children up to these days?
M: Her daughter is in Tennessee. Her thirty-year-old son is living with his father
Q: What does the woman say about Jane Smith’s son?

You will read:


A. Her son, who is thirteen years old, is leaving his father.
B. Her son, who is thirty years old, stays with his dad.
C. Her thirty-year-old son will go a little farther.
D. Her third son lives with his father.

Explanation
In this question, several sound-alikes are used. Answer (A) contains the number thirteen, which
sound likes thirty. It also contains leaving his father which sound like living with his father. Answer (C)
contains farther, which sounds like father. Answer (D) contains third, which sounds like thirty. Answer
(B) is the correct answer to the question even though it sounds the least like the spoken sentence

5. NEGATIVES
Negative meaning is common in English sentences. There are several negative structures.

1. Not (-n’t) and other negative words such as never, nobody, no, none, not one, nothing and
nowhere
Jane is not here (isn’t) here today Nothing makes him better
There is no reason to worry about. Nobody/No one cares about this matter
James has never been here before None of us took the offer
2. Negative prefixes such as de-, dis-, il-, im-, in-, ir-, non-, and un-. These prefixes can be
added to the beginning of some words to change them to their opposite meanings:
deemphasize (not emphasize) impossible (not possible)
disinterested (not interested) inconsiderate (not considerate)
illogical (not logical) irreplaceable (not replaceable)
nonpayment (lack of payment) unattractive (not attractive)
3. Almost negative expressions such as barely, hardly, and scarcely (which describe how
much), and rarely and seldom (which describe how often):
We barely
scarcely had enough money to pay for the concert.
hardly
We rarely go to concert
seldom

Listening Section | 3
ELPT_ Level 2

It is common for one negative structure to occur in an English sentence. In addition, two or more
negatives can be used in one sentence. When this happens, the total meaning of the sentence is
sometimes difficult to understand.

John was not impressed.


John was not unimpressed. (John was impressed)

Example 1

You will hear


W: I think John is in big trouble.
M: Why? He didn’t disobey his father
Q: What does the man mean?

You will read:


A. John didn’t do what his father wanted him to do.
B. John did what his father wanted him to do.
C. John’s father was not happy with him.
D. This wasn’t approved by John’s father.

Explanation
There are two negatives in the mas’s sentence, -n’t (not) and dis- (disobey). These two negatives give
the sentence an affirmative meaning. Answers (A), (C), and (D) all have negative meanings. Answer
(B) is the correct answer to this question because even though it contain no negative structures, it
accurately restates the meaning of the spoken sentence.

Example 2

You will hear


W: Peter is so modest about his accomplishments!
M: He can’t deny that he is very clever. Everybody knows it!
Q: What does the woman mean?

You will read:


A. Peter must admit that he is clever.
B. No one can say that Peter is clever.
C. Peter is not very clever.
D. Peter pays careful attention to details.

Explanation
In the woman’s sentence, the negative –n’t (not) is used to change the meaning of deny from not
admit to admit. Answer (A) is the correct answer to this question even though it does not contain a
negative.

6. WHO DOES WHAT


It is sometimes difficult to understand who does what in English sentences. This can be true
whenever there is more than one person being talked about. It is especially difficult to understand
who does what in passive sentences and in causative constructions using have, let, make and get.

Jamie gave Louise ring.


Who gave the ring? Jamie

The ring was given to Louise by Jamie. (Passive sentence)


Who gave the ring? Jamie

Jamie got Andrew to give Louise a ring. (Causative construction)


Who gave the ring? Andrew

Listening Section | 4
ELPT_ Level 2

Example

You will hear


W: Who can help us cook for the party?
M: I’ll have Sarah do it?
Q: What does the woman mean?

You will read:


A. Sarah will cook for the party for her.
B. Sarah is at the party.
C. She will cook for the party for Sarah.
D. Sarah cooked for her.

Explanation
It is important to understand that Sarah is the one who will do the cooking for the party. Answer (B)
mentions Sarah, but does not talk about cooking. Answer (C) has the opposite meaning of the
woman’s sentence. Answer (D) takes place in the past tense, while the dialogue takes place in the
present. Answer (A) is the correct answer because it restates what the woman means.

7. SITUATION
Understanding the situation in which a dialogue takes place is an important part of understanding the
meaning of a dialogue. It is important to understand who is speaking and where or in what
circumstance they are talking.

Example 1

You will hear


W: I should be able to perform your surgery tomorrow morning.
M: I’ll be glad when it’s over.
Q: What job does the woman probably have?

You will read:


A. Doctor
B. Performer
C. Lawyer
D. Plumber

Explanation
The question specifically asks about the situation. You need to be able to infer that the woman in the
dialogue is a doctor. The key word surgery makes it possible for you to do this. Answer (A) is the
correct answer to this question because it identifies who the woman is in the situation.

8. TOPIC
Once you understand the situation in which a dialogue takes place, it is important to understand the
topic of the dialogue, or what the speakers are talking about.

Example 1

You will hear


W: Today is my birthday.
M: Really? How old are you?
Q: What does the woman want to know about the man?

You will read:


A. What he does for a living.
B. How many days he will be staying in town.
C. His age
D. His name

Listening Section | 5
ELPT_ Level 2

9. LANGUAGE FUNCTION
In English conversation, one sentence can have several different functions, depending upon the
context in which it is used. These functions include expressing agreement or disagreement;
complaining; showing excitement, confusion or surprise; making and refusing requests; stressing the
importance of an idea; and making suggestions and giving advice.

Language Function Some Expressions Used

Do you mind if I use your pen?


Requesting permission
Would it be OK (with you) if I came, too?
I wonder if I could borrow your pencil.

Do you know where the library is?


Requesting information
C Can you tell me how to get to Church Street?
Is there a library around here?

Would you mind opening the window?


Requesting that someone do something Would you please take out the trash?
Could you please get me that book?

What do you think of this dress?


Do you think Larry will come?
Requesting advice or an opinion
Would it be better for me to call or write?
Should I ask for a raise?

Shouldn’t you eat before you go out?


Giving advice or an opinion Why not do your homework now?
Try calling him in the afternoon.
You really should be careful on the highway.

Why don’t we go to the concert tonight?


Suggesting speaker and listener Let’s see that movie we were talking about
do something together What would you say to a vacation?
How about going to dinner on Tuesday?

Shall I do that for you?


May I take your hat?
Offering to do something
Can I take your coat?
Would you like me to get you some more tea?

Sure. Sound good to me.


I’ll say! No kidding.
Agreeing with suggestions, advice, etc.
No problem. So do I.
Great idea. Neither do I.
Sure, why not. Isn’t/Won’t/Doesn’t it.

No, thanks. I really shouldn’t.


Disagreeing with suggestions,
Thanks anyway, but I can’t.
advice, opinions, etc.
Sorry, but I don’t really agree with you.

That’s out of the question.


Refusing offers and requests
Not likely!
You must be kidding.

Listening Section | 6
ELPT_ Level 2

PART B & C:
LONGER CONVERSATION AND TALKS
10. ANTICIPATE THE TOPIC AND THE QUESTIONS
It’s very helpful to your overall comprehension if you know what topic to expect in the long
conversation.
The following is a set of answer choices for a longer dialogue. While you read through the answer
choices, try to predict the topic of the conversation and some of the questions that might be asked
about it.

You will read :


1. (A) Buy a car. 4. (A) The woman should make her
(B) Have her car repaired. decision right away.
(C) Borrow her friend’s car. (B) The woman should get a loan
(D) Walk to the gas station. from the bank.
(C) The woman should wait until the
2. (A) At a car dealership. prices are reduced.
(B) In the man’s apartment. (D) The car is too expensive for the
(C) On the freeway. woman
(D) In the woman’s new car.

3. (A) She doesn’t know yet.


(B) The black one.
(C) The red one.
(D) The white one.

Explanation
Without hearing the passage, you can predict the topic and some of the questions that will be asked
about it simply by thinking about the answer choices. Related key words can be found in all four sets
of answer choices. Related key words can be found in all four sets of answer choices. These key
words include buy, car, car dealership, get a loan from the bank, and prices. With these key word in
mind, you might predict that the topic of this passage will be buying a car.
You can also predict the question or at least part of the questions in this set of answer choices.

Topic: buying a car

Questions:
1. What does the woman want to do?
2. Where does this conversation take place?
3. Which car does the woman want?
4. What does the man advise the woman to do?

11. THE FIRST LINE OF EACH PASSAGE

The first few lines of any talk or longer conversation contain information that is essential for
understanding overall meaning. These lines give information about the situation (who and where) of
the passage, as well as about the topic (what) of the passage.
Understanding who, where, and what italics in each passage is important to understand overall
meaning. Some part B and part C questions specifically ask for this information. These italics
questions include: what is the topic of this conversation? Where does this conversation take place?
Who is the speaker? What is the woman’s / man’s job? Often this information is not stated directly in
the passage, but can be inferred from the first few lines. Other times this information will be stated
directly in the beginning of the passage.

You will hear :


M: May I help you?
W: Well I hope so. I’m thinking about buying a new car, and I saw one that I liked in your showroom.

Listening Section | 7
ELPT_ Level 2

Explanation
These are the first two lines from a longer conversation. Just from these two lines, you learn that the
speakers are a car salesman and a woman shopping for a car. You also learn that the conversation
probably takes place where cars are sold – at a car dealership. The key expression, May I help you?,
which is used by the salespeople, and the key words buying, car, and showroom help you infer this
information.
At least part of this information will be asked about in the spoken questions for this conversation. This
information is also important for understanding the rest of the conversation.

12. FACT AND INFERENCE QUESTIONS


Although the first lines of passages in Part B and Part C are important for answering fact and inference
questions, it is also important to listen carefully to the rest of the passage. You should concentrate on
the overall meaning of the passage. From this information, you will be asked to make further
inferences.
What can we assume about the speaker?
What can be inferred about X?
What will the speaker do next?
When you listen to the rest of the passage, you should also listen for facts. These facts may include
general information about what the speakers say. They may also include detailed information such as
numbers, dates, amounts, and names. Many fact questions are possible. The following are
examples:
What does the speaker say about his job?
How long has the speaker been living in the dorm?
Which book does the professor recommend?
When will the students have their exam?

Example

You will hear


M: May I help you?
W: Well I hope so. I’m thinking about buying a new car, and I saw one that I liked in your showroom.
M: Which one is that?
W: The red one with the black interior. Sitting next to the big white one.
W: Oh, that little beauty. Well, you’re in luck. That one’s on sale – today only. The sale ends at 6:00
this evening.

Question: Which car does the woman want?

You will read:


A. She doesn’t know yet.
B. The black one.
C. The red one.
D. The white one.

Question: What does the man imply?

You will read:


A. The woman should make her decision right away.
B. The woman should get a loan from the bank.
C. The woman should wait until the prices are reduced.
D. The car is too expensive for the woman.

Explanation
These two questions ask about information that comes in the middle and at the end of the passage.
Question 3 is a fact question. To answer this question correctly, you need to listen carefully for the
details in the passage. Answer (C) is the correct answer to this question.
Question 4 is an inference question. The man does not directly say that the woman should buy the
car right away; however, he does mention that the car will be on sale for only one day. From this
information, we can infer that he thinks the woman should hurry and buy the car. Answer (A) is the
correct answer to this question.

Listening Section | 8
ELPT_ Level 2

Exercise Part A
Practice with Idioms
Listen carefully to each dialogue and the corresponding question. Circle the letter that best answers
the question you hear.

1. (A) She is having trouble holding on to 8. (A) The house needs to be put in order.
Janet. (B) He needs to straighten the structure of
(B) Janet is holding a meeting this week. the house.
(C) She has been unable contact Janet. (C) The house really stands straight up.
(D) Janet is getting four weeks vacation. (D) He wants to move out of the house.

2. (A) He stopped to buy this in the 9. (A) She and Bob never look at each other.
afternoon. (B) She and Bob never agree with one
(B) She thinks he will visit her today. another.
(C) She will stop waiting for him. (C) Neither she nor Bob sees very well.
(D) He is hoping to stop before the end of (D) She and Bob haven’t been together for
the day. a long time.

3. (A) They can’t make anything to take to 10. (A) He came for her at her house and took
the dinner. her to the hospital.
(B) They are afraid of going to the dinner. (B) He carried her from her house to the
(C) They might not be able to go to the hospital.
dinner tonight. (C) He drove to the hospital in his truck.
(D) They fear they will be late to tonight’s (D) He chose her to work with him at the
dinner. hospital.

4. (A) They were unable to do what they had 11. (A) Marsha is learning how to make ropes
planned because it started raining. where she works.
(B) They slipped and fell through the ice (B) Marsha is learning what to do at her
when it started to melt in the rain. new job.
(C) They planned to take their ski trip in (C) Marsha feels uncomfortable where she
spite of the rain. works.
(D) They weren’t through skiing when it (D) All of Marsha’s work is tied up.
started to rain.

5. (A) He scratched his eye on the necklace.


(B) It hurt eyes to look at the diamond
necklace.
(C) He didn’t like looking at the diamond
necklace.
(D) That diamond necklace really attracted
his attention.

6. (A) She doesn’t take care of her new bike.


(B) It doesn’t bother her if you borrow her
new bike.
(C) She doesn’t really like her new bike
very much.
(D) Her new bike doesn’t need much
maintenance.

7. (A) Write this information down.


(B) Forget this information.
(C) Not give anyone else this information.
(D) Give this information to him.

Listening Section | 9
ELPT_ Level 2

Practice with words with many meanings


Read the four answer choices in each question set. Listen to each sentence on the tape. Circle the
letter of the sentence that is closest in meaning to the sentence you hear on the tape.

1. (A) Please check this for me. 4. (A) Andrew wasn’t sure what to call you.
(B) May I please have money in exchange (B) You didn’t answer when Andrew
for this check. telephoned last night.
(C) I will put a check next to each incorrect (C) Andrew visited you last night but you
answer. weren’t home.
(D) I will not be able to pay with cash. (D) The man you met last night is called
Andrew.
2. (A) He found a lovely source of water at
the bottom of the hill. 5. (A) This restaurant is for people with
(B) He hurt his foot this spring while money.
running down the hill. (B) This restaurant is beautifully
(C) It has been a beautiful spring for him. decorated.
(D) He accidentally uncovered a beautiful (C) They serve very rich food at this
spring with his foot. restaurant.
(D) The owners of this restaurant are rich.
3. (A) Clara wears as much makeup as
possible to work every day. 6. (A) John is associated with me on this
(B) It will not be possible for Clara to come project.
to work today. (B) It is difficult for me to associate with
(C) Clara will make it upstairs to work as John.
soon as she can. (C) My colleague has been working with
(D) Clara wants to do the work she missed John.
right away. (D) John’s project is related to mine.

Practice with Sound-Alikes


Listen carefully to each dialogue and the corresponding question. Circle the letter that best answers
the question you hear.

1. (A) They took a seat by the door. 5. (A) She can borrow his book, but he must
(B) They looked for it on the floor. have it back.
(C) They have seen that concert before. (B) She can borrow his story, but she can’t
(D) They sat down on the floor. keep it.
(C) He’ll send her his story about the
2. (A) They tried very hard to leave that thirsty man.
place. (D) She can borrow his book on Thursday.
(B) They hid the rest in another place.
(C) They tried very hard to win the 6. (A) She has an elderly cousin who gets up
competition. precisely at 6:15 A.M.
(D) They did the rest before the race. (B) Her young cousin gets up each day at
exactly 6:50 A.M.
3. (A) Larry and Dave aren’t going. (C) Her eighteen-year-old cousin nicely
(B) The days are getting long. gets up each day at 6:50 A.M.
(C) Larry has been gone for days. (D) Her eighty-year-old cousin gets up
(D) Larry and Dave are incompatible. every day at 6:50 A.M.

4. (A) The preacher was shocked when he 7. (A) Tomorrow Anne is going to the
opened the door. neighbor to bake.
(B) The teacher found chalk when she (B) Anne is with her neighbor at lake.
opened the drawer. (C) Anne has gone to the neighbor to
(C) Each of her talks was behind closed borrow a garden tool.
doors. (D) Anne is going to her neighbor’s lake
(D) The teacher was shocked when she tomorrow.
opened the drawer.

Listening Section | 10
ELPT_ Level 2

8. (A) On Sundays, he sees her walking in 10. (A) She wants advancement.
the garden. (B) She lives by the ocean near here.
(B) On Sundays, she works in the garden. (C) She doesn’t like the motion here.
(C) She works for the Gardiners some day. (D) She’ll soon hear about her promotion.
(D) Some days he sees her walking in the
garden. 11. (A) Bill gets by at the store.
(B) Bill got a good buy at the store.
9. (A) She troubled her aunt for tea a lot (C) We filled the safe by the door.
before she returned. (D) Bill bid us farewell at the entrance.
(B) She had a lot of trouble with her aunt
when she was fourteen.
(C) She traveled a lot with her aunt before
she was fourteen.
(D) She and her aunt took a lot of trips
together before she was forty.

Practice with Negatives


Listen carefully to each dialogue and the corresponding question. Circle the letter that best answers
the question you hear.

1. (A) Carl said he would improve his grade.


(B) Carl had a great job.
(C) Carl’s grade wasn’t very good.
(D) Carl wasn’t unhappy about his grade.

2. (A) Visits from his parents are rare.


(B) He visits his parents often.
(C) His parents live near here.
(D) He doesn’t see his parents very often.

3. (A) Nothing is surprising.


(B) This news is surprising.
(C) She is surprised by these views.
(D) Why is the news so surprising?

4. (A) We don’t really like losing the game.


(B) We aren’t likely to win the game.
(C) We will probably win the game.
(D) This is a game that we don’t like.

5. (A) She didn’t think Andrew was feeling well.


(B) Andrew isn’t very well thought of.
(C) She doesn’t understand why Andrew isn’t feeling well.
(D) She thought Andrew was feeling much better.

6. (A) The results of the experiment were incorrectly analyzed.


(B) The scientist accurately interpreted the result of his experiment.
(C) The scientist hasn’t had time to analyze the result of his experiment.
(D) The scientist needs to re-analyze the results of his experiment.

Listening Section | 11
ELPT_ Level 2

Practice with Who Does What


Circle the letter of the sentence that is closest in meaning to the sentence you hear on tape.

1. (A) Greg got a cake for Nancy. 4. (A) I’ll have someone repair my car as
(B) Greg and Nancy took the truck to the soon as I can afford it.
lake. (B) I’m in a terrible fix for money.
(C) Nancy went and got the cake. (C) I’m going to work on my car as soon as
(D) Greg will take the truck later. I can.
(D) I have to find a new car as quickly as
2. (A) Harry delivered the car on Tuesday. possible.
(B) Harry wanted to see his new car today.
(C) Harry arranged for the delivery of his 5. (A) The hiker helped the park employees.
car on Tuesday. (B) Many park employees helped the
(D) Harry asked about the condition of his hiker.
new car. (C) The employees parked the hiker’s car.
(D) The hiker dedicated the park to the
3. (A) Karen told her to start her new job next employees.
week.
(B) Karen has several forms to fill out 6. (A) Her saw works well for chopping wood.
before she begins work next week. (B) She saw me chopping wood.
(C) Karen said that she started felling (C) She was chopping wood when I saw
weak at work. her.
(D) Karen will begin her new job next (D) I saw her when I was shopping.
week.

Practice with Situations


Listen carefully to each dialogue and the corresponding question. Circle the letter that the best
answers the question you hear.

1. (A) Dentist.
(B) Nurse.
(C) Weather reporter.
(D) Teacher.

2. (A) In a library.
(B) In a classroom.
(C) In a bookstore.
(D) In lawyer’s office.

3. (A) She thinks the man’s luggage will arrive soon.


(B) She wants to turn at the next street.
(C) She is too short to see what’s in the bag.
(D) She will pick up the man’s groceries for him.

4. (A) A librarian.
(B) A teacher.
(C) A student.
(D) The woman’s best friend.

5. (A) In a movie theater.


(B) In airport.
(C) In a car.
(D) In Chicago.

6. (A) She would like to have her coat back.


(B) Her house doesn’t need another coat of paint.
(C) She wants the man to pay her now.
(D) She would like to buy the coat she has on.

Listening Section | 12
ELPT_ Level 2

REVIEW PART A
Directions: In this checkpoint test you will hear short conversations between two people. After each
conversation, you will hear a question about the conversation. The conversations and questions will
not be repeated. After you hear a question, read the four possible answers in your book and choose
the best answer. Then, circle the letter that corresponds to the answer you have chosen.

1. (A) Receptionist 9. (A) She will try to help the man find a
(B) Teacher good job.
(C) Doctor (B) The descent is too steep for her bike.
(D) Librarian (C) She will be able to ride her bike to
work.
2. (A) She doesn’t know the house. (D) She can’t purchase a bike until she
(B) She agrees with the man. earns some money.
(C) She likes the house very much.
(D) She doesn’t know what to say. 10. (A) Anxiety about exams is keeping the
man awake.
3. (A) Larry didn’t want to borrow the book. (B) Final exams are nothing to worry
(B) Larry hadn’t read the book yet. about.
(C) Larry had lost the book. (C) She’s finally going to take her exam
(D) Larry found the book he had lost. next week.
(D) The man must be careful not to fall
4. (A) She didn’t want to see Peter. asleep during exams.
(B) She saw Peter after she left.
(C) She arrived after Peter left. 11. (A) The man thinks buying a used car is a
(D) She will miss Peter. bad idea.
(B) The man wants to borrow Jane’s car.
5. (A) It is very frank. (C) The man thinks Jane should buy a
(B) Frank typed it. car.
(C) It is the wrong type. (D) Jane will find the man a used car
(D) He got it from Frank. after all.

6. (A) She has never had such a delicious 12. (A) He doesn’t want to proofread the
dessert before. woman’s paper.
(B) She loves living in the desert. (B) He doesn’t see very well.
(C) She is quite sure there are better (C) He will be back in two days
desserts available. (D) He isn’t there.
(D) She can’t understand why the man
likes the desert so much. 13. (A) It is violent.
(B) The volume is too high.
7. (A) In a bank (C) He wants to think about it.
(B) In an attorney’s office (D) It helps him think.
(C) In a math class
(D) In an art class 14. (A) A coach
(B) An athlete
8. (A) Bob and Judy shouldn’t go away for (C) A close friend of the woman’s
so long. (D) A teacher
(B) Bob is feeling too weak to go
anywhere. 15. (A) Has everyone been invited to the
(C) They should also tell Bob about their wedding?
plans. (B) Should she invite her family?
(D) He doesn’t want Bob to know about (C) Did the man take her family to the
their plans. wedding?
(D) Is her family included in the people
counted?

Listening Section | 13
ELPT_ Level 2

Exercise Part B & C


Directions: In this checkpoint test you will hear several short talks and longer conversations. After
each of them, you will hear several questions. The talks and conversations will not be repeated.
After you hear a question, read the four possible answers in your book and choose the best answer.
Then, circle the letter that corresponds to the answer you have chosen. Answer all questions on the
basis of what is stated or implied in the talk or conversation.

1. (A) The man’s trip


(B) Buying luggage 10. (A) The top of a tree
(C) How to file a baggage claim (B) A very hot fire
(D) Travel by train. (C) Tree roots that grow close to the
surface
2. (A) He walked. (D) The highest flame in a fire
(B) He took a taxi.
(C) He drove. 11. (A) He needs some experience acting
(D) Someone picked him up. (B) The drama club needs some male
actors
3. (A) Her baggage has been lost. (C) She thinks he’s the best actor on
(B) She has been delayed. campus
(C) She has taken a taxi. (D) She wants him to meet the playwright
(D) She has taken the man to the airport.
12. (A) He doesn’t enjoy acting
4. (A) He forgot. (B) He doesn’t like comedies
(B) He didn’t think it would help. (C) He wants to be free on Friday nights
(C) He was too tired. (D) He doesn’t think he’ll have enough
(D) He found his baggage. time

5. (A) Buy a ticket 13. (A) A student


(B) Go to the airport (B) The woman
(C) Get some rest (C) A professor
(D) Look for the man’s ticket (D) A famous playwright

6. (A) Preventing Forest Fires 14. (A) One or two times a week
(B) Factors Affecting a Tree’s Resistance (B) On Friday nights
to Fire (C) Every day
(C) The Redwood’s Ability to Withstand (D) Whenever they have time
Fire
(D) Ground Fires and Crown Fires 15. (A) Write a play as soon as possible
(B) Meet the woman at 6:30
7. (A) 100 degrees centigrade (C) Find a part that is perfect for him
(B) 20 degrees centigrade (D) Wait a week then decide what to do
(C) 40 degrees centigrade
(D) 120 degrees centigrade 16. (A) To document the first contact between
Native people of the Americas and
8. (A) They have deep roots Europeans
(B) They have few leaves (B) To demonstrate how virtual reality and
(C) They have thick bark other computer techniques can be
(D) They grow close together used effectively and effortlessly
(C) To cover the rise and the fall of native
9. (A) They are more likely to have large civilizations in Western North America
leaves and Central America before contact
(B) They are susceptible to ground fires with Europeans
(C) They have a high resistance to all (D) To explain how to work a video
types of fire recorder to tape a desired program on
(D) They usually fall down during a fire the television

Listening Section | 14
ELPT_ Level 2

19. (A) Read about it instead


17. (A) On Wednesday night (B) Ask a friend to tape it
(B) On Thursday morning (C) Discuss it with someone who watched
(C) On Wednesday morning it
(D) On Thursday night (D) Notify the instructor

18. (A) That it was written and produced by 20. (A) Loan his copy of the program to
Kevin Costner students who want to watch it
(B) That it uses computer animation and (B) Go to the video store and rent the
virtual reality program for viewing in class
(C) That it discusses in depth the key (C) Have another student tape the
cities of the empires program for those who cannot watch it
(D) That it uses authentic musical (D) Take notes on the program and
instruments and songs distribute them to those who are not
able to watch it

Listening Section | 15
ELPT_ Level 2

Practice Test 1

Part A
Directions: In Part A you will hear short conversations between two people. After each conversation,
you will hear a question about the conversation. The conversations and questions will not be repeated.
After you hear a question, read the four possible answers in your test book and choose the best
answer. Then, on your answer sheet, find the number of the question and fill in the space that
corresponds to the letter of the answer you have chosen.

1. (A) Her concerns were expressed.


(B) She wonders if the concert was good.
(C) The music was fantastic. 8. (A) She is found of Joe.
(D) She wandered about during the night. (B) Joe is not lost anymore.
(C) He visits Joe regularly.
2. (A) In a bus station. (D) The call was already made.
(B) In a post office
(C) In an airport. 9. (A) He was quite fortunate.
(D) In a travel agency. (B) He lacked of the opportunity to play
the game.
3. (A) He spends half his time with his (C) He didn’t actually win the game.
daughter. (D) He came sometimes after Luke.
(B) His daughter’s under a year old.
(C) His daughter will start school in a 10. (A) It’s quite clean.
year and a half. (B) It doesn’t move.
(D) His daughter’s eighteen months old. (C) It’s clearly better.
(D) It’s rather dirty.
4. (A) He’s on the basket team.
(B) He doesn’t even like to play 11. (A) A bus driver.
basketball. (B) An art teacher.
(C) He’s too short to make the team. (C) A flight attendant.
(D) He thinks he’s tall enough. (D) A travel agent.

5. (A) She’s thinking about her grade in the 12. (A) His desk is made of metal.
seminar. (B) He never meddles with his office
(B) The seminar is quite realistic. workers.
(C) The seminar will take a great deal of (C) His desk is on the right side of the
time. office.
(D) She agrees with the man about the (D) He works in the center of the office.
seminar.
13. (A) Learning something about golf.
6. (A) The location of the meeting. (B) Taking fewer exercises than he has
(B) The purpose of the meeting. in the past.
(C) Who will attend the meeting. (C) Letting her play golf instead of him.
(D) The time the meeting starts. (D) Going to play golf together.

7. (A) Preparing dinner later. 14. (A) He does not live nearby.
(B) Going to a restaurant. (B) They will go later.
(C) Cooking a full dinner. (C) It is not time to deliver the paper.
(D) Eating dinner outside in the garden. (D) He does not have time to read.

Listening Section | 16
ELPT_ Level 2

15. (A) She will assign the homework 23. (A) He’s a bit too strong.
tomorrow. (B) He’s bitter about the accident.
(B) The man can do the homework next (C) He’s still weak.
week. (D) He feels the accident was not his
(C) She will return the homework next fault.
week.
(D) The assignment must be turned in 24. (A) There seems to be a problem with
tomorrow. the motor.
(B) He doesn’t want to be an engineer.
16. (A) The woman is really beautiful. (C) The music on the car stereo is good.
(B) The woman should repeat what she (D) He likes the sound of the engine.
said.
(C) He shares the woman’s opinion. 25. (A) An artist.
(D) He has time this year. (B) A tour guide.
(C) A teacher.
17. (A) in a shoe store. (D) A pilot.
(B) in a motel.
(C) in a clothing store. 26. (A) Buying some cream for coffee.
(D) in a storage facility. (B) Cleaning out the garage.
(C) Painting the apartment walls green.
18. (A) She is going to the supermarket. (D) Putting the apartment in order.
(B) She works in grocery store.
(C) She’s not going out. 27. (A) He was happy to write the check.
(D) She doesn’t have enough money for (B) He received a large sum of money.
groceries. (C) He was working in a huge factory.
(D) He found some hidden bones.
19. (A) He didn’t really get the highest.
(B) He’s rather intelligent. 28. (A) She doesn’t need to sore anything.
(C) He’s not done with the exam. (B) She refers shopping nearby.
(D) He’s not even in the class. (C) The sores are all too far away.
(D) She doesn’t want to go shopping.
20. (A) Resting for a new minutes.
(B) Studying a bit longer. 29. (A) The door was closed really hard.
(C) Taking a five-hour break. (B) Alice left because she was hungry.
(D) Studying for shorter periods of time. (C) The door was smoothly sanded.
(D) Alice went out through the door on
21. (A) She needs a new suitcase. the left side.
(B) She’s putting on a suit.
(C) She’s preparing for a trip. 30. (A) Take a bike ride.
(D) She just received a package. (B) Take the high road.
(C) Go for a walk in the forest.
22. (A) It was the last game. (D) Look for firewood.
(B) The dream was really bad.
(C) The man never lets her talk.
(D) She feels the same way as the man.

Listening Section | 17
ELPT_ Level 2

Part B
Direction: In this Part of the test, you will hear longer conversations. After each conversation, you will
hear several questions. The conversation and question will not be repeated. After you hear a
question, read the four possible answers in your book and choose the best answer. Then, on your
answer sheet, find the number of the question and fill in the space that corresponds to the letter of the
answer you have chosen. Remember, you are not allowed to take notes or write in your test book.

31. (A) A new bicycle. 35. (A) Planning a trip.


(B) An inexpensive bicycle. (B) Camping in the woods.
(C) A fast bicycle. (C) Putting up a tent.
(D) A stationary bicycle. (D) Looking at photos.

32. (A) A half mile. 36. (A) It looks comfortable.


(B) A mile. (B) It doesn’t seem very big.
(C) Two miles. (C) Sitting in front of the tent.
(D) Four miles. (D) It’s full of fish.

33. (A) He doesn’t like it 37. (A) Standing in a river.


(B) It doesn’t work very well. (B) Putting up a tent.
(C) It’s broken. (C) It looks funny.
(D) He got a new one. (D) Swimming in the river.

34. (A) Go see his friend’s bicycle. 38. (A) Lots.


(B) See her new apartment. (B) A few.
(C) Walk to school. (C) One.
(D) Buy a new bicycle (D) None.

Listening Section | 18
ELPT_ Level 2

Part C
Direction: In Part C of this section, you will hear some questions. The talks and questions will not be
repeated. After you hear a question, read the four possible answers in your book and choose the best
answer. Then, on your answer sheet, find the number of the question and fill in the space that
corresponds to the letter of the answer you have chosen.

Here is an example.
On the recording, you will hear:
(narrator) Listen to an instructor talk to his class about painting.
(man) artist grant Wood was a guiding force in the school of painting known as
American regionalist, a style reflecting the distinctive characteristics of art from
rural areas of the United States. Wood began drawing animals of the family farm
at the age of three, and when he was thirty-eight, one of his painting received a
remarkable amount of public notice and acclaim. This painting, called American
Gothic, is a starkly simple depiction of serious couple staring directly out at the
viewer.

Now listen to a sample question.

(narrator) what style of painting is known as American regionalist?


In your test book, you will read:
(A) Art from America’s inner cities.
(B) Art from the central region of the United States.
(C) Art from various urban areas in the United States.
(D) Art from rural sections of America.

The best answer to the question, “What style of painting is known as American Regionalist?” is (D),
“Art from rural sections of America”. Therefore, the correct choice is (D).

Now listen to another sample question.


(narrator) What is the name of the Wood’s most successful painting?
In your test book, you will read:
(A) American regionalist.
(B) The family Farm in Iowa.
(C) American Gothic.
(D) A Serious Couple.

The best answer to the question, “What is the name of Wood’s most successful painting?” is (C).
Remember, you are not allowed to take notes or write in your test book.

39. (A) Only three chapters. 41. (A) Half an hour.


(B) Three chemistry books. (B) Fifty minutes.
(C) Lecture notes and parts of the book. (C) An hour.
(D) Only class notes from the lectures. (D) Ninety minute.

40. (A) Only multiple choice. 42. (A) Listen to a lecture


(B) Short and long essays. (B) Study for exam.
(C) Three short essays. (C) Read three chapter.
(D) Essays and multiple choices. (D) Take an exam.

Listening Section | 19
ELPT_ Level 2

43. (A) In a zoo. 47. (A) Artificial plants.


(B) On a boat. (B) Plants in plastics.
(C) In a prison. (C) Plants that resemble plastic.
(D) In a lecture hall. (D) Plants that produce a usable
substance.
44. (A) A type of rock.
(B) A Spanish. 48. (A) It lasts longer.
(C) A prison in San Francisco. (B) It is more artificial.
(D) A kind of bird. (C) It is easy to make in a laboratory.
(D) It is good for the environment.
45. (A) Five.
(B) Ten. 49. (A) It biodegrades slowly.
(C) Twenty-four. (B) It kills plants.
(D) Thirty-nine. (C) People never throw it away.
(D) It is not very strong.
46. (A) It is open to visitors.
(B) It is still in use as a prison. 50. (A) Engineering.
(C) It is closed to the public. (B) Art.
(D) It contains few prisoners. (C) Botany.
(D) Geology

Listening Section | 20
ELPT_ Level 2

Practice Test 2
Part A
Directions: In Part A you will hear short conversations between two people. After each conversation,
you will hear a question about the conversation. The conversations and questions will not be repeated.
After you hear a question, read the four possible answers in your test book and choose the best
answer. Then, on your answer sheet, find the number of the question and fill in the space that
corresponds to the letter of the answer you have chosen.

1. (A) Carla does not live very far away. 8. (A) She needs to use the man’s notes.
(B) What Carla said was unjust (B) Yesterday’s physics class was quite
(C) He does not fear what anyone says boring.
(D) Carla is fairly rude to others. (C) She took some very good notes in
physics class.
2. (A) She thinks it’s an improvement. (D) She would like to lend the man her
(B) The fir trees in it are better. notes.
(C) It resembles the last one.
(D) It is the best the man has ever done. 9. (A) It’s her birthday today.
(B) She’s looking for a birthday gift.
3. (A) He graduated last in his class. (C) She wants to go shopping with her
(B) He is the last person in his family to dad.
graduate. (D) She wants a new wallet for herself.
(C) He doesn’t believe he can improve
gradually. 10. (A) He took a quick trip.
(D) He has finally finished his studies. (B) The big boat was towed through the
water.
4. (A) He thought the dress was so chic. (C) There was coal in the water.
(B) He was surprised the dress was not (D) He didn’t go for a swim.
expensive.
(C) He would like to know what color 11. (A) She just left her sister’s house.
dress it was. (B) Her sister left the sweater behind.
(D) The dress was not cheap. (C) She believes her sweater was left at
her sister’s house.
(D) She doesn’t know where her sister
lives.
5. (A) Leave the car somewhere else.
(B) Ignore the parking tickets. 12. (A) She doesn’t have time to complete
(C) Add more money to the meter. additional reports.
(D) Pay the parking attendant. (B) She cannot finish the reports that she
is already working on.
6. (A) He does not like to hold too many (C) She is scared of having responsibility
books at one time. for the reports.
(B) There is no bookstore in his (D) It is not time for the accounting
neighborhood. reports to be compiled
(C) It’s not possible to obtain the book
yet. 13. (A) He’s had enough exercise.
(D) He needs to talk to someone at the (B) He’s going to give himself a reward
bookstore. for the hard work.
(C) He’s going to stay on for quite some
7. (A) It was incomplete. time.
(B) It finished on time. (D) He would like to give the woman an
(C) It was about honor. exercise machine as a gift
(D) It was too long.
14. (A) He cannot see the huge waves.
(B) The waves are not coming in.
(C) He would like the woman to repeat
what she said.
(D) He agrees with the woman.

Listening Section | 21
ELPT_ Level 2

15. (A) The exam was postponed. 23. (A) The article was placed on reserve.
(B) The man should have studied harder. (B) The woman must ask the professor
(C) Night is the best time to study for for a copy.
exams. (C) The woman should look through a
(D) She is completely prepared for the number of journals in the library.
exams. (D) He has reservations about the
information in the article.
16. (A) Students who want to change
schedules 24. (A) He needs to take a nap.
(B) It is only possible to make four (B) He hopes the woman will help him to
changes in the schedule. calm down.
(C) It is necessary to submit the form (C) The woman just woke him up.
quickly. (D) He is extremely relaxed.
(D) Problems occur when people don’t
wait their turn. 25. (A) She doesn’t think the news report is
false.
17. (A) In a mine. (B) She has never before reported on the
(B) In a jewelry store. news.
(C) In a clothing store. (C) She never watches the news on
(D) In a bank. television.
(D) She shares the man’s opinion about
18. (A) A visit to the woman’s family. the report.
(B) The telephone bill.
(C) The cost of a new telephone. 26. (A) Management will offer pay raises on
(D) How far away the woman’s family Friday.
lives. (B) The policy has not yet been decided.
(C) The manager is full of hot air.
19. (A) She hasn’t met her new boss yet. (D) The plane has not yet landed.
(B) She has a good opinion of her boss. 27. (A) He doesn’t believe that it is really
(C) Her boss has asked her about her snowing.
impressions of the company. (B) The snow had been predicted.
(D) Her boss has been putting a lot of (C) The exact amount of snow is unclear.
pressure on her. (D) He expected the woman to go out in
the snow.
20. (A) The recital starts in three years.
(B) He intends to recite three different 28. (A) She’s going to take the test over
poems. again.
(C) He received a citation on the third of (B) She thinks she did a good job on the
the month. exam.
(D) He thinks the performance begins at (C) She has not yet taken the literature
three. exam.
(D) She’s unhappy with how she did.
21. (A) Choose a new dentist.
(B) Cure the pain himself. 29. (A) The door was unlocked.
(C) Make an appointment with his (B) It was better to wait outside.
dentist. (C) He could not open the door.
(D) Ask his dentist about the right way to (D) He needed to take a walk.
brush.
30. (A) He nailed the door shut.
22. (A) It is almost five o’clock. (B) He is heading home.
(B) The man doesn’t really need the (C) He hit himself in the head.
stamps. (D) He is absolutely correct.
(C) It is a long way to the post office.
(D) It would be better to go after five
o’clock.

Listening Section | 22
ELPT_ Level 2

Part B
Directions: In this part of the test, you will hear longer conversations. After each conversation, you will
hear several questions. The conversations and questions will not be repeated.

After you hear a question, read the four possible answers in your test book and choose the best
answer. Then, on your answer sheet, find the number of the question and fill in the space that
corresponds to the letter of the answer you have chosen.

Remember, you are not allowed to take notes or write in your test book.

31. (A) The haircut is unusually short. 35. (A) Every evening.
(B) This is Bob’s first haircut. (B) Every week.
(C) Bob doesn’t know who gave him the (C) Every Sunday.
haircut. (D) Every month.
(D) After the haircut, Bob’s hair still
touches the floor. 36. (A) That she was eighty-five years old.
(B) That a storm was coming.
32. (A) It is just what he wanted. (C) That she was under a great deal of
(B) He enjoys having the latest style. pressure.
(C) He dislikes it immensely. (D) That she wanted to become a
(D) He thinks it will be cool in the weather forecaster.
summer.
37. (A) In her bones.
33. (A) A broken mirror. (B) In her ears.
(B) The hairstylist. (C) In her legs.
(C) The scissors used to cut his hair. (D) In her head.
(D) Piles of his hair.
38. (A) Call his great-grandmother less
34. (A) You should become a hairstylist. often.
(B) Please put it back on. (B) Watch the weather forecasts with his
(C) It’ll grow back. great-grandmother.
(D) It won’t grow fast enough. (C) Help his great-grandmother relieve
some of her pressures.
(D) Believe his great-grandmother’s
predictions about the weather.

Part C
Directions: In this part of the test, you will hear several talks. After each talk, you will hear some
questions. The talks and questions will not be repeated.

After you hear a question, you will read the four possible answers in your test book and choose the
best answer. Then, on your answer sheet, find the number of the question and fill in the space that
corresponds to the letter of the answer you have chosen.

39. (A) In a car. 42. (A) Taking photographs.


(B) On a hike. (B) Getting closer to the crocodiles.
(C) On a tram. (C) Exploring the water’s edge.
(D) In a lecture hall. (D) Getting off the tram.

40. (A) It means they have big tears. 43. (A) Water Sports.
(B) It means they like to swim. (B) Physics.
(C) It means they look like crocodiles. (C) American History.
(D) It means they are pretending to be (D) Psychology.
sad.
44. (A) To cut.
41. (A) They are sad. (B) To move fast.
(B) They are warming themselves. (C) To steer a boat.
(C) They are getting rid of salt. (D) To build a ship.
(D) They regret their actions.

Listening Section | 23
ELPT_ Level 2

45. (A) To bring tea from China. 48. (A) He or she would see butterflies.
(B) To transport gold to California. (B) He or she would break a leg.
(C) To trade with the British. (C) He or she would have shaky knees.
(D) To sail the American river system (D) He or she would stop breathing.

46. (A) A reading assignment. 49. (A) By staring at the audience.


(B) A quiz on Friday. (B) By breathing shallowly.
(C) A research paper for the end of the (C) By thinking about possible negative
semester. outcomes.
(D) Some written homework. (D) By focusing on what needs to be
done.
47. (A) Writers.
(B) Actors. 50. (A) At two o’clock.
(C) Athletes. (B) At four o’clock.
(D) Musicians. (C) At six o’clock.
(D) At eight o’clock.

Listening Section | 24
ELPT_ Level 2

Practice Test 3
In this section, you will demonstrate your skills in understanding spoken English. There are three parts
in the Listening Comprehension, with different tasks in each.

Part A
Directions: In Part A you will hear short conversations between two speakers. At the end of each
conversation, a third speaker will ask a question about what the first two speakers said. Each
conversation and each question will be spoken only one time. Therefore, you must listen carefully to
understand what each speaker says. After you hear a conversation and the question, read the four
choices and select the one that is the best answer to the question the speaker asked. Then, on your
answer sheet, find the number of the question and blacken the space that corresponds to the letter for
the answer you have chosen. Blacken the space completely so that the letter inside the space does
not show.

Listen to the following example.

Sample Answer: D
On the recording, you hear:
(Man) Does the car need to be filled?
(Woman) Mary stopped at the gas station on her way home.
(Narrator) What does the woman mean?

In your test book, you will read:


(A) Mary bought some food.
(B) Mary had car trouble.
(C) Mary went shopping.
(D) Mary bought some gas.

From the conversation you learn that Mary stopped at the gas station on her way home. The best
answer to the question "Does the car need to be filled?" is (D), "Mary bought some gas.” Therefore,
the correct answer is (D).

Now let us begin Part A with question number 1.

1. (A) Peter didn't pay Bill two times. 4. (A) It’s better to change this job.
(B) Peter didn't pay the phone company. (B) Betty decided to take the job.
(C) Peter is the second in line to pay. (C) In my opinion, Betty should change
(D) Peter forgot to call Bill. her.
(D) I thought Betty was taking John with
2. (A) The course is closed for registration. her.
(B) The instructor decides who can
enroll. 5. (A) If she dropped Jack off.
(C) Registration for this course is (B) If she took his jacket to be cleaned.
permitted. (C) If she was leaving Jack.
(D) The instructor doesn't give students (D) If she had cleaned his jacket yet.
permission.
6. (A) The teachers didn't go to the
3. (A) We expected only fourteen people to meeting.
come. (B) The meeting on Wednesday was
(B) Forty people came to work in the crowded.
shop. (C) The teachers knew about the
(C) More people came than had been meeting.
acted. (D) Everyone forgot about the meeting.
(D) We expanded the shop to include
forty people. 7. (A) He was nice to give up his part.
(B) He gave a party for the man.
(C) was kind to do his part.
(D) was kind to come to the party.

Listening Section | 25
ELPT_ Level 2

8. (A) Pharmacists shouldn't carry boxes to 17. (A) After he broke a glass, he had to use
the station. cup.
(B) Paper goods are not usually sold in (B) After the flat, he's been using a spare
drug stores. tire.
(C) Mobile pharmacies are not stationed (C) When he broke the glass, he cut
here. himself.
(D) The pharmacy needs to order paper (D) He has been using different glasses.
goods.
18. (A) My brother had an idea for our
9. (A) The exam seems to be easier than it father’s gift.
first appeared. (B) My brother wants to buy our father a
(B) He thinks the exam appears to cover pool.
the material. (C) We wanted to bake a pie for our
(C) He thinks they made the exam father.
difficult on purpose. (D) We celebrated our father’s birthday.
(D) The exam is more difficult than he
thought. 19. (A) Ann will be 21 next month.
(B) No one now lives next to Ann.
10. (A) The number of the unemployed has (C) Ann moved here three weeks ago.
been increasing. (D) Ann is going on a 21-day vacation.
(B) Drama clubs have been helping the
unemployed. 20. (A) If he could sell her a product to
(C) Unemployed actors can find work in remove grease.
drama. (B) If he knew he had a stain.
(D) Dramas about the unemployed are (C) If he wanted to remove his shirt.
gaining popularity. (D) If she had a crease in her shirt.

11. (A) Linda's grades don't leave much 21. (A) She approves.
room for improvement. (B) He is good at tennis.
(B) Linda deserves better grades in her (C) He was nice to clean the house.
courses. (D) She wants to come, too.
(C) Linda's grades aren't as high as they
should be. 22. (A) He is going out of town.
(D) Linda's grades should be left alone. (B) He is expecting guests.
(C) He is changing companies.
12. (A) She had to add salt to her (D) He is moving to another town.
hamburger.
(B) She had a hamburger at 1:30. 23. (A) At an amusement park.
(C) She needs something to drink. (B) In a restaurant.
(D) Her hamburger was excellent. (C) On the sidewalk.
(D) Near a parking meter.
13. (A) They dropped her sandwich.
(B) She didn't have food on her mind. 24. (A) Buying vegetables.
(C) They will bring her a sandwich. (B) Planting a garden.
(D) She should be on her way home. (C) Cooking a meal.
(D) Loading a truck.
14. (A) Doug is too old to go on a boat.
(B) Ducks can't be transported by boat. 25. (A) Can you hear this noise?
(C) Doug doesn't like to vote. (B) Do you know this sound?
(D) Doug is too young to vote. (C) Where would you prefer?
(D) Where is this sound coming from?
15. (A) Harry is working to buy new shoes.
(B) Harry's shoes are at school. 26. (A) At a craft show
(C) Harry is a student. (B) At a bookstore
(D) Harry walks to the seashore. (C) At a hardware store
(D) At a video rental shop
16. (A) There is no room on the third floor.
(B) Room 7 is ready for the third 27. (A) A gas station attendant
meeting. (B) A university course grader
(C) The room is on a different floor. (C) An income tax accountant
(D) The floor in this room is dirty. (D) A technical team leader

Listening Section | 26
ELPT_ Level 2

29. (A) Go for dinner


28. (A) She is upset that she forgot to send (B) Order Italian food
the card. (C) Mail her report
(B) She thinks the man should share (D) Finish her report
responsibility.
(C) She has been forgetting a lot of 30. (A) She believes her services should be
things lately. noticed.
(D) She is asking whether the man sent (B) She is taking a two-week leave.
his own card. (C) She was promoted to the position of
a manager.
(D) She is planning to take another job.

Part B

Directions: In this part of the test, you will hear longer conversations. After each conversation, you will
hear several questions. The conversations and questions will not be repeated.
After you hear a question, read the four choices in your test book and select the best answer. Then,
on your answer sheet, find the number of the question and fill in the space that corresponds to the
letter of the answer you have chosen. Remember, you are not allowed to take notes or write in your
test book.

Listen to the following example:

You will hear:


You will read:
(A) He has changed jobs. Sample Answer: C
(B) He has two children.
(C) He has two jobs.
(D) He is looking for a job.

From the conversation you learn that Tom has taken an additional job. The best answer to the
question "Why is Tom tired?" is (C), "He has two jobs." Therefore, the correct answer is C.

31. (A) In a film studio 35. (A) At a county fair


(B) In a record company (B) In the woods
(C) At an art gallery (C) On a sidewalk
(D) At a movie theater (D) On a playground

32. (A) He thinks his children should watch 36. (A) In the evening
educational programs. (B) In the morning
(B) His daughters are too old to watch (C) In the fall
cartoons. (D) In the spring
(C) He isn't a child any longer.
(D) He doesn't want to watch cartoons. 37. (A) They shed their leaves in the fall.
(B) There are fewer daylight hours.
33. (A) The drawings are made. (C) Young trees require a blanket.
(B) The story is created. (D) Red and yellow are prettier that
(C) The shots are framed. green.
(D) The artists are organized.
38. (A) She doesn't care about his
34. (A) To make cartoons a form of graphic explanation.
art (B) She doesn't understand scientific.
(B) To enhance the visual and auditory (C) She doesn't believe the man.
elements (D) She doesn't like autumn.
(C) To add action to animation
(D) To speed up the feature plot

Listening Section | 27
ELPT_ Level 2

Part C
Directions: In Part C you will hear short lectures and conversations. At the end of each, you will be
asked several questions. Each lecture or conversation and each question will be spoken only one
time. For this reason, you must listen carefully to understand what each speaker says. After you hear
a question, read the four possible choices and select the one that best answers the question the
speaker asked. Then, on your answer sheet, find the number of the question and blacken the space
that corresponds to the letter for the answer you have chosen. Answer all questions according to what
is stated or implied in the lecture or conversation.

Listen to this sample talk.


You will hear:
Now listen to the following example.
You will hear: Sample Answer: B
You will read:
(A) By cars and carriages
(B) By bicycles, trains, and carriages
(C) On foot and by boat
(D) On board ships and trains

The best answer to the question "According to the speaker, how did people travel before the invention
of the automobile?" is (B), "By bicycles, trains, and carriages." Therefore, the correct answer is (B).
Now listen to another sample question.
(Narrator): Approximately how many people are employed in the automobile service industry?

You will read:


(A) One million Sample Answer: C
(B) Ten million
(C) Twelve million
(D) Ninety million

The best answer to the question "Approximately how many people are employed in the automobile
service industry?" is (C), "Twelve million." Therefore, the correct answer is (C).
You are not allowed to make notes during the test.

39. (A) It changes climatic conditions. 44. (A) Business and finance
(B) It controls the indoor environment. (B) Law and science
(C) It circulates water particles in the air. (C) Sports
(D) It reduces overhead expenditures. (D) Chief editors

40. (A) By cleaning off dirt and dust 45. (A) Copy editing
(B) By modifying the temperature (B) Approaching people
(C) By removing and adding moisture (C) Writing
(D) By bringing in and taking out air (D) Typing

41. (A) To replace it with fresh air 46. (A) Many reports have been filed.
(B) To create a breeze (B) Many reporters have been laid off.
(C) To retard heat conduction (C) Many publications have closed.
(D) To remove local moisture (D) Many are hiring new specialists.

42. (A) Hardly ever in cold regions 47. (A) They will stay sluggish.
(B) Exclusively in business settings (B) They may continue to decrease.
(C) Basically as it does in summer (C) They are likely to expand.
(D) Mainly as needed for repairs (D) They may be difficult to break into.

43. (A) They can dress comfortably. 48. (A) It represents an essential supply of
(B) They become tired. water.
(C) They feel alert. (B) Its sources can be renewed
(D) They stay healthy. indefinitely
(C) It always provides a usable source
water.
(D) Its development is important in the
United States.

Listening Section | 28
ELPT_ Level 2

49. (A) Increased rain and agricultural 50. (A) Several years
irrigation (B) Centuries
(B) Increased use and contamination (C) Dozens of years
(C) Airborne bacteria and atmospheric (D) Thousands of years
oxygen
(D) Surface runoff and turbulence

Listening Section | 29
ELPT_Level 2
ELPT_ Level 2

The purpose of Section Two of the ELPT is to test your knowledge of the structure of standard written
English. The language tested in this section is mostly formal, not conversational. Many of the
sentence topics used in this section are of a general academic nature.

The Structure questions in Section Two of the ELPT measure your understanding of basic grammar.
There are 15 incomplete sentences in this part of the test. Four possible completions are provided in
the answer choices for each of the sentences. You must choose the one correct answer that complete
the sentences.

The Written Expression questions in Section Two of the ELPT measure your understanding of the
grammar written English. There are 25 sentences in this part of the test. Each sentence has four
underlined segments. You must choose the one incorrect segment.

Careful review of grammar and practice with Structure and Written Expression questions outlined
below will help you prepare for Section Two of the ELPT. General language study inside and outside
the classroom will develop your overall mastery of English grammar. The following points will help you
master the specific points of grammar tested on the test. These combined types of study will greatly
increase your chances to succeed on the ELPT.

1. SUBJECTS, OBJECTS AND NOUN COMPLEMENTS

All of the English sentences tested in Section Two of the ELPT contain subjects. In addition, many of
them contain objects (direct, indirect, or objects of prepositions) and noun complements.
The subjects, objects, and noun complements of English sentences normally occur as noun (phrase),
pronoun, gerund (phrase), infinitive (phrase), or noun clause.

Subject: Doer of the action in a sentence

Example: Andi has breakfast at 07.00 in the morning.

Object:
1. Direct receiver of the action of a sentence (DO)
2. Indirect receiver of the action of a sentence (IO)
3. Object of a preposition (O of Prep)

Example: I gave Andrew a CD for his birthday


IO DO O of Prep
I gave a CD to Andrew for his birthday
DO O of Prep O of Prep

Noun Complement:
1. Subject identifier—after the verb BE (SC)
2. Object identifier—after the direct object of a sentence (OC)

Example: Drew is a teacher.


SC
They elected him president
OC

The subjects, objects, and noun complements of English sentences normally occur as one of the five
noun structure forms given below.

Noun (phrase): Bali is a beautiful island.


S SC
Pronoun: We love Indonesia.
S
I taught them mathematics last semester.
S DO
Gerund (phrase): Reading books is my favorite hobby.
S
I relax by reading books.
O of Prep

Structure Section | 30
ELPT_ Level 2

Infinitive (phrase): We like to read books.


DO
Noun Clause: Everyone says that you like reading books
DO

Model test:

They didn’t know ___________________

(A) what to do
(B) do
(C) to do what
(D) they should do

What is needed in this sentence? The direct object

Explanation: In this question, the direct object is being tested. Answer (B) is a verb. It cannot be a
direct object. In answer (C), the word order of the direct object is incorrect. In answer (D), the direct
object is incomplete. Answer (A) is the correct answer to the question because it contains all the parts
of a direct object in the correct order.

2. SUBJECTS AND VERBS

The most common types of problems in the structure section of ELPT test have to do with subjects
and verbs. The sentence may miss a subject, a verb, or both a subject and a verb.

Model test:

_____ was backed up for miles on the freeway.

(A) At the beach


(B) In the morning
(C) Traffic
(D) Cars

Explanation: In this example, there is a verb (was), but there is no subject. Answer (C) is the best
answer because it is a singular subject that agrees with the singular verb was.

3. EXPRESSIONS OF QUANTITY

Some English expressions of quantity are used only with countable nouns (nouns which can be made
plural, e.g. boy – boys). Other expressions of quantity are used only with uncountable nouns (nouns
that normally cannot be made plural, e.g. happiness. Still other expressions of quantity are used with
both countable and uncountable nouns.

USED WITH COUNTABLE NOUNS USED WITH UNCOUNTABLE NOUNS

many much
number of amount of
few little
a few a little
fewer less
none none
some some
any any
a lot of a lot of
one, two, three,….
Several

Structure Section | 31
ELPT_ Level 2

Model test:

The professor will require us to conduct research, many of which can be done in our own library.
A B C D

What is wrong with this sentence? Many and research are not in agreement.

Explanation: In this question research is an uncountable noun. The expression of quantity, many,
refers back to research, but many cannot be used with uncountable nouns. Answer (C) is the correct
answer to the question because many is incorrect.
Correction: much

4. ARTICLES
The English articles are a/an and the. The article a/an is used with singular, countable indefinite nouns
(nouns referring to things that are new to either the speaker or the listener). The spelling of the article
changes from a to an when the word it precedes begins with a vowel sound.

I need to buy a book.


I need to buy an English book.

The article the is used with singular and plural nouns as well as uncountable nouns. However, the
article, the only occurs with definite nouns (nouns referring to things the speaker and listener already
know about or which are made specific by their use in a sentence.)

The book I wanted is The ELPT Preparation Course.

Model test

1. Patricia is taking a very difficult classes this semester.


A B C D
What is wrong with this sentence? This article and the noun it refers to do not agree.

Explanation: In this question, a singular article is being used with a plural noun. The article a should
only be used with singular, indefinite, countable nouns. In this sentence, a is used with the noun
classes which is plural. Answer (C) is the correct answer because classes is the incorrect form of the
noun.
Correction: class

2. The mongoose is very clever creature that catches snakes.


A B C D

What is wrong with this sentence? An article is missing.

Explanation: In this question, the singular indefinite noun creature needs an article. Answer (B) is the
correct answer to the question because is very clever is incorrect.
Correction: is a very clear

3. A sun was shining brightly when we arrived at the lake.


A B C D

What is wrong with this sentence? An incorrect article has been used.

Explanation: In this question the wrong article has been chosen. Sun is a definite noun. There is only
one sun above the earth; it is definite to everyone. Definite nouns do not take the article a. Answer (A)
is the correct answer to this question because a sun is incorrect.
Correction: the sun

Structure Section | 32
ELPT_ Level 2

5. PERSONAL, POSSESSIVE, AND REFLEXIVE PRONOUNS

In English, pronouns are used to replace or refer to nouns, gerunds, infinitives, and sometimes entire
clauses. Pronouns change form depending on their functions in sentences.

PRONOUN

Personal Possessive
Reflexive
Subject Object Adjective Pronoun

I me my mine myself
You you your yours yourself
He him him his himself
She her her hers herself
It it its ------ itself
We us our ours ourselves
They them their theirs themselves
One one one’s ------- oneself

Model test

Although Alexander liked him new apartment very much, he found it somewhat noisy.
A B C D
What is wrong with this sentence? The possessive adjective form

Explanation: In this question, the pronoun him is in object form. It should be in possessive adjective
form. Answer (A) is the correct answer to this question because him is incorrect.
Correction: his

6. ADJECTIVES AND NOUNS

English nouns are often modified by adjectives. Adjectives usually come before the nouns they modify.
Adjectives cannot be made plural. Nouns must be made plural when they are countable and plural in
meaning.

Model test:

1. A bolt of lightning travels twenty thousands miles in one second.


A B C D

What is wrong with this sentence? The form of an adjective

Explanation: In this question, twenty thousands is a compound adjective modifying miles. As


discussed above, adjectives cannot be made plural in English. Answer (B) is the correct answer to this
question because twenty thousands is incorrect.
Correction: twenty thousand

2. Some trees grow to be over three hundred foot high.


A B C D

What is wrong with this sentence? The form of a noun

Explanation: In this question, three hundred is a compound adjective. It is plural in meaning, and the
nun it modifies should be plural. Foot is not in its plural form in this sentence. Answer (D) is the correct
answer to this question.
Correction: feet

Structure Section | 33
ELPT_ Level 2

3. The paint bright yellow that she chose for her bedroom creates a cheery atmosphere.
A B C D

What is wrong with this sentence? The order of the adjectives

Explanation: In this question, the adjectives bright and yellow come after the noun paint, which they
modify. They should come before this noun. Answer (A) is the correct answer to this question because
paint bright yellow is incorrect.
Correction: bright yellow paint

7. VERB TENSE FORM

In English, only two tenses are marked in the verb alone, present (as in "he sings") and past (as in "he
sang"). Other English language tenses, are marked by other words called auxiliaries.

The basic form (or root of the verb is the form listed in the dictionary and is usually identical to the
first person singular form of the simple present tense (except in the case of the verb "to be"):walk,
paint, think, grow, and sing

The past form of verbs is a little trickier. If the verb is regular (you can create the past form by adding
"-ed", "-d", or "-t" to the present form. When a basic form ends in "-y", you changed the "-y" to "-i-"; in
many cases you should also double terminal consonants before adding "-ed": walked, painted,
thought, grew, and sang).

There are three most common problematic situations with verbs:

 Check what comes after have


Whenever you see the verb have in any of its forms (have, has, having, had), be sure that the
verb that follows it is in the past participle form.

Have + Past Participle

For example:
They had walk* to school. (should be had walked)
We have see* the show (should be have seen)
He has took the test (should be has taken)
Having ate*, he went to school (should be having eaten)
She should have did* the work (should be should have done)

 Check what comes after be


The verb be in any of its forms (am, is, are, was, were, be, been, being) can be followed by
another verb. This verb should be in the present participle or the past participle.

BE + (1) Present Participle


(2) Past Participle

For example:
We are do* our homework (should be are doing)
The homework was do* early (should be was done)
Tom is take* the book (should be is taking)
The book was take* by Tom (should be was taken)

Structure Section | 34
ELPT_ Level 2

 Check what comes after will, would, and other modals.


Whenever you see a modal, such as will, would, shall, should, can, could, may, might, or
must, you should be sure that the verb that follows it is in its base form.

MODAL + Main form of the Verb

For example:
The boat will leaving* at 3.00 (should be will leave)
The doctor may arrives* soon (should be may arrive)
The students must taken* the exam (should be must tak)

Model test :

1. The assignment was did by most of the class


A B C D
What is wrong with this sentence? The formation of the passive

Explanation: In this question, the passive is incorrectly formed. The past form of do has been used
when the past participle is needed. Answer (B) is the correct answer to the question because was did
is incorrect.
Correction: was done

2. The people of this country have been expressed great concern about the environment
A B C D

What is wrong with this sentence? The passive has been used when the active is needed

Explanation: In this question, the passive form of the verb is used. However, the sentence contains a
direct object, great concern. Direct objects come after active verbs, not passive verbs. Answer (C) is
the correct answer to this question because has been expressed is incorrect.
Correction: have expressed

3. The books and toys ____________ to the orphanage.


(A) being donated
(B) that was donated
(C) donated
(D) was donated

What is needed in this sentence? The passive form.

Explanation: In this question, the passive form of the verb is needed. Answer (A) does not contain a
finite verb. Answer (B) contains more than a finite verb. Answer (C) looks like an active finite verb.
However, the books and toys cannot be the subject of donate. A piano cannot donate things. Only
humans can donate. Answer (D), which contains a correctly formed passive verb, indicates that
someone donated the piano. Using this passive verb form, the piano becomes the receiver, not the
doer, of the action. Answer (D) is the correct answer to this question.

8. VERB TENSE MEANING

Simple Present They walk


Present Perfect They have walked
Simple Past They walked
Past Perfect They had walked
Future They will walk
Future Perfect They will have walked

Structure Section | 35
ELPT_ Level 2

The meaning of a tense used in an English sentence must agree with the time meaning of the rest of
the sentence. The time meaning of a sentence is often determined by word or expressions that act as
time markers. Some of the most common English time markers are listed below.

Simple Present today; usually; during


Present Perfect since; for; yet; already; over the past few.......
Simple Past yesterday; in..; during;
Simple Continuous now; tomorrow;
Future tomorrow
Model test

In 1992, Bill Clinton has become president of the United States, beating his opponent by a wide
A B C D
margin.

What is wrong with this sentence? There is no agreement in meaning between the time marker and
the verb.

Explanation: in this sentence, the verb has become does not agree in meaning with the rest of the
sentence. The time marker in 1992 indicates that action in this sentence took place in the past only.
Has become indicates an action that began in the past and has continued to the present. Answer (A)
is the correct answer to the question because has become is incorrect.
Correction: became

9. PASSIVE AND ACTIVE SENTENCES

To help you to minimize the problems in this area, you need to be able to recognize the passive form
of the passive and to be able to determine when a passive verb rather than an active verb is needed in
a sentence.
The difference between an active and a passive verb is that the subject in an active sentence does
the action of the verb, and the subject in a passive sentence receives the action of the verb.
Two changes must be made to convert active to passive sentence:

1. The subject of the active sentence becomes the object of the passive sentence
2. The verb in the passive sentence is formed by putting the helping verb be in the same form in the
active sentence and then adding the past participle of this verb.

Look at this example:


Margaret wrote the letter
SUBJECT OBJECT

The letter was written by Margaret


SUBJECT OBJECT

It should be noted that in a passive sentence, by + object does not need to be included to have a
complete sentence. The following are both examples of correct sentences.
The letter was written yesterday by Margaret.
The letter was written yesterday.

Following are examples of passive errors that might be appear on the test.
1. The portrait is painting* by a famous artist.
(present participle painting is incorrect, it should be the past participle, painted)
2. The project will finished* by Tim
(some form of be is necessary for a passive sentence which has not been included yet, it
should be will be finished)

Structure Section | 36
ELPT_ Level 2

Model test

1. The assignment was done by most of the students.


A B C D
What is wrong with this sentence? The formation of the passive

Explanation: In this question, the passive is incorrectly formed. The past form of know has
been used when the past participle is needed. Answer (B) is the correct answer to the question
because was knew is incorrect.
Correction: was known

2. The people of this country have been expressed great concern about the environment.
A B C D
What is wrong with this sentence? The passive has been used when the active is needed
Explanation: In this question, the passive form of the verb is used. However, the sentence contains a
direct object, great concern. Direct objects come after active verbs, not passive verbs. Answer (C) is
the correct answer to this question because has been expressed is incorrect.
Correction: have expressed

3. The pianos ____________ to the orphanage.


(A) being donated
(B) that were donated
(C) donated
(D) were donated

What is needed in this sentence? The passive

Explanation: In this question, the passive form of the verb is needed. Answer (A) does not contain a
finite verb. Answer (B) contains more than a finite verb. Answer (C) looks like an active finite verb.
However, piano cannot be the subject of donate. A piano cannot donate things. Only humans can
donate. Answer (D), which contains a correctly formed passive verb, indicates that someone donated
the piano. Using this passive verb form, the piano becomes the receiver, not the doer, of the action.
Answer (D) is the correct answer to this question.

10. MODALS AND MODAL-LIKE VERBS

In English sentences, modals (also called modal verbs, modal auxiliary verbs, modal auxiliaries) and
modal-like verbs are followed by the base form of the main verb. They give additional information
about the function of the main verb that follows them.

Model test

1. According to Joe, his puppy can recognizing its own name


A B C D

What is wrong with this sentence? The verb form that follows a modal

Explanation: In this sentence, the verb following can is in its present participle form. It should be in its
base form. Answer (B) is the correct answer to the question because can recognizing is incorrect.
Correction: can recognize

2. The book ________________ in the library.


(A) can be finding
(B) found
(C) to be found
(D) can be found

What is needed in this sentence? A modal passive

Structure Section | 37
ELPT_ Level 2

Explanation: In this question, a modal passive is needed. Answer (A) contains a present participle
where a past participle is needed. Answer (B) and (C) do not contain modal. Answer (D) is the correct
answer to the question because it contains the correct forms for this sentence.

11. SUBJECT – VERB AGREEMENT

Subject-Verb Agreement is actually simple: if the subject of a sentence is singular, the verb must be
singular; if the subject of a sentence is plural, the verb must be plural as well.

For example: The boy walks to school (singular/simple present tense)


The boys walk to school (plural/simple present tense)

However, you should be careful because the questions found in the ELPT test are often tricky. Verbs
don’t always follow the subject immediately. There are often phrases, inverted verbs, and other words
coming in between.

Strategy: Always identify the ‘real’ subject and verb

 Phrases

Prepositional phrase is a group of words beginning with a preposition (Ex.: to the main hall, on my
desk, during the session, etc). It usually contains a noun or verb word, so people often get confused in
identifying the subject and verb of the sentence. Be extra careful when the prepositional phrase is
in the middle of the sentence.

A. The climbers on the sheer face of the mountain needs to be rescued. (incorrect)
B. The climbers on the sheer face of the mountain need to be rescued. (correct)
S Prep. phrase V

In above example, the subject is the climbers, not the mountain. We can see that the word mountain
follows a preposition. It is a prepositional phrase. In the example, there are actually two prepositional
phrases: ‘on the sheer face’ and ‘of the mountain’. Either sheer face or the mountain is not the subject.

Other Examples:

Everybody are going to the theatre. (incorrect)


Everybody is going to the theatre. (correct)
S V
All of the pie was gone.
Singular
All of the pies were gone.
Plural
All of the information was interesting.
Uncount/singular

Model test

My friends and my teacher is coming to my show tonight.


A B C D

What is wrong with this sentence? Subject – Verb agreement

Explanation: In this question, the subject has two parts, my friends and teacher. These two together
make the plural subject. The form is is used for singular subject. Answer B is the correct answer to
the question because is coming is incorrect.
Correction: are coming

Structure Section | 38
ELPT_ Level 2

12. PREPOSITION

Many English sentences contain preposition. ELPT tests your understanding of preposition by:
1. Leaving out necessary prepositions or parts of compound prepositions;
2. Using incorrect prepositions, especially after certain verbs or adjectives;
3. Adding extra words to prepositional phrases.

Model test

1. Anne relaxes by she swimming in the lake every afternoon after class.
A B C D

What is wrong with this sentence? Extra parts have been added to a prepositional phrase.

Explanation: A prepositional phrase consists of a preposition and an object. By she swimming


contains a preposition, a subject, and an object. Answer (B) is the correct answer to the this question
because by she swimming is incorrect
Correction: by swimming

2. Last meeting, We talked ________________ the discovery of gold in California in the 1800s.
(A) on
(B) in
(C) for
(D) about

What is needed in this sentence? A preposition

Explanation: In this question, a verb + preposition combination is being tested. The verb talk is often
followed by the preposition about. Answer (D) is the correct answer to the question.

13. MAIN AND SUBORDINATE CLAUSE MARKERS

All English sentences contain at least one main clause. A main clause contains a subject and a verb and can
stand alone as a sentence

Example: Karina has a new book. = main clause


S V

Many English sentences also contain subordinate clauses. Like a main clause, a subordinate clause
contains a subject and a verb. However, a subordinate clause cannot stand alone as a sentence.
Example: Although Karina has a new book = subordinate clause
S V
There are three types of subordinate clauses in English:
1. Noun clauses
2. Adjective clauses
3. Adverb clauses

1. A noun clause functions as a subject, object, or complement in a sentence.


I like what you said.
Noun clause direct object

What you said is unbelievable


Noun clause direct object

2. An adjective clause functions as an adjective in a sentence.


I like the book that you gave me.
Adjective clause describing book

The man who gave you the book is my old friend.


Adjective clause describing the man

Structure Section | 39
ELPT_ Level 2

3. An adverb clause functions as an adverb in a sentence.


Tom is singing while Paul is dancing.
Adverb clause

Because it is raining outside, I am taking the umbrella with me.


Adverb clause

Clause markers (CM) are used to connect clauses in English. Each English clause type has its own special
clause markers.
Model test

1. Stephen is still coming to school even although it is raining.


A B C D

What is wrong with this sentence? The clause marker has extra part.

Explanation: A prepositional phrase consists of a preposition and an object. By she swimming


contains a preposition, a subject, and an object. Answer (B) is the correct answer to the this question
because by she swimming is incorrect
Correction: by swimming

2. Last meeting we talked ________________ the discovery of gold in California in the 1800s.
(A) on
(B) on
(C) for
(D) about

What is needed in this sentence? A preposition

Explanation: In this question, a verb + preposition combination is being tested. The verb talk is often
followed by the preposition about. Answer (D) is the correct answer to the question.

14. PREPOSITIONAL PHRASES AND SUBORDINATE CLAUSES

English prepositional phrase and subordinate clauses are easily confused. Confusion often happens
because, although they are formed differently, prepositional phrases and subordinate clauses can
function in similar ways in English.

Because of her financial difficulties, Anne could not go.


Prepositional phrase

Because she had financial difficulties, Anne could not go


Subordinate phrase

Model test

1. _____________ we have nothing in common, We can work well and submit the assignment on
time.
(A) In spite of
(B) That
(C) Although
(D) Despite

What is needed in this sentence? An adverb clause marker

Explanation: In this question, there are two clauses, We have nothing in common and we can work
well and submit the assignment on time. A clause marker is needed at the beginning of the first
clause. Answers (A) and (D) do not contain a clause marker. Instead, they contain prepositions.
Answer (B) contains a clause marker. However, this clause marker would create a noun clause or an
adjective clause. The clause in this sentence is an adverb clause. Answer (C) is the correct answer
to this question because it contains a clause marker that can begin an adverb clause.

Structure Section | 40
ELPT_ Level 2

2. _____________ we have nothing in common, we can work well and submit the assignment on
time.
(A) In spite of
(B) That
(C) Although
(D) Despite

What is needed in this sentence? An adverb clause marker

Explanation: In this question, there are two clauses, We have nothing in common and we can work
well and submit the assignment on time. A clause marker is needed at the beginning of the first
clause. Answers (A) and (D) do not contain a clause marker. Instead, they contain prepositions.
Answer (B) contains a clause marker. However, this clause marker would create a noun clause or an
adjective clause. The clause in this sentence is an adverb clause. Answer (C) is the correct answer
to this question because it contains a clause marker that can begin an adverb clause.

15. APPOSITIVE AND THE NOUN STRUCTURES THEY RENAME

An appositive is a noun that comes before or after another noun and has the same meaning.
Sally, the best student in the class, got an A on the exam.

In this example Sally is the subject of the sentence and the best student can easily be recognized as
an appositive phrase because of the noun student and because of the comma. Sally and the best
student are the same person.

Model test

1. ______, George, is attending the lecture.

(A) Right now


(B) Happily
(C) Because of the time
(D) My friend

What is needed in this sentence? A subject

Explanation: This sentence still needs a subject, because George is an appositive and not a Subject.
Answer (A), (B), and (C) is not a Subject. Therefore, the best answer is (D).

16. ONE-WORD –ING AND –ED ADJECTIVES


Verb forms ending in –ed and –ing can be used as adjectives. For example, the verbal adjectives
cleaned and cleaning come from the verb to clean.

For example

The woman cleans the car. (cleans is the verb of the sentence)
VERB

The cleaning woman worked on the car.


ADJ
(cleaning is a verbal adjective describing the woman)

The woman put the cleaned car back in the garage.


ADJ
(Cleaned is a verbal adjective describing car)

Remember:
The –ing adjective means that the noun it describes is DOING the action.
The –ed adjective means that the noun it describes is RECEIVING the action from the verb.
Look at the table below

Structure Section | 41
ELPT_ Level 2

Type Meaning Use Example

–ing active It does the action of the verb …the happily playing children.
(The children play)
–ed passive It receives the action of the verb …the frequently played record.
(Someone plays the record)

17. –ING AND –ED MODIFYING PHRASE


The –ing form of the verb (a present participle) can cause confusion because it can be either a part of
the verb or an adjective. It is part of the verb when it is preceded by some form of the verb be.
e.g.: The man is talking to his friend.

A present participle is an adjective when it is not accompanied by some form of the verb be.
e.g.: The man talking to his friend has a beard.

Model test

1. The child ______ playing in the yard is my son.


(A) now
(B) is
(C) he
(D) was

Explanation: ‘Playing in the yard’ is not part of the verb. It is an –ing modifying phrase. In this sentence
there is a complete subject (child) and a complete verb (is), so this sentence does not need another
subject or verb. The best answer is (A).

The –ed form of the verb (a past participle) can cause confusion because it can be either a part of the
verb or an adjective. It is part of the verb when it appears with have or be.
e.g.: The family has purchased a television.
The poem was written by Paul.

A past participle is an adjective when it is not accompanied by some form of the verb be or have.
e.g.: The television purchased yesterday was expensive.

Model test

2. The packages ______ mailed at the post office will arrive Monday.
(A) have
(B) were
(C) them
(D) just

Explanation: ‘Mailed at the post office’ is not part of the verb. It is an –ed modifying phrase. In this
sentence there is a complete subject (The packages) and a complete verb (will arrive), so this
sentence does not need another subject or verb. The best answer is (D).

18. GERUNDS AND INFINITIVES

Gerunds and infinitives are verb forms that can take the place of a noun in a sentence. The following
guidelines will help you figure out whether a gerund or infinitive is needed.

Following a verb (gerund or infinitive)


Both gerunds and infinitives can replace a noun as the object of a verb. Whether you use a
gerund or an infinitive depends on the main verb in the sentence.

I expect to have the report done by Friday. [INFINITIVE]


I anticipate having the report done by Friday. [GERUND]

Structure Section | 42
ELPT_ Level 2

Following a preposition (gerund only)


Gerunds can follow a preposition; infinitives cannot.

Can you touch your toes without bending your knees?


He was fined for driving over the speed limit.
She got the money by selling the car.

Note: Take care not to confuse the preposition "to" with an infinitive form, or with an auxiliary form
such as have to, used to, going to

He went back to writing his paper. [PREPOSITION + GERUND]


I used to live in Mexico. [AUXILIARY + VERB]
I want to go home. [VERB + INFINITIVE]

- Following an indirect object (infinitive only)


Some verbs are followed by a pronoun or noun referring to a person, and then an infinitive.
Gerunds cannot be used in this position.

I must ask you to reconsider your statement.


They begged her to stay for another term.
His findings caused him to investigate further.

Model test

In automotive companies, employees are rewarded for ______ with the firm for lengthy time periods.
(A) to stay
(B) staying
(C) to staying
(D) stay

What is needed in this sentence? A gerund


Explanation: The clue in this sentence is the preposition for. Preposition is followed by a gerund.
Answers (A), (C), and (D) are not prepositions. Therefore, Answer (B) is the best answer.
19. IT AND THERE SENTENCES

THERE
There is used as a dummy subject with part of the verb be followed by a noun phrase

 To introduce a new topic:


There is a meeting this evening. It will start at seven.
There has been an accident. I hope no one is hurt.

 With numbers or quantities:


There was a lot of rain last night.
There must have been more than five hundred in the audience.

 To say where something is:


There used to be a playground at the end of the street.
There are fairies at the bottom of the garden.

 With an indefinite pronoun or expressions of quantity and the to-infinitive:


There is nothing to do in the village.
There was plenty to read in the apartment
There is a lot of work to do

 With an indefinite pronoun or expressions of quantity and an -ing verb:


There is someone waiting to see you.
There were a lot of people shouting and waving.

Structure Section | 43
ELPT_ Level 2

 A singular verb is used if the noun phrase is singular:


There is a meeting this evening. It will start at seven.
There was a lot of rain last night.

 A plural verb is used if the noun phrase is plural:


There are more than twenty people waiting to see you.
There were a lot of people shouting and waving.

IT
We use it to talk about:

 Times and dates:


It’s nearly one o’clock.
It’s my birthday.

 Weather:
It’s a lovely day.
It was getting cold.

 To give an opinion about a place:


It’s very cold in here.
It will be nice when we get home.

 To give an opinion followed by to-infinitive:


It’s nice to meet you.
It will be great to go on holiday.

 To give an opinion followed by an -ing verb:


It’s great living in Spain.
It can be hard work looking after young children.

20. WORD FORM AND FUNCTION

The form of an English word must agree with its function in a sentence. For many English words
several forms are possible. Different word forms are created by endings, called suffixes, to the original
form of word.

Below are some of the common suffixes used to create different word forms.

NOUN (THING) SUFFIXES


-ism socialism -ment government
-ence excellence -ty specialty
-ion education -age marriage
-ness kindness -ship friendship
NOUN (PERSON) SUFFIXES
-er teacher -ist specialist
-or actor -ian mathematician
ADJECTIVE SUFFIXES
-ent excellent -ive effective
-ant important -ous dangerous
-ful beautiful -al natural
-ic panic -able capable
-less careless -ible sensible
VERB SUFFIXES
-ate educate -en darken
-ify beautify -ize organize
ADVERB SUFFIXES
-ly beautifully -ward backward

Structure Section | 44
ELPT_ Level 2

Model test

1. She told the truthful when we asked her about her past.
A B C D

What is wrong with this question? An adjective form has been used where a noun form is needed.

Explanation: In this question, a noun is needed after the article the. Truthful is not a noun; it is an
adjective. Answer (B) is the correct answer to this question because truthful is incorrect.
Correction: truth

2. The happily man spoke cheerfully about his family and friends.
A B C D

What is wrong with this sentence? An adverb form has been used where an adjective form is
needed.

Explanation: In this question, and adjective form is needed to describe man. Happily is an adverb
formed from the adjective happy. Answer (A) is the correct answer to this question because happily is
incorrect.
Correction: happy

21. WORDS THAT DON’T EXIST IN ENGLISH

Occasionally, a word that does not exist in English is used on the ELPT. This form is always closely
related to a real English word form. Below are a few examples of words that don’t exist and their
corresponding real English word forms.

DOESN’T EXIST ENGLISH WORD


adaptator adaptor
estable stable
explorator explorer
plastical plastic
Model test

Christopher Columbus may not have been the first explorator to discover America.
A B C D
What is wrong with this sentence? A word has been used that does not exist in English.

Explanation: In this question, the word explorator is a word that does not exist in English. Answer (C)
is the correct answer to this question because exploratory is incorrect.
Correction: explorer

22. EQUATIVE, COMPARATIVE AND SUPERLATIVE DEGREE

To make the equative, comparative, and superlative degree of adjectives and adverbs, the following
forms are used:
One syllable Two syllable Two or more syllable Adj. or Adv.
Adj. or Adv. Adj. or Adv.
ending in -y
Equative as … as as … as as … as
as tall as as happy as as beautiful as
Comparative -er than … -er than more/less … than
taller than happier than more beautiful than
Superlative the … -est the … -est the most/least …
the tallest the happiest the most beautiful

NOTE: Some words have irregular degree forms. The most common of these are:

Structure Section | 45
ELPT_ Level 2

ADJECTIVE ADVERBS
good better best well better best
bad worse worst badly worse worst
much/many more most much more most
little less least little less least
far farther farthest far farther farthest
far further furthest far further furthest

 The equative degree is used to show equality


Annie is as tall as Jacob.
This sentence tells us that Annie and Jacob are the same height.

 The comparative degree is used to compare two things that are not equal.
Annie is taller than Jacob
This sentence tells us that Annie’s height are not the same. In addition, it tells us that Annie’s
height is greater than Jacob’s.

 The superlative degree is used to compare three or more things that are not equal.
Annie is the tallest student in the class.
This sentence tells us that there are more than two students in the class and that Annie’s
height is the greatest of all the students.

Model test

1. Although Tom’s paper was longer than Alice’s, Alice’s paper was more insightful that Tom’s.
A B C D
What is wrong in this sentence? The comparative degree has been incorrectly formed.

Explanation: In this question, there are two comparative structures. The first, longer than, is correctly
formed. However, the second, more insightful that, is incorrectly formed. That is not used in forming
the comparative degree. Answer (D) is the correct answer to this question because more insightful that
is incorrect.
Correction: more insightful than

2. Professor Clark’s chemistry class is __________ Professor Smith’s.


A. more than difficult
B. to the difficult
C. as difficult as
D. the most difficult

What is needed in this sentence? the equative degree

Explanation: In this question, two classes are being compared. Therefore, either the comparative or
the equative degree is needed. Answer (A) contains an incorrectly formed comparative degree.
Answer (B) contains a prepositional phrase. Answer (D) contains the superlative degree. Answer (C) is
the correct answer to this question because it contains a correctly formed equative degree.

23. STANDARD WORD ORDER

The standard word order of the principal parts of English sentence is:

SUBJECT + VERB (+OBJECT AND/OR + COMPLEMENT)

The subject comes before the verb. The verb comes before its object and/or before the complement of
the sentence.

Kathy is eating
S V

Structure Section | 46
ELPT_ Level 2

Kathy is happy
S V Comp
Kathy is eating chocolate
S V DO
Model test

1. Karen’s home __________ on a busy downtown street


(A) Small, is an apartment
(B) A small apartment is
(C) An apartment is small
(D) Is a small apartment

What is needed in this sentence? A verb and a complement

Explanation: In this question, the verb is missing. The verb BE is contained in all of the answer
choices. A complement, a small apartment is also contained in all of the sentences. However, the
word order is incorrect in all of the answer choices except for answer (D). Answer (D) is the correct
answer to this question.

24. WORD ORDER IN SUBORDINATE CLAUSES BEGINNING WITH


QUESTIONS WORDS

Question words such as who, what, when (ever), where (ever), why, how (ever), how long, how much,
and how many occur in many English sentences. These words are often used to begin subordinate
clauses that do not ask a direct question. The sentences that contain these clauses are not followed
by a question mark (?). In these clauses the subject and the verb are not inverted.

I don’t know who she is


Noun Clause
When she arrives, please show her in.
Adverb clause
Tell me the reason why she didn’t come
Adjective clause
Model test

1. I don’t know why ________________.


(A) did she leave me
(B) she leaves me
(C) she did leave me
(D) she left me

What is needed in this sentence? A subject and a verb

Explanation: In this question, the word why is used to connect the two clauses so a subject and verb
are needed after this connector; this is not a question, so the subject and verb should not be inverted.
The best answer is therefore answer (D).

25. INVERTED SUBJECT-VERB WORD ORDER


WITH SPECIAL EXPRESSIONS AND IN CONDITIONAL SENTENCE
The standard word order of subject + verb (+ object and/or + complement) is not followed in all English
Sentences. In certain situations, inverted subject-verb word order is used. That is, the subject of a
sentence is placed after the first helping verb or other BE. If there is no verb BE or if there is no
helping verb, the helping verb DO is added as the first verb of the sentence. This inverted subject-verb
word order is most common in direct questions, but it is also common in other situations.

1. After special expressions of location:


On the beach were five beach umbrellas
Expression of location V S
Nowhere did he see her
Expression of location first V S

Structure Section | 47
ELPT_ Level 2

2. After special negative (no, not, and never) and almost negative (hardly, rarely, scarcely, not
only, no time, barely, only, seldom, etc) expressions:
Never had I seen such a glorious sight
Negative first V S
Only after he saw her did he understand *
Almost negative first V S

*NOTE: When the negative or almost negative expression is a part of a subordinate clause,
the subject and verb of subordinate clause are not inverted. The subject and the verb of the
main clause are inverted.

3. After the special expressions so and either:


So happy was she that she danced around the room
V S
I liked the coffee, so did Mike
V S
I didn’t like the coffee, and neither did Mike
V S

4. In conditional sentences that do not begin with if:


With if: If he had seen you, he would have greeted you
S V
Without if: Had he seen you, he would have greeted you first
V S

Model test

1. Rarely____________ happy.
(A) John is ever
(B) Is john ever
(C) Ever John is
(D) John ever is

What is needed in this sentence? An inverted subject and verb

Explanation: In this question, rarely is the first word of the sentence. Inverted subject-verb word order
occurs after rarely. Answer (B) is the correct answer to this question. It is the only answer choice that
contains inverted subject-verb word order.

2. ____________, he would have understood the movie better.


(A) Carl had read the book
(B) The book had been read by Carl
(C) Had Carl read the book
(D) Read the book had Carl

What is needed in this sentence? A conditional that does not begin with if

Explanation: In this question, each answer choice contains parts of a conditional without if. In these
types of conditionals, inverted subject-verb word order is needed. Answer (A) does not contain
inverted word order. Answer (B) contains a passive construction. This is not the inverted structure that
is needed. Answer (D) contains inverted word order, but using the wrong part of the verb. Answer (C)
is the correct answer to the question because it contains correct inverted subject-verb word order.

26. PARALLEL STRUCTURE in Comparisons and in Series


Joined By And, But, or Or

When words, phrases, or clauses have parallel (similar) functions in English sentences, they should
have parallel structure as well. It is important to maintain parallel structure in English sentences.

Clarice is beautiful, smart, and talented.


Adj Adj Adj
He won the lottery, quit his job, and bought presents for all his friends.
Verb phrase Verb phrase Verb phrase

Structure Section | 48
ELPT_ Level 2

You should be especially careful about parallel structure when:

1. Parallel structure in words, phrases, and clauses used in series joined by and, but, or or:

Incorrect: He likes to swim, to hike, and riding his bike.


infinitive infinitive gerund
Correct: He likes to swim, to hike, and to ride his bike.
infinitive infinitive infinitive

Incorrect: She went home because she needed money and due to her mother’s illness.
Subordinate clause Prepositional phrase
Correct: She went home because she needed money and because her mother was ill.
Subordinate clause Subordinate clause

Incorrect: Her cheerful outward manner and she told interesting stories made her very popular.
Noun phrase Main clause
Correct: Her cheerful outward manner and her interesting stories made her very popular.
Noun phrase Noun phrase

2. Parallel structure with equative (as…as) and comparative (-er than/more…than) constructions
and with other expressions of comparison (e.g., similar to, the same as):

Incorrect: How to pronounce English is more difficult than reading it.


Infinitive phrase Gerund phrase
Correct: Pronouncing English is more difficult than reading it.
Gerund phrase Gerund phrase

In comparisons, you must be especially careful that the two compared things are parallel in both
meaning and structure.

Incorrect: The shelves in Durick library are taller than Norwich Library
In this sentence, shelves and Norwich Library are being compared. These two nouns are
not comparable (parallel in meaning) even though they are parallel in structure.

Correct: The shelves in Durick library are taller than those in Norwich Library
In this sentence, shelves are being compared to shelves, referred to by the pronoun
those. These two structures are parallel in meaning and in structure.
Model test

1. Lance’s efforts led to a promotion,___________, and an award of $10,000.


(A) His peers recognized him
(B) recognition by his peers
(C) he received recognitions from his peers
(D) to receive recognition from his peers

What is needed in this sentence? A noun phrase parallel to a promotion and an award

Explanation: In this sentence, noun phrase are being joined in a series with and. Answers (A) and (C)
contain more than a noun phrase. They contain complete sentences. Answer (D) contains an infinitive
phrase. Answer (B) is the correct answer to this question. It contains a noun phrase that is paralleled
in structure to a promotion and an award.

2. Sleeping well is as important to good health as to eat well


A B C D

What is wrong with this sentence? A comparison is being made between two structures which are
not parallel

Explanation: In this question, the gerund sleeping is compared to the infinitive to eat. These two
structures are not parallel. Answer (D) is the correct answer to this question because to eat is
incorrect.
Correction: eating

Structure Section | 49
ELPT_ Level 2

27. PAIRED EXPRESSION

There are pairs of expressions in English which can be especially confusing. These paired
expressions are:

B both….and
either….or
neither….nor
not only….but also

Paired expressions can act as clause markers, or they can function to join words or phrases.

As clause markers:
Either he will go to the movie, or he will go to the play
CM Clause CM Clause

Joining phrases:
He will go either to the movie or to the play
C Prep phrase CM Prep phrase

Joining words:
He will go either today or tomorrow
CM Word CM Word

Paired expressions need to be checked for a variety of things. Parallelism is important for paired
expressions. The same grammatical from should follow each word of a paired expressions.

Incorrect: Laura both enjoys books and music


Verb Noun

Correct: Laura enjoys both books and music


Noun Noun

Model Test

1. We must ___________ but also our clothes.


(A) Not only pack our books
(B) Pack not only our books
(C) Our books not only pack
(D) Not our only books pack

What is needed in this sentence? The first half of a paired expression + a noun phrase

Explanation: In this question, the second half of a paired expression, but also, is present. But also is
followed by the noun phrase our clothes. To keep parallel structure in this sentence, not only followed
by a noun phrase is needed. Answers (A) and (C) contain not only followed by a verb. Answer (D)
separates not from only, which creates an incorrect form. Answer (B) is the correct answer to the
question because it contains not only followed by a noun phrase.

2. Both Leo also Margaret were waiting for the bus when the storm began
A B C D

What is wrong with this sentence? A paired expression has been incorrectly formed

Explanation: In this question, the first part of a paired expression, both, is used. However, the second
part of this expression is missing and has been replaced by also. Answer (A) is the correct answer to
the question because also is incorrect.
Correction: and

Structure Section | 50
ELPT_ Level 2

28. CONFUSING WORDS AND EXPRESSION

There are words and expressions in English which are especially confusing, because they sound very
much alike and/or because they have very similar functions in English sentences.

Incorrect: He was formally a teacher, but now he is a car dealer

Correct: He was formerly a teacher, but now he is a car dealer

Formally and formerly in this pair of sentences sound very much alike and have similar function.
However, their meanings are quite different. Formally means officially and does not have the meaning
necessary for this sentence. Formerly, which means before, has the correct meaning for this sentence.

Incorrect: Melissa often does a cake for dessert

Correct: Melissa often makes a cake for dessert

Does and make in this pair of sentences do not sound alike. However, they both function as verb, and
they have some similarities in meaning. The verb DO, however, often expresses the idea of
performing of completing. Make often expresses the idea of creating or constructing. The difference in
meaning between make and do is great enough that they are not interchangeable. Since Mellissa had
created rather than performed the cake, make is the correct verb for this sentence.

Model Test

1. The tests in this class are less difficult than tests in another classes
A B C D

What is wrong with this sentence? A word that sounds like the correct word has been substituted
for the correct word

Explanation: In this question, another has been used to describe classes. However, another is used
only with singular, countable, indefinite nouns. Answer (D) is the correct answer to this question
because another is incorrect.
Correction: other

Structure Section | 51
ELPT_ Level 2

Review Exercise
Exercise A
1. Subjects, Objects, and Noun Complements
2. Subjects and Verbs
3. Expressions of Quantity
4. Articles
5. Personal, Possessive, and Reflexive Pronouns
6. Adjectives and Nouns

1. Although he has written several plays and poems, John Ashbery is best known for him work
A B C
in the area of art criticism.
D
2. In the late spring, the honeysuckle, with its delicate yellow blossoms, fills the air with a aroma
A B C D
resembling that of honey.

3. Kites vary in size, shape, and weight, according to the fancy of its creators.
A B C D

4. The most visible changes physical of aging take place in the skin.
A B C D

5. The Richter scale provides a fairly accurate measurements of seismic disturbances.


A B C D
6. Analysis of the small number of known native language families in eastern North America
A
demonstrates that they are no more than five thousands years old.
B C D
7. There is fewer oxygen available in the atmosphere of Jupiter than there is on earth.
A B C D

8. ________ can result from damage to DNA 10. ________ is susceptible to subtle political
molecules. changes is well known.
(A) Cells mutate (A) That the stock market
(B) Cell mutation (B) What the stock market
(C) While cell mutate (C) Because the stock market
(D) During cell mutation (D) The stock market

9. New York ironweed, with its small, dark 11. Phillis Wheatly, whose first book of poems
purple thistle flowers, sometimes ________ was published when she was only
to a height of eight feet nineteen years old, ________ the object
(A) Grows of public attention because she was black
(B) Growing slave.
(C) It grows (A) Who was
(D) Up (B) Being
(C) As
(D) Was

Structure Section | 52
ELPT_ Level 2

Exercise B

7. Verb Tense Form


8. Verb Tense Meaning
9. Passive and Active Sentences
10. Modals and Modal-Like Verbs
11. Subject-Verb Agreement

1. Nutritionists currently believe that vitamin A and beta-carotene aids in preventing some kinds of
A B C D
cancer.

2. The Swedish settlers who builded the first log cabins in the state of Delaware brought their
A B C
logs with them from their homeland.
D

3. During the early 1970’s the American public collectively has become health conscious,
A B
turning away from the highly processed foods that had been so popular in the past.
C D

4. In 1968, John Steinback was gave the Nobel Prize for literature for his acclaimed novel, The
A B C D
Grapes of Wrath

5. It has always been thought that cell mutation is a random event; however, geneticist find
A B C D
evidence to the contrary over the past few years.

6. Important news are now conveyed electronically from one side of the globe to the other
A B C
in a matter of seconds.
D

7. Democrats had dominate the White House for five terms when Republican Dwight D. Eisenhower
A B C
was elected in 1952
D

8. Unlike the young of most animal species, 10. Glaciers covering the west coast of
human children ________ to depend on Greenland ________ about 7,500
adult care for many years icebergs a year into the North Atlantic.
(A) must (A) are dropped
(B) needing (B) drops
(C) have (C) dropping
(D) has (D) drop

9. One variety of wild rose, the sweetbrier, 11. Newly installed gypsum board walls
________ to the United States by the ________ before they are painted.
pilgrims. (A) with a sealant coated
(A) bringing (B) should coat with a sealant
(B) was brought (C) should be coated with a sealant
(C) brought (D) coating with a sealant
(D) that was brought

Structure Section | 53
ELPT_ Level 2

Exercise C

12. Preposition and Prepositional Phrase


13. Main and Subordinate Clause Markers
14. Prepositional Phrase and Subordinate Clause
15. Appositive and the Noun Structure They rename

1. Since 1905, ________ of Albert Einstein’s 4. The pony express, _______ between Saint
first important scientific publication, the Joseph, Missouri, and Sacramento,
real world has become the world of California, was put out of business with the
mathematician. invention of electric telegraph.
(A) the year (A) was a ten-day mail service
(B) his year (B) its ten-day mail service
(C) years (C) a ten-day mail service
(D) a year (D) of a ten-day mail service

2. ________ in his autobiography, Benjamin 5. Scientists still know little about variations
Franklin was born in Boston and was _______ the moon’s gravitational field or
apprenticed to his brother James to learn about its surface composition.
the printer’s trade. (A) into
(A) That he states (B) in
(B) As he states (C) at
(C) He states (D) during
(D) States
6. The four U.S presidents ________ in log
3. ________ colonial statesman William cabins were Lincoln, Fillmore, Buchanan,
Bradford attended no formal school, he and Garfield.
was a well – educated man. (A) were born
(A) Despite (B) they were born
(B) In spite of (C) who were born
(C) Although (D) who they were
(D) Even
7. In linguistics, ________ morphology refers
to the study of the formation of words and
smaller units of meaning.
(A) is termed
(B) to term
(C) be termed
(D) the term

8. Even when the warmest part of the day in the hottest season of the year, desert sand dunes
A B
teem with Bembex, more commonly known as sand wasp.
C D
9. The 1960 presidential campaign was marked by an innovation into American politics – a series of
A
television debates in which the two candidates responded to questions put by newspaper reporters.
B C D
10. In 1914, Congress established the Federal Trade Commission, who steadily increased the
A B
extent and nature of its regulation of the advertising industry.
C D
11. Even although the negative and positive charges of isotopes are identical, their masses are not.
A B C D

Structure Section | 54
ELPT_ Level 2

Exercise D

16. One-Word –ing and –ed Adjectives


17. Ing and –ed Modifiying Phrases
18. Gerunds and Infinitives
19. IT and THERE Sentences

1. ________ the Depression years, American 4. ________ several weeks for a person to
painters seemed to turn away from Europe starve to death, but without sleep the
as a source of inspiration. human body dies in about 10 days.
(A) It was (A) Usually taking it
(B) While (B) Usually takes it
(C) There were (C) It usually takes
(D) During (D) To take it usually

2. ________, Dwight Eisenhower resigned his 5. ________ its rigid home on its back, the
post as commander of NATO in 1951. land tortoise is well protected from
(A) He was seeking the Republican predators.
presidential nomination (A) Carried
(B) The Republican presidential (B) It carries
nomination seeking (C) Carrying
(C) Seeking the Republican presidential (D) To carry
nomination
(D) The Republican presidential 6. Descendant of a long line of New
nomination was sought Englanders, Robert Lowell chose, in his
early poems, ________ against his
3. ________ Native American myths and background of Bostonian eminence and
poems, American anthropologists have public service
preserved much that is beautiful in a (A) for reacting
threatened culture. (B) to react
(A) By recording (C) reacted
(B) Recorded (D) to the reaction
(C) Record
(D) To be recorded 7. Today Edward Taylor is generally regarded
as the finest poet _____________ in
America before the nineteenth century.
(A) he wrote
(B) written
(C) wrote
(D) writing

8. In the spring of 1923, some 15,000 unemploying American war veterans converged Washington
A B
and established the shanty community of Anacostia Flats.
C D
9. Special tools have been designed opening coconuts without throwing them against hard surface.
A B C D
10. It is wise avoiding overconsumption of alcohol because of its detrimental effects on the heart,
A B C D
liver, and central nervous system.

11. Ms.magazine, which began in 1972, has long been considered one of the led publications of
A B C
the feminist movement.
D

Structure Section | 55
ELPT_ Level 2

Exercise E

20. Word Form and Function


21. Words That Don’t Exist In English
22. Equative, Comperative and Superlative Degree

1. Thomas Bangs Thorpe, American author and artist of the nineteenth century, produced short
A B
stories and sketches which are appreciated for their humorous.
C D
2. An amazing variety of animals, by means of elaborate adapt ors , are able to thrive in conditions
A B C D
of extreme heat and dryness.

3. At the time of their first contact with Europeans, the Native Americans of the Great Plains were
A
considerable more spread out than they are now.
B C D
4. The processes involved in the creation of the universe remain mysteriously to astronomers.
A B C D
5. Most Americans’ blood cholesterol levels rise as they grow more older.
A B C D
6. Musical celebrities from all over the world appear regular in the exclusive nightclubs and
A B C D
casinos of Las Vegas.

7. Many successful American film directions are former actors with desire to expand their experience
A B C D
in film industry.

8. Homing pigeons are especial good at finding their way home over hundreds of miles of
A B C
unfamiliar terrain.
D
9. Humans have a very large and densely brain in proportion to their body size.
A B C D
10. The worse winter of all for the settlers at Jamestown was that of 1607, when several in their
A B C D
party died

11. In the nineteenth century, new nails, screws, and cutting tools revolutionized the constructed
A B C D
of houses.

Structure Section | 56
ELPT_ Level 2

Exercise F

23. Standard Word Order


24. Word Order in Subordinate Clauses Beginning with Question Words
25. Inverted Subject-Verb Word Order with Special Expressions and In Conditional Sentences
26. Parallel Structure in Comparisons and in Series Joined By And, But, or Or
27. Paired Expression
28. Confusing Words and Expression

1. John Updike has published not only many 5. Astronomers rely on measurements of
novels and stories ________ four books mass and brightness to determine
of poems, a play, and numerous book ______________.
reviews and other prose writing. (A) how old is a star
(A) but he has also written (B) is a star how old
(B) also (C) a star is how old
(C) but (D) how old a star is
(D) in addition to writing
6. At one end of an amino acid
2. Fat ___________for the energy from food _______________________.
eaten in excess of need. (A) an amine group is
(A) is the body’s chief storage form (B) is an amine group
(B) the body’s chief storage form it is (C) an amine group is there
(C) is it the body’s chief storage form (D) is where an amine group
(D) the body’s is chief storage form
7. Fats __________ and help insulate and
3. Not until several years after a war has protect the body.
ended ________ to feel the severe (A) an efficient storage material is
psychological damage it can cause. formed
(A) do many of its veterans begin (B) their efficient storage material is
(B) many of its veteran begin formed
(C) and many of its veterans begin (C) form an efficient storage material
(D) many of its veterans beginning (D) efficient storage material is formed

4. The deuterium in sea water _________


with a billion times the energy now left in
all our coal and oil reserves.
(A) could it provide us
(B) us could provide
(C) it could provide us
(D) could provide us

8. The long-term affects of the American’ Civil War, which split families and friends apart, are still
A B C
being felt in the twentieth century.
D
9. Topology is a branch of mathematics that deals with the ways in which surface can be twisted,
A B
bend, pulled, or otherwise transformed from one shape to another.
C D
10. Scientist still know little either about variations in the moon’s gravitational field also about its
A B C D
surface composition.

11. The ladybug beetle is such helpful in controlling other, more harmful insects, that it is often sold to
A B C
gardeners as a natural method for pest control.
D

Structure Section | 57
ELPT_ Level 2

Practice Test 1
1. Kansas_____ at the geographical center of the United States.
(A) it is located
(B) the location is
(C) its location
(D) is located

2. ______first pizza restaurant opened in New York City in 1895.


(A) The
(B) It was the
(C) At the
(D) It was at the

3. _________ without a backbone is the Atlantic squid.


(A) For the largest creature
(B) It is the largest creature
(C) The largest creature
(D) The largest creature is

4. The last star on the handle of________ Polaris, or the orth Star.
(A) to call the Little Dipper
(B) calling the Little Dipper
(C) the Little Dipper calling
(D) the Little Dipper is called

5. Societies________ to at least 1500 B.C made use of molds to treat superficial infections.
(A) they date back
(B) the date is back
(C) dating back
(D) date back

6. Eugene Debs ran for the presidency of the United States five times, _________was never elected.
(A) he
(B) but he
(C) for him
(D) for his

7. The oldest known daggers are ones that Neolithic humans___________out of flint
(A) skillfully chipped
(B) skillful chips
(C) chipping skillfully
(D) chips are skillful

8. Woodrow Wilson served as president of Princeton_______was elected precident of the United


States.
(A) before
(B) to him
(C) he
(D) before he

Structure Section | 58
ELPT_ Level 2

9. Sound waves are produced when_____________ quickly.


(A) objects vibrate
(B) vibrating of objects
(C) objects to vibrate
(D) the vibration of objects

10. A genetic predisposition determines _____________ is most likely to form freckles.


(A) he
(B) it
(C) who
(D) when

11. The vast west-central section of North American, ______ from Texas to Canada, is one of the
world’s largest grasslands.
(A) it stretches
(B) strecthes
(C) which stretches
(D) is stretching

12. Characteristically, the dulcimer features three or four strings_____along a fingerboard.


(A) stretched
(B) are stretched
(C) stretch them
(D) they are stretched

13. Diamond Head is what ____________ of an extinct volcano.


(A) to remains
(B) remains
(C) remaining
(D) the remnants

14. Any member of a group of drugs_______________barbituric acid is a barbiturate.


(A) the derivation of
(B) is derived from
(C) derived from
(D) derives from

15. Henry VIII ruled that people born on February 29______their birthdays on February 28 in non-leap
years.
(A) a celebrity
(B) a celebration
(C) celebrating
(D) would celebrate

16. Areas of rock may shifts up or down on each side of a fault


A B C D
17. When someone take a depressant, the level of activity in the central nervous system is lowered.
A B C D
18. The world’s largest deposits of asbestos have be located in the Appalachian Mountains.
A B C D
19. Both Thomas Jefferson or John Adams died on the very same day, July 4, 1826.
A B C D
20. Most tundra plant are mosses and lichens that hug ground.
A B C D
21. Uranium atoms in a nuclear reaction is split into smaller atoms to produce heat.
A B C D

Structure Section | 59
ELPT_ Level 2

22. Virginia Dare, the first child born to English parents in the new World, was named after the Virginia
A B C
colony where her was born.
D
23. The apple tree appears in the mythology, traditions, historical, and archeology of the most ancient
A B C D
nations.
24. The first horse-drawn chariots were introduce around 2500 B.C.
A B C D
25. In 1917, Clyde Cessna designed an innovative monoplane, and later the Cessna Aircraft company
A B C
was started by he.
D
26. Not only the Estruscan alphabet and the Latin alphabet are descendants of the Greek alphabet.
A B C D
27. All of the oceans of the world is home to members of the dolphin family.
A B C D
28. The Earth rotates on its axis and follows an elliptically orbit around the Sun.
A B C D
29. Theodore Roosevelt was the first precident to ride in a car, fly in an airplane, and submerging in a
A B C
submarine.
D
30. The carbonation of water and soft drinks were one of the first uses found for gaseous carbon
A B C D
dioxide.
31. After an Apple is cut, chemicals inside her combine with oxygen to form a brown coat.
A B C D
32. Cribbage is a card game in which points are tally by moving pegs on a board.
A B C D
33. Most of the birds living in desert regions inhabits the fringers of the desert.
A B C D
34. The U.S Constitution, in Article 1, Section 2, provided for a census of the population every
A B C
decades.
D
35. The constellation Aquarius was associated with the rainy seasons by a large amount of ancient
A B C D
civilizations.

36. Only a small percentage of mined diamonds are actually suitably for use as gemstones.
A B C D
37. A turkey that Abraham Lincoln had save from Thanksgiving dinner became a beloved family pet.
A B C D
38. The game of dominoes is played with flat, oblong blocks identified by the number of dots on theirs
A B C D
faces.
39. Sodium hydroxide and other alkali solutions work effective as degreasing agents.
A B C D
40. The flintlock pistol was popular as aweapon in the seventeenth century because they could be fired
A B C
one-handed by a rider on horseback.
D

Structure Section | 60
ELPT_ Level 2

Practice Test 2
1. __________ range in color from pale yellow to bright orange.
(A) Canaries
(B) Canaries which
(C) That canaries
(D) Canaries that are

2. Carnivorous plants _________ insects to obtain nitrogen.


(A) are generally trapped
(B) trap generally
(C) are trapped generally
(D) generally trap

3. A federal type of government results in ______________.


(A) a vertical distribution of power
(B) power is distributed vertically
(C) vertically distributed
(D) the distribution of power is vertical

4. February normally has twenty-eight days, but every fourth year, ____________ has twenty-nine.
(A) there
(B) its
(C) is a leap year
(D) a leap year, it

5. Evidence suggests that one-quarter of operations ________ bypass surgery may be unnecessary.
(A) they involve
(B) involve
(C) involving
(D) which they involve

6. ______________ a tornado spins in a counterclockwise direction in the northern hemisphere, it


spins in the opposite direction in the southern hemisphere.
(A) However
(B) Because of
(C) Although
(D) That

7. The Caldecott Medal, __________ for the best children’s picture book, is awarded each January.
(A) is a prize which
(B) which prize
(C) which is a prize
(D) is a prize

8. Sports medicine is a medical specialty that deals with the identification and treatment of injuries to
persons ______________.
(A) sport are involved
(B) involved in sports
(C) they are involved in sports
(D) sports involve them

Structure Section | 61
ELPT_ Level 2

9. The Wilmington Oil Field, in Long Beach, California, is one of ________ oil fields in the
continental United States.
(A) productive
(B) the most productive
(C) most are productive
(D) productivity

10. Thunder occurs as ________ through air, causing the heated air to expand and collide with layers
of cooler air.
(A) an electrical charge
(B) passes an electrical charge
(C) the passing of an electrical charge
(D) an electrical charge passes

11. The population of Houston was ravaged by yellow fever in 1839 ______________ in 1867.
(A) it happened again
(B) and again
(C) was ravaged again
(D) again once more

12. Researchers have long debated ______ Saturn’s moon Titan contains hydrocarbon oceans and
lakes.
(A) over it
(B) whether the
(C) whether over
(D) whether

13. According to Bernoulli’s principle, the higher the speed of a fluid gas, __________ the pressure.
(A) it will be lower
(B) lower than the
(C) the lower
(D) lower it is

14. The flight instructor, _________ at the air base, said that orders not to fight had been issued.
(A) when interviewed
(B) when he interviewed
(C) when to interview
(D) when interviewing

15. In the northern and central parts of the state of Idaho ___________ and churning rivers.
(A) majestic mountains are found
(B) are majestic mountains found
(C) are found majestic mountains
(D) finding majestic mountain

Structure Section | 62
ELPT_ Level 2

16. Light can travels from the Sun to the Earth in eight minutes and twenty seconds.
A B C D
17. Every human typically have twenty-three pairs of chromosomes in most cells.
A B C D
18. In the sport of fencing, three type of swords are used: the foil, the epee and the sabre.
A B C D
19. The Internal Revenue Service uses computers to check tax return computations, to determine
A B
the reasonableness of deductions, and for verifying the accuracy of reported income.
C D
20. There was four groups of twenty rats each involved in the test.
A B C D
21. The type of jazz known as “swing” was introduced by Duke Ellington when he wrote and records
A B C D
“It Don’t Mean a Thing If It Ain’t Got That Swing.”

22. The bones of mammals, not alike those of other vertebrates, show a high degree of
A B C D
differentiation.
23. The United States receives a large amount of revenue from taxation of a tobacco products.
A B C D
24. Much fats are composed of one molecule of glycerin combined with three molecules of
A B C
fatty acids.
D
25. The capital of the Confederacy was originally in Mobile, but they were moved to Richmond.
A B C D
26. A pearl develops when a tiny grain of sand or some another irritant accidentally enters into the
A B C D
shell of a pearl oyster.
27. The English horn is an alto oboe with a pitch one fifth lower as that of the soprano oboe.
A B C D
28. In the Milky Way Galaxy, the most recent observed supernova appeared in 1604.
A B C D
29. Although the name suggests otherwise, the ship known as Old Ironsides was built of oak and
A B C
cedar rather than it was built of iron.
D
30. Never in the history of humanity there have been more people living on this relatively small
A B C D
planet.
31. Because of the mobile of Americans today, it is difficult for them to put down real roots.
A B C D
32. For five years after the Civil War, Robert E. Lee served to president of Washington College,
A B
which later was called Washington and Lee.
C D
33. Doctors successfully used hypnosis during World War II to treat fatigue battle.
A B C D
34. The lobster, like many crustaceans, can cast off a damaging appendage and regenerate a new
A B C
appendage to nearly normal size.
D

Structure Section | 63
ELPT_ Level 2

35. The main cause of the oceans’ tides is the gravitation pull of the Moon.
A B C D
36. The curricula of American public school are set in individual state; they do not determine by
A B C D
the federal government.
37. The fact that the sophisticated technology has become part of revolution in travel
A B
delivery systems has not made travel schedules less hectic.
C D
38. Balanchine’s plotless ballets, such Jewels and The Four Temperaments, present dance purely
A B C D
as a celebration of the movement of the human body.

39. In a solar battery, a photosensitive semiconducting substance such as silicon crystal is the source
A B C
of electrician.
D
40. In early days hydrochloric acid was done by heating a mixture of sodium chloride with iron sulfate.
A B C D

Structure Section | 64
ELPT_ Level 2

Practice Test 3
1. When ________________ in arctic regions, the Aleuts construct igloos as temporary winter
shelters.
(A) travel
(B) to travel
(C) traveling them
(D) traveling

2. Most substances contract when they freeze so that the density of a substance's solid is
_______________ of its liquid.
(A) than the higher density
(B) higher than the density
(C) the density is higher than that
(D) the higher the density

3. The mechanism by which brain cells store memories is ___________ clearly understood.
(A) none
(B) no
(C) not
(D) nor

4. Desert animals ______ a means of retaining moisture in such a hot, dry climate if they are to
survive.
(A) need
(B) needing
(C) to need
(D) was needed

5. _____________________ state of Wyoming is also known as the "Equality State" because


Wyoming women were the first in the nation to vote.
(A) The
(B) There is a
(C) That the
(D) As the

6. Fructose is a monosaccharide sugar that is much sweeter ___________________________.


(A) than cane sugar does
(B) does cane sugar
(C) cane sugar
(D) than cane sugar

7. Ground plans and contour maps of the Earth ____________________ from aerial photographs.
(A) can be drawn
(B) can draw
(C) to draw
(D) drawn

8. By the middle of the twentieth century, painters and sculptors in the United States had begun to
exert _____________________ over art.
(A) influence worldwide a great
(B) a great worldwide influence
(C) influence a great worldwide
(D) a worldwide influence great

9. ______________________ millions of galaxies exist in the vast space outside the Milky Way.
(A) It is estimated that
(B) An estimate that
(C) That is estimated
(D) That the estimated

Structure Section | 65
ELPT_ Level 2

10. The extent of the harmful effect of locoweeds on animals depends on the soil ____________ the
plants grow.
(A) which
(B) which in
(C) in which
(D) in

11. The operetta first ____________as a popular form of musical theater in the nineteenth century.
(A) to emerge
(B) emerging
(C) has emerged
(D) emerged

12. _________________ complex organic catalysts originating in living cells.


(A) Enzymes
(B) Enzymes are
(C) Enzymes which are
(D) Enzymes while they

13. In the eastern part of New Jersey _____________, a major shipping and manufacturing center.
(A) lies the city of Elizabeth
(B) the city of Elizabeth lies there
(C) around the city of Elizabeth lies
(D) there lies the city of Elizabeth around

14. Work in parapsychology, _____________ , has attracted a relatively small number of scientists.
(A) is a very controversial field
(B) which a very controversial field is
(C) a very controversial field
(D) a field very controversial which

15. ______________, the constitution of the Cherokee Nation is provided for a chief executive, a
senate, and a house of representatives.
(A) In 1827 they drafted
(B) The draft in 1827
(C) In 1827 was drafted
(D) Drafted in 1827

16. Sociological studies have found that deeply hold values and principles are highly resistant to
A B C D
change.

17. For centuries large communities of people have living on houseboats in parts of the world
A B
where the climate is warm and the waters are calm.
C D
18. Benjamin Franklin made the first bifocal spectacles for self by sawing the lenses of his
A B C
eyeglasses in half.
D
19. Not only do artificial reefs provide fish with food and shelter, they also serve as importantly
A B C D
underwater landmarks.

20. The United States Department of Agriculture supervises the quality, clean, and purity of meat.
A B C D
21. All birds, like most reptiles and a little primitive mammals, develop from embryos in eggs outside
A B C D
the mother's body.

Structure Section | 66
ELPT_ Level 2

22. The expansion of adult training programs has resulted partially from the feminist movement,
A B
which encouraging women to improve their skills for the job market.
C D
23. The most significant cosmological characteristic of the galaxies are the red shift in their
A B C
optical spectra.
D
24. James Whistler was indifferent to the titles of his painted and even changed the names of
A B C
some works years after their completion.
D
25. Duke Ellington's orchestra, playing his original compositions and arrangements, achieving a
A B
fine unity of style and made numerous innovations in modern jazz.
C D
26. Moles are almost completely blind, although its tiny eyes can distinguish light from dark.
A B C D
27. Noise is a psychological term referring toward unpleasant, unwanted, or intolerable sound.
A B C D
28. Elizabeth Blackwell, the first woman medical doctor in the United States, founded the New
A
York Infirmary, an institution that have always had a completely female medical staff.
B C D
29.Criminal contempt, committed in the presence of the court, may consist of disorderly behavior,
A B C
disrespectful, or disobedience of a judge's orders.
D
30.The Cubist movement in art was a reaction against traditional methods of portray reality.
A B C D
31. During the 1600's skilled shoemakers scarce were in what is now the United States.
A B C D
32. If a atom loses any of its electrons, it becomes positively charged and can combine chemically
A B C
with other atoms.
D
33.The National Education Association conduct extensive research on a great number of aspects
A B C
of education.
D
34.The pain-killing agent most commonly administered in dentistry is the local anesthetic, who
A B C
produces loss of feeling only in a specific area.
D
35.Certain types of computers work properly only in environments with controlled precisely
A B C D
temperatures.

36.The gorilla, while not as curious than the chimpanzee, shows more persistence and memory
A B C
retention in solving a problem.
D

Structure Section | 67
ELPT_ Level 2

37.The belief in fairies have existed from earliest times, and the literature of many countries includes
A B C
tales of fairies and their relationship to humans.
D
38. Acrylic paint enables artists to experiment with many colors effects.
A B C D
39. Salt Lake City, Utah's capital and largest city, is industrial and banking center.
A B C D
40. A rat's sharp teeth can gnaw through wood, plaster, or soft metallic such as lead.
A B C D

Structure Section | 68
ELPT_ Level 2

Practice Test 4
1. The flexibility of film allows the artist __________ unbridled imagination to the animation of cartoon
characters.
(A) to bring
(B) bringing
(C) is brought
(D) brings
2. Traditionally, _____________in New England on Thanksgiving Day.
(A) when served is sweet cider
(B) when sweet cider is served
(C) is served sweet cider
(D) sweet cider is served

3. Typical of the grassland dwellers of the continent _____________, or pronghorn.


(A) it is the American antelope
(B) the American antelope is
(C) is the American antelope
(D) the American antelope

4. Lillian D. Wald, public health nurse and __________, was born in Cincinnati Ohio, in 1867.
(A) reforming society
(B) social reformer
(C) who reformed society
(D) her social reform

5. Copper sulfate, spread in judicious amounts, kills algae __________ harming fish or aquatic
invertebrates.
(A) does not
(B) but does no
(C) except
(D)without

6. Of the millions who saw Haley's comet in 1986, how many people __________long enough to see
it return in the twenty-first century.
(A) will they live
(B) they will be living
(C) will live
(D) living

7. _________that fear, happiness, sadness, and surprise are universally reflected in facial
expressions.
(A) Anthropologists have discovered
(B) Anthropologists discovering
(C) The discovery by anthropologists
(D) Discovered by anthropologists

8. In 1964__________of Henry Ossawa Tanner's paintings was shown at the Smithsonian Institution.
(A) was a major collection
(B) that a major collection
(C) a collection was major
(D) a major collection

9. __________irritating effect on humans, the use of phenol as a general antiseptic has been largely
discontinued.
(A) Its
(B) Where its
(C) Since its
(D) Because of its

Structure Section | 69
ELPT_ Level 2

10. In order to remain in existence, __________must, in the long run, produce something consumers
consider useful or desirable.
(A) a profit-making organization
(B) a profit-making organization which
(C) therefore a profit-making organization
(D) whichever a profit-making organization

11. The greater the population, _________for water, transportation, and disposal of refuse.
(A) the greater the need there is
(B) greater need
(C) is there great need
(D) the great need
12. A historical novel may do more than mirror history; __________future events.
(A) even influencing
(B) it may even influence
(C) may even influence
(D) that it may even influence
13. __________a child, sculptor Anne Whitney showed an eager intellect and artistic talent that her
parents recognized and encouraged.
(A) Has been
(B) It was while
(C) She was
(D) As
14. It is widely believed that the pull of gravity on a falling raindrop changes _________round shape
into a teardrop shape.
(A) of the drop
(B) the drop's
(C) drop of
(D) drops their
15. __________modern offices becoming more mechanized, designers are attempting to personalize
them with warmer, less severe interiors.
(A) If
(B) But
(C) With
(D) Once

16. Not woman held a presidential cabinet position in the United States until 1933, when Frances
A B C
Perkins became secretary of labor.
D
17. The human body relies on certainty nutrients for its survival.
A B C D
18. Too much electric current may flow into a circuit as a result either of a fault in the circuit and
A B C D
of an outside event such as lightning.

19. The Appalachian Trail, extending approximately 2,020 miles from Maine to Georgia, is the
A B
longer continuous marked footpath in the world.
C D
20. For years, elephants were hunted for food and ivory, and as a result theirs numbers have been
A B C
greatly reduced.
D
21. Barges which carrier most of the heavy freight on rivers and canals are usually propelled by
A B C
towing.
D

Structure Section | 70
ELPT_ Level 2

22. Although afflicted by serious eyesight problems, Alicia Alonso was one the principal stars of
A B C
the American Ballet Theater and later formed her own dance company.
D
23. The ritual combat of animals are triggered by precise signals.
A B C D
24. It is more difficult to write simply, directly, and effective than to employ flowery but vague
A B C D
expressions that only obscure one's meaning.
25. Different species of octopuses may measure anywhere from two inches to over thirty feet in long.
A B C D
26. According to some theories derived from psychoanalysis, life is supposedly easier and more
A B C
pleasant when inhibitions overcoming.
D
27. When rainbows appear, they are always in the part of the sky opposite directly the Sun.
A B C D
28. Benjamin Franklin drew a political cartoon that is credited raising 10,000 volunteers for the
A B C D
American Revolutionary War.

29. The begins of the modern chemistry laboratory go back to the workrooms of medieval chemists.
A B C D
30. In many pieces of music there is a dominant theme on which the restful of the composition is
A B C
centered.
D
31. Luminescence refers to the emission of light by means another than heat.
A B C D
32.In a representative democracy, the people election delegates to an assembly.
A B C D
33.George Washington Carver found hundred of uses for the peanut, the sweet potato, and the
A
soybean and thus stimulated the cultivation of these crops.
B C D
34. A citadel, a fortress designed for the defense of a city, usually standed on top of a hill.
A B C D
35. Conservative philosophers argue that the very structure of society is threatening by civil
A B
disobedience, while humanists stress the primacy of the individual conscience.
C D
36. Since 1971 the regional corporations set up in Alaska by Congress managing everything from
A B C
fishing to banking.
D
37. A rocket burns propellant rapidly and most rockets carry a supply that last just a few seconds.
A B C D
38. Textile art is known for both its tactile and vision qualities.
A B C D
39. The metal aluminum has been first isolated early in the nineteenth century.
A B C D
40. Gulls can often be see swooping over large bodies of water.
A B C D

Structure Section | 71
ELPT_ Level 2

Practice Test 5
1. The difference between libel and slander is that libel is printed while__________.
(A) spoken is slander
(B) is spoken slander
(C) slander is spoken
(D) is slander spoken

2. Great numbers of tiny shelled animals ________________on the ocean floor.


(A) live
(b) living
(c) they will live
(d) if they lived

3. The knee is the joint ______________the thigh bone meets the large bone of the lower leg.
(A) when
(B) where
(C) why
(D) which

4. Closed plane figures like the square or the equilateral triangle can be grouped into a class
____________polygons.
(A) called
(B) to call
(C) is called
(D) call as

5. Acids are chemical compounds that, in water solution, have __________, a corrosive action on
metals, and the ability to turn certain blue vegetable dyes red.
(A) tastes sharp
(B) sharp-tasting
(C) a sharp taste
(D) tasting sharp

6. ______________the history of the tough, strong-willed Nebraska farmer.


(A) Not only is much of the history of Nebraska
(B) Although it is much of the history of Nebraska that is
(C) It is much the history of Nebraska's being
(D) Much of the history of Nebraska is

7. Billie Holiday's reputation as a great jazz-blues singer rests on her ability __________emotional
depth to her songs.
(A) be giving
(B) are given
(C) being given
(D) to give

8. __________1895 did Cornell University begin to offer a degree in ornithology.


(A) Not until
(B) Not since
(C) Until
(D) In

9. Uniform acceleration occurs __________the rate of change remains the same over successive and
equal intervals of time.
(A) according
(B) if
(C) with
(D) under

Structure Section | 72
ELPT_ Level 2

10. People's expectations for a higher standard of living increase ______________.


(A) conditions in their community improve
(B) since conditions in their improving community
(C) conditions improve in their community
(D) as conditions in their community improve

11. Essentially, a theory is an abstract, symbolic representation of __________reality.


(A) what it is conceived
(B) that is conceived
(C) what is conceived to be
(D) that is being conceived of

12. All of the plants now raised on farms have been developed from plants __________wild.
(A) once they grew
(B) they grew once
(C) that once grew
(D) once grew

13. __________relatively costly, the diesel engine is highly efficient and needs servicing infrequently.
(A) Even
(B) It is
(C) Even though
(D) There is

14. __________images out of clay, stone, and metal.


(A) The shaping of sculpture
(B) Sculpting the shapes
(C) To shape sculpture
(D) Sculptors shape

15. __________dates from the end of the eighteenth century.


(A) The modern circus
(B) That the modern circus
(C) While the modern circus
(D) The modern circus that

16. The boiled point of any liquid is determined by the pressure of the surrounding gases.
A B C D
17. The Ranger spacecraft it provided more than 17,000 pictures of the moon.
A B C D
18. Many people who live in New York City thinks that life in a large city offers special advantages.
A B C D
19. The scientific revolution of the early 1900's affected education by change the nature of technology.
A B C D
20. Meadowlarks are about the same size than robins, but they have heavier bodies, shorter tails,
A B C
and longer bills.
D
21. On May 20,1932, Amelia Earhart became the first woman fly solo across the Atlantic Ocean.
A B C D
22. Translated into terms of psychological theory, association has been thought of as the basis of
A B
to learn, conditioning, and creative thinking.
C D
23. The Statue of Liberty was originally proposed in 1865 to commemoration the alliance of France
A B C D
with the American colonies during the American Revolution.

Structure Section | 73
ELPT_ Level 2

24. Reptiles are widely distributed all over the world, but are much abundant in warm regions
A B
and are virtually absent beyond the treeline in the Arctic.
C D
25. Alike light waves, microwaves may be reflected and concentrated.
A B C D
26. Industrial buyers are responsible for supplying the goods and services that an organization
A B C
required for its operations.
D
27. The most easiest process for mining gold is panning, which involves using a circular dish
A B C
with a small pocket at the bottom.
D
28. Farm animals have been regardless by nearly all societies as a valuable economic resource.
A B C D
29. Although it is any longer the big business that it was in the forties, radio continues to be a
A B C
medium of essential communication, especially at the local level.
D
30. The field of dynamics in physics is concerned with a particle's motion in relation to the
A B C
forces acting it.
D
31. In the United States, both the federal and state governments have laws designed to guard
A B
consumers against deceptive advertise.
C D
32. Gore Vidal has steadily pursue a literary career remarkable for its productivity, versatility,
A B C
and unpredictability.
D
33. When overall exports exceed imports, a country said to have a trade surplus.
A B C D

34. Instructors at the school of American Ballet first examine a young applicant's instep to see
A B C
whether it is pliant and shows promising of a good arch.
D
35. Anthropologists agree that our primitive ancestors who inhabited the tropics probably have
A B C
natural protection against the sun.
D
36. Behavior modification techniques work best with problems that manifest itself in overt actions.
A B C D
37. Because they are generally taken simply to obtain a recognizable and relatively clear image, most
A B C
nonprofessional photographs demand few equipment.
D
38. At birth, an infant exhibits a remarkable number of motor response.
A B C D
39. Common to North America, those cinnamon fern is found in wet places.
A B C D
40. The origins of the Democratic party is often traced to the coalition formed behind Thomas
A B C
Jefferson in the 1790's to resist the policies of George Washington's administration.
D

Structure Section | 74
ELPT_ Level 2

Practice Test 6
1. The role of the ear is _________ acoustic disturbances into neural signals suitable for transmission
to the brain.
(A) to code
(B) so that coded
(C) coded
(D) it coding

2. The imagist movement in poetry arose during the second decade of the twentieth century
__________
against romanticism,
(A) when a revolt
(B) as a revolt
(C) a revolt was
(D) that a revolt
3. Virtually ____________ species have biological clocks that regulate their metabolism over a 24-hour
period.
(A) all there are
(B) all
(C) all are
(D) they all

4. According to United States criminal law, insanity may relieve a person from the usual legal
consequences ___________________.
(A) what his or her acts have
(B) of his or her acts are
(C) of his or her acts
(D) what of his or her acts

5. In addition to __________a place where business deals are made, a stock exchange collects
statistics,
publishes price quotations, and sets rules and standards for trading.
(A) being
(B) it is
(C) that which
(D) where is
6. The first inhabitants of the territories _____________ Canada came across the Bering Strait
and along the edge of the Arctic ice.
(A) make up that now
(B) make up now that
(C) that make up now
(D) that now make up

7. ______________ need for new schools following the Second World War that provided the sustained
thrust for the architectural program in Columbus, Indiana.
(A) Since the
(B) To be the
(C) The
(D) It was the

8. The soybean contains vitamins, essential minerals ______________ high percentage of protein.
(A) a
(B) and a
(C) since a
(D) of which a

Structure Section | 75
ELPT_ Level 2

9. Hail is formed when a drop of rain is carried by an updraft to an altitude where _________________
to freeze it.
(A) is the air cold enough
(B) the air cold enough
(C) the cold enough air
(D) the air is cold enough

10.Geometrically, the hyperbolic functions are related to the hyperbola, __________ the trigonometric
functions are related to the circle.
(A) just as
(B) same
(C) similar to
(D) and similar

11. ______________, Kilauea is one of the world's most active volcanoes, having erupted dozens of
times since 1952.
(A) The big island of Hawaii's location
(B) Locates the big island of Hawaii
(C) Located on the big island of Hawaii
(D) On the big island of Hawaii's location

12. Not until the eighteenth century ___________________ the complex chemistry of metallurgy.
(A) when scientists began to appreciate
(B) did scientists begin to appreciate
(C) scientists who were beginning to appreciate
(D) the appreciation of scientists began

13. ___________1810, water-powered textile manufacturing arrived in New Hampshire with the founding of
a company in Manchester that manufactured cotton and wool.
(A) Early
(B) In the early
(C) As early as
(D) When early

14. The settings of Eudora Welty's stories may be rather limited, but ____________ about human nature
is quite broad.
(A) exposes
(B) exposes that
(C) she exposes
(D) what she exposes

15. Lichens grow extremely well in very cold parts of the world ___________________ plants can survive.
(A) where few other
(B) few others
(C) where do few others
(D) there are few others

16. The pear tree has simple, oval leaves that are smoother and shinier than them of the apple.
A B C D
17. In the orbit of a planet around the Sun, the point closest to the Sun is called it the perihelion.
A B C D
18. In the early 1900’s, Roy Harris created and promoted a distinctly American style of classical music
A B C
and greatly influenced a number of composer in the United States.
D
19. The eighteenth century witnessed the emergence of North American ports, particular Boston,
A B
New York,and Philadelphia, as major commercial centers within the British empire.
C D

Structure Section | 76
ELPT_ Level 2

20. Guitarlike instruments have exist since ancient times, but the first written mention of the guitar
A B C
itself is from the fourteenth century.
D

21.The law of biogenesis is the principle what all living organisms are derived from a parent or
A B C
parents.
D
22.Onyx is a mineral that can be recognized its regular and straight parallel bands of white, black,
A B C
or brown.
D

23.There are as many as 200 million insects for every human beings, and in fact their total number
A B
exceeds that of all other animals taken together.
C D

24.Native to South America and cultivated there for thousands of years, the peanut is said to
A B C
have introduced to North America by early explorers.
D

25.Originally canoes were made by the hollowing out of logs and used were for combat as well as
A B C D
transport.

26.Among the symptoms of measles, which takes about twelve days to incubate, are a high fever,
A B C
swelling of glands in the neck, a cough, and sensitive to light.
D
27. Ice crystals in a glacier tends to melt and recrystallize within a brief moment of travel on a downhill
A B C D
glide.

28.Photograph was revolutionized in 1851 by the introduction of the collision process for making
A B C D
glass negatives.

29.The piano is a stringed musical instrument in which the strings are strike by felt-covered hammers
A B C
controlled by a keyboard.
D

30.The sounds used in human languages to create meaning consist of small variation in air pressure
A B C
can be sensed by the ear.
D
31.The mountains, especially the Rocky Mountains, formerly constituted a seriously barrier to east-
A B C
west trade in British Columbia.
D

32.Telescope are frequently used in astronomy to collect light from a celestial object, bring the light
A B
into focus, and producing a magnified image.
C D
33.Diamond is the hardest known substance, so diamond can be cut only by another diamonds.
A B C D

Structure Section | 77
ELPT_ Level 2

34.There are about 350 species and subspecies of birds in danger of become extinct, with a large
A B
number of them, 117 in all, found on oceanic islands.
C D

35.The nineteenth-century romantic movement in art was partially a reaction to what was perceived
A B C
as overemphasis on reasonable and order in neoclassicism.
D
36. Like triglycerides, cholesterol is a type of fat that is both consumed in the diet but manufactured
A B C D
by the body.

37.Both the United States silver dollar and half-dollar, first minted in 1794, had a figure of Liberty
A B C
on one side and a eagle on the reverse side.
D

38.For an advertisement to be effective, its production and placement must to be based on


A B C
a knowledge of human nature and a skilled use of the media.
D

39.While photosynthesis in green plants, light energy is captured and used to convert
A B C
water, carbon dioxide, and minerals into oxygen and energy-rich organic compounds.
D

40.The Democratic Party, the most oldest existing political party in the United States, has played
A B C
a vital role in the nation’s history.
D

Structure Section | 78
ELPT_Level 2
ELPT_Level 2

The purpose of Reading Section is to test your knowledge of the meanings and uses of words in
written English and your ability to understand a variety of reading materials. The reading topics in
Section Three are taken from general and formal English. For the most part, informal and
conversational language is not tested in this section. The questions based on the reading passage
will not require you to have outside knowledge of the topics.

1. SKIMMING

Skimming means reading quickly for general meaning. You skim reading material to find out about the
topic, the main ideas, and the general organization of a passage.

2. SCANNING

Scanning means knowing what information you need to find before you read. Then your eyes move
quickly to find that particular information. You can scan when you are looking for a fact or a detail or a
particular vocabulary word.

Model Test

People are still reading books, and lots of them. The number of new books published keeps
growing-90,000 new titles in the United States during the last year by the latest industry count.
The number is double the total output two decades ago. Reading is still an important part of
our lives.

1. What is this passage about?


A. The amount of reading people do
B. Book reviews
C. Leisure time activities
D. A popular new book

2. How many new books were published in the United States last year?
A. 60,000
B. 900
C. 9,000
D. 90,000

Explanation
The correct answer to question 1 is (A) The amount of reading people do. By reading the first
sentence of the passage you find the phrases reading books and lots of them. As you quickly read the
rest of the paragraph you find these words and ideas repeated in different ways. You could answer
the question by quickly skimming the passage for the general idea.
The correct answer to question 2 is (D) 90,000. The question asks how many, and this tells you to
look for a number. By scanning the passage for a number, you are able to find the correct answer of
90,000.

3. TOPIC AND MAIN IDEAS

Almost all passages contain one question about either the topic or subject, or the main ideas of a
reading passage. This type of question is usually the first question about the passage. These
questions ask about the whole passage, not just a part of the passage. In order to correctly answer
these questions, you should use the skimming and scanning strategies.

To answer questions about the topic and the main ideas of a passage, follow these steps:
1. Read the first several sentences for the topic and main ideas.
2. Read the last sentences for the conclusion and a possible restatement of the topic and the
main ideas.
3. Skim the rest of the passage for the key words that will confirm the topic and the main ideas
and show the organization of the passage.
4. Read the first question about the passage and answer it.
5. Read the answer choices. Eliminate any answers that are definitely wrong, and choose the
best answer from the remaining ones.

Reading Section | 79
ELPT_Level 2

Model Test

In the critical area of food production, new cooperative efforts in agricultural research and
development are paying off. Food scientists are discovering that humankind is nowhere near
the limits of plant, livestock, and soil productivity. There is still room to boost yields and learn
how to use more efficiently the earth’s acreage for animal and crop husbandry. Investigation
of irrigation procedures, pest control, intercropping, and multi cropping are several areas that
are providing useful information for the world’s farmers.

1. What is the topic of the passage?


A. Agricultural research and food production.
B. Farmers throughout the world.
C. Food scientists.
D. Investigation of irrigation procedures.

2. The passage supports which of the following statements?


A. New information from agricultural research can increase world food production.
B. Food production is declining around the world.
C. Agricultural research and development will be expensive.
D. There are few initiatives to help today’s farmers.
Explanation
Question 1 asks about the topic of the passage, which is the most general statement you make about
the passage. In the first sentence of this passage the phrases food production and agricultural
research and development give you this basic concept. Answer (B), (C) and (D) are too specific to be
the topic of the passage. Answer (A) is the most general statement in the passage
Question 2 asks you about idea of the passage. A main idea of a passage tells you something more
about the topic. In this passage the author tells us that efforts in agricultural research and
development (in the area of food production) are paying off, which means they are beneficial. The
correct answer is (A) New information from agricultural research can increase world food production.
Answer (B) and (D) are not true, and answer (C) is not mentioned in the passage.

4. VOCABULARY IN CONTEXT

The Reading Section tests your knowledge of vocabulary by asking questions about certain words in
the reading passages. For the most part, you can answer these questions about word meanings by
using the context of the sentence in which the word was found or of the passage in general.

Model Test

The hippopotamus spends as much time in the water as on land. It swims and dives well,
but prefers to spend its days submerged in the water with only its eyes and ears sticking out. It
lives in the swamps, streams and marshes of tropical Africa.

1. As used in lines 2 and 3, the phrase “submerged in the water” means that the animal is
A. swimming in the water
B. standing covered with water
C. floating in the water
D. wading in water

Explanation
Since all of the answers choices involve the water, you must read carefully for context clues in the
sentence. The sentence states that it spends its days .... in the water with only its eyes and ears
sticking out. Using the full context of the sentence, we read that the hippo swims and dives well, but
prefers to spend its days submerged ....; therefore, answer (A) swimming .... is not the same as
submerged. You know that answers (C) floating on the water and (D) wading in water both expose
more than the eyes and ears of the hippo, and are incorrect. The correct answer choice is (B)
standing covered with water.
You reach this conclusion by inferring information from the passage and from the answer choices and
by using logical reasoning.
NOTE: In some questions, however, you cannot rely on context for figuring out the meaning of the
words. In this case your knowledge of synonyms, word forms, Latin and Greek roots, prefixes, and
suffixes will help you to answer the questions about word meanings.

Reading Section | 80
ELPT_Level 2

In the following example sentences, common context cues in written discourse are illustrated for the
word “adversity”.

Contrast It was adversity, not prosperity, that shaped the lives of early American pioneers.
Example Pioneer women met adversity head on; they used paper for windows, carried water for
miles for the weekly washing, and made do with corn meal rather than flour for the
daily baking.
Definition The adversities for the winter – that is, the hardships and calamities that the family
lived with daily – disappeared with the warmth of spring.
Appositive Locusts (the dreaded migratory grasshoppers of the prairies) could destroy a family’s
whole corn crop in a matter of minutes. From these adversities, the trials and
tribulations of the times, came the legends of the hardy American pioneers.
Series The diaries of the early farmers told of hardships, calamities, and adversities that
made their lives miserable: Lack of water for daily use, sudden epidemics that wiped
out whole families, and severe blizzards that kept people house-bound for weeks.
Parallelism The settlers never gave up. They struggled through the early years, dealt with their
difficulties, and in the end, overcame their adversities.
WH marker (where, when, which, who, whom, that) Relative pronouns often suggest the meaning
of words and phrases.
e.g.,The cold-cellar was where the farmers stored their fruits and vegetables for the
winter.
Reason logically to make sense of unfamiliar words and phrases in a reading passage by using
other words in the sentence and the passage.
e.g., The settlers never gave up. They struggled through the early years, dealt with
their difficulties, and in the end, overcame their adversities.
The general meaning of adversities can be deduced by considering the “core”
meanings that the other key words in the context share: persistence (never
gave up, struggled, overcame) and hardship (struggle, difficulties).
Use your knowledge of the world and personal experience.
e.g., Pioneer women met adversity head on; they used paper for windows, carried
water for miles for the weekly washing, and made do with corn meal rather than
flour for the daily baking.
What you know about windows, washing, and baking from your personal
knowledge and experience will help you to infer that the life of the pioneer
woman was very difficult and that adversity must include this concept of
difficulty or hardship.

5. PURPOSE AND ORGANIZATIONAL PATTERN

Some Reading Comprehension questions ask about the author’s purpose for the passage or about the
organization of the passage.

The purpose of passage is the reason the author wrote the passage or the intent of the author in
writing the passage. The organizational pattern of a reading passage is the way that the author
arranges the information to carry out his or her writing rather than the purpose of the whole passage.

Model Test
The art of writing itself is a good example of what students of the past call independent
invention, since systems of writing have evolved in isolation at different times in different parts
of the world. For example, one system—the Chinese ideogram—can be traced to its origin in
archaic signs engraved on the scapular bones of sheep or the shells of turtles in the second
millennium B.C. as a means of asking questions of heaven. Roughly 1,000 years later an
entirely independent system of writing arose halfway around the world in Mesoamerica. It
combined a simple system of numerical notation with complex hieroglyphs and was
principally used to indicate the dates of various events according to an elaborate calendrical
system.

1. What is the purpose of this passage?


A. To show that writing is an example of independent invention.
B. To explain the origin of writing
C. To describe two systems of writing
D. To compare writing in China to writing in Mesoamerica

Reading Section | 81
ELPT_Level 2

2. Which of the following best describes the organization of the passage?


A. A comparison of two competing systems.
B. An examination of a problem
C. A statement supported by examples.
D. A chronological development.

3. Where in the passage does the author describe the early writing system of Mesoamerica?
A. Lines 1-3
B. Lines 3-6
C. Lines 5-7
D. Lines 7-9

Explanation
The correct answer to Question 1 is (A). The sentence …writing is an example of independent
invention contains both topic and main idea of the passage. The use of examples to support this is
represented by To show that. Answer (B) and (C) are not true, and answer (D) is not specific enough
to this passage.

The correct answer to Question 2 is (C). The first sentence of the passage states that systems of
writing have evolved in isolation at different times in different parts of the world. From this we
understand the topic (systems of writing) and the main idea (evolved in isolation). Answer (C) correctly
represents the organization of the passage answer (A) is incorrect because there is no comparison
made, and the systems of writing are not competing. Answer (B) is incorrect because the main idea is
not a problem but a statement of fact. Answer (D) is incorrect because although dates and years are
mentioned in the passage, they are used to support the main idea and not the organizational pattern.
The correct answer to Question 3 is (D). in line 7 we find the word Mesoamerica. The question asks
about the description of the writing system of Mesoamerica, which is found in line 7-9.

6. REFERENCE WORDS

Reference words are those words in a passage that refer back to words or phrases mentioned earlier
in the passage or refer forward to words or phrases that will be introduced.

In determining the referent (the word or phrase that reference words refer to), you should use both the
structure and the meaning of the sentence.

Throughout the year, chimpanzee food is quite varied, but it is mainly vegetable material. At
times, however, the Gombe Park is loaded with insects-termites, ants, caterpillars- and the
chimpanzees will eat huge numbers of them. The chimpanzees’ really remarkable behavior
appears when they gather termites. According to Suzuki and van Lawick-Goodall, when
chimpanzees see that termites have pushed open their tunnels on the surface, they will go
off to find a suitable termiting tool. It may look simple, but the job takes skill and patience.

1. The word “their” in line 6 refers to?


A. the termites’
B. the tunnels’
C. the chimpanzees’
D. Suzuki’s and van Lawick-Goodall’s

2. The pronoun ‘they” in line 6 refers to?


A. Suzuki and van Lawick-Goodall
B. termites
C. tools
D. chimpanzees

3. The pronoun “It” in line 6 refers to


A. suitable termite tool
B. The job
C. Skill
D. patience

Reading Section | 82
ELPT_Level 2

Explanation
The answer to Question 1 is (A) the termites’. This question might be confusing because you know
that their can refer to a plural noun. The sentence, Suzuki and ........chimpanzees, .... termites are all
plural concepts. Logical reasoning will tell you t hat termites are the ones that live in tunnels.

The answer to Question2 is (D) Chimpanzees. In this case, the plural pronoun they could refer to
either chimpanzees or termites. The meaning of the whole passage, however, makes it clear that the
chimpanzees are looking for a tool to gather termites, and they refers to chimpanzees.

The answer to Question 3 is (B) the job. The pronoun it could be possibly refer to any singular object,
and at first glance you might choose a suitable tool. However, as you read on, it becomes clear that
what looks simple is the job. In this sentence the pronoun refers forward.

7. DETAIL and FACTUAL INFORMATION


There are two types of fact and detail questions: questions about what IS true according to the
information and questions about what IS NOT true. Answer choices for questions about facts and
details may use the exact words of the passage, but more often they are restatements of the
information and require that you know synonyms and related words.

Model Test

Throughout the year, chimpanzee food is quite varied, but it is mainly vegetable material. At
times, however, the Gombe Park is loaded with insects-termites, ants, caterpillars- and the
chimpanzees will eat huge numbers of them. The chimpanzees’ really remarkable behavior
appears when they gather termites. According to Suzuki and van Lawick-Goodall, when
chimpanzees see that termites have pushed open their tunnels on the surface, they will go off
to find a suitable termiting tool. It may look simple, but the job takes skill and patience.

1. According to the passage, which of the following are NOT mentioned as part of the
chimpanzees’ diet?
A. Termites
B. Vegetable materials
C. Ants
D. Mosquitoes

2. The author states in the passage that the chimpanzees’s most remarkable behavior can
best be seen
A. when they are hungry
B. as they are resting
C. when they are looking for termites
D. in the spring

3. According to the author, when chimpanzees gather termites they show


A. a dependence on each other
B. remarkable strength
C. understanding and caring
D. ability and persistence

Explanation
The correct answer to Question 1 is (D) Mosquitoes. This question asks to you to identify the answer
that is NOT in the passage. By knowing where in the passage the food chimpanzees eat is mentioned,
you can quickly look at those sentences and match the items in the sentence with those in the answer
choices. Mosquitoes are not mentioned in the passage.

NOTE: in the questions that ask what is NOT in the passage, information that is true is not the correct
answer.
The correct answer to Question 2 is (C) when they are looking for termites. To answer this question
you need to match the words chimpanzees’ most remarkable behavior in the question with those
words in the passage. This will tell you in what part of the passage you will find the answer. After
careful reading of the sentence, you can match the information in the passage with the answer choice.
In this case the passage states when they gather termites, and restatement of this is found in answer
choice (C).

Reading Section | 83
ELPT_Level 2

The correct answer to Question 3 is (D) ability and persistence. In this question you are asked to find
what chimpanzees show rather than do when they gather termites. The last sentence of the passage
states that the job (of gathering termites) takes skill and patience. Scanning the answer choices will tell
you that the best restatement of the information in the passage is ability and persistence.

8. MAKING INFERENCE

Making inference (a prediction or conclusion) ask you to use your understanding of the facts and
details which are directly stated in a reading passage. Information that is not directly stated in the
passage is said to be implied by the author. Questions about implied information may be about a part
of the passage or about what came before or will come after the passage. You may be asked to draw
conclusions about the passage itself, or to make predictions about another related situation.

Model Test

Think about the following sentence and questions:

In last year’s competition of the five contestants chosen, one was from White Springs, two from
other towns in Idaho, and the rest from neighboring areas of the Pacific Northwest.

Using the facts of the text itself, you can make several inferences (conclusions based on facts)
about this sentence.

1. What or where is White Springs?


We can infer White Springs is a town in Idaho based on the phrase other towns in Idaho.
2. Where is Idaho?
We can infer that both White Springs in Idaho are in the Pacific Northwest, based on from
neighboring areas of the Pacific Northwest.
3. How many contestants are from the rest of the Pacific Northwest?
We can infer there where two contestants from the neighboring areas of the Pacific North
West by using simple arithmetic.

9. ATTITUDE OF THE AUTHOR AND TONE OF THE PASSAGE

A question on the test that asks about the attitude of the author or the tone of the passage requires
that you think about the whole passage. Most often reading passage give information in an objective
way, and the author’s point of view is neutral. The tone of these passages is informational. However,
in some passages the author may express how or she feels about the topic, the ideas, or the issue
that he or she has written about.

Exercise
Match the word on the left with the best explanation on the right. Then, write the letter of the correct
explanation in the space provided. Use your dictionary if you need help.

____ 1. emotional A. disinterested; neutral


____ 2. informative B. trying to change an attitude, an opinion, or a behavior
____ 3. humorous C. objective; factual reporting; neutral
____ 4. descriptive D. angry
____ 5. sarcastic E. giving chronological facts about the pas
____ 6. complimentary F. showing emotion; happy, sad
____ 7. critical G. making fun of something
____ 8. indifferent H. disagreeing with something
____ 9. sympathetic I. giving reasons and support for a point
____ 10. cautionary J. funny
____ 11. persuasive K. worried; very interested; distressed
____ 12. explanatory L. agreement with a point; showing understanding
____ 13. outraged M. describing something – a process, a place, a person
____ 14. concerned N. giving a warning
____ 15. historical O. showing approval or praise

Reading Section | 84
ELPT_Level 2

Practice Test 1
Allow yourself 50 minutes to complete this checkpoint test. There are six passages and 46 questions.
Circle the letter of the correct answer to each question.
Questions 1 – 9

Spiders produce three basic types of webs. The sheet web is a two-dimensional layer of
threads seemingly laid out at random. The space web is a three-dimensional, wispy structure. The orb
web, by far the most familiar, is two-dimensional cartwheel pattern.
Of the 30,000 spider species, some 6,000 are orb spinners. For three decades Dr. Peter N.
Witt has studied orb spinners, especially a species called Areneus diadematus, and their webs. Witt is
a German-born medical doctor and self-taught arachnologist, whose passion is to understand the
ways of spiders. Witt has delved deeply into the behavior of spiders and vastly expanded our
knowledge about orb spinners and their webs. Some of his findings have even amazed other
arachnologists.
“We have actually compared human building activities to spider building, and we find an
enormous amount of parallel between the two, “Witt says. For one thing, just like their human
counterparts in the building trades, orb spinners erect a form of removable scaffolding as they weave
their webs.
Orb spinners are solitary creatures who dwell one to a web. The web is home, food source,
and mating ground, and it is guarded aggressively. When a male arrives at mating time, the courtship
ritual is an intricate set of advances and retreats until the female is finally won over and no longer tries
to kill her would-be lover.
Orb spinners each weave a new web every day, working in the predawn darkness and
executing the distinctive pattern of concentric circles and radial lines in a half hour or less. There is
nothing as important as web building, because without the web there is no food” Witt says.

1. The topic of this passage is


(A) Spiders 5. The word “their” in line 11 refers to
(B) Different types of webs spiders make (A) humans who build
(C) Dr. Peter N. Witt (B) other arachnologists
(D) Orb spinners and their webs (C) Witt and his associates
(D) orb spinners
2. According to the passage, the difference
between the sheet web and the orb web is 6. According to the passage, web- making
(A) The pattern by spiders and human building activities are
(B) The size (A) both dependent on removable
(C) The texture scaffolding
(D) The length of threads spun by (B) hard to compare
the spiders (C) simple to analyze
(D) lengthy procedures
3. The phrase “at random” in line 2 is closest
in meaning to 7. The word “it” in line 15 refers to
(A) arbitrarily (A) the web
(B) quickly (B) the food source
(C) deftly (C) the female spider
(D) incongruously (D) the mating ground

4. We can infer from the passage that an


arachnologist is
(A) a photographer
(B) a medical doctor
(C) a person who studies spiders
(D) a person who intensely dislikes spiders

Reading Section | 85
ELPT_Level 2

8. We can infer that female orb spinner is 9. We can conclude from the passage
NOT that the purpose of webs is
(A) hard-working (A) to initiate courtship of spiders
(B) cautious (B) to engage spiders in useful activity
(C) solitary (C) to provide a way for spiders to
(D) easily wooed entrap food
(D) to display artistic talents of spiders

Questions 10 – 17

For any business, the cost of transportation is normally the largest single item in the overall
cost of physical distribution. It doesn’t necessarily follow, though, that manufacturer should simply pick
the cheapest available form of transportation. Many companies today use the total physical distribution
concept, an approach that involves maximizing the efficiency of physical distribution activities while
minimizing their cost. Often, this means that the company will make cost tradeoffs between the various
physical distribution activities. For instance, air freight may be much more expensive than rail
transport, but a national manufacturer might use air freight to ship everything from a single warehouse
and thus avoid the greater expense of maintaining several warehouses.
When a firm chooses a type of transportation, it has to bear in mind its other marketing
concern-storage, financing, sales, inventory size, and the like. Transportation, in fact, can be an
especially important sales tool. If the firm can supply its costumers’ needs more quickly and reliably
than its competitors do, it will have a vital advantage: so it maybe more profitable in the long run to pay
higher transportation costs, rather than risk the loss of future sales. In addition, speedy delivery is
crucial in some industries. A mail-order distribution sending fruit from Oregon to Pennsylvania needs
the promptness of air freight. On the other hand, a manufacturer shipping lingerie from New York to
Massachusetts may be perfectly satisfied with slower (and cheaper) truck or rail transport.

10. The passage supports which of the 12. The author states in the passage that the
following statements? total physical distribution concept
(A) Business should use the least expensive (A) is based on the capability and cost-
form of transportation effectiveness of a transportation system
(B) Transportation is an important aspect of (B) advocates the use of air freight because
business of its efficiency
(C) Rail transportation is usually better for (C) suggests trading goods for
companies because it is cheaper than transportation services
air transport (D) relies on using warehouses for storing
(D) Most manufacturers choose the fastest goods
form of delivery
13. The phrase “cost tradeoffs” in line 5 means
11. According to the passage, all of the that companies
following would influence the type of (A) a sometimes engage in bartering goods
transportation that a company might choose (B) may choose an expensive form of
EXCEPT transportation if costs can be cut in
(A) the type of goods to be shipped another area
(B) the expense of the shipping (C) prefer warehouses to air transportation
(C) the time it takes for delivery (D) rarely use rail transport
(D) the size of its warehouses

Reading Section | 86
ELPT_Level 2

14. It can be inferred from the passage that 16. The word “its” in line 11 refers to which of
transportation is the following?
(A) important to continued successful sales (A) Competitors
(B) independent of the other business (B) Firm
concerns (C) Customers
(C) not used effectively by business (D) Sales tool
concerns
(D) too expensive for most mail- order 17. This passage would probably be assigned
industries to use reading in which of the following academic
courses?
15. We can conclude from the passage that a (A) Marketing
business that deals in perishable goods (B) Statistics
would probably choose to ship by (C) Mechanical engineering
(A) rail (D) History
(B) truck
(C) air freight
(D) any type of cheap transport

Questions 18 – 23

Insect control is only the one of the problems being addressed by cooperative agricultural
research teams. Besides the problem of pests, great quantities of food are lost by improper threshing
methods and by poor handling, storage, and food preservation.
Fermentation and mold during wet-season crop harvesting and badly organized drying and
milling facilities lose much grain. Grain dryers that work for North America may be useless in tropical
climates. Grain bins designed for gentle prairie winds are no good for Africa’s blazing sun. Developing
the right storage facilities for local conditions is a great need.

18. This passage mainly discusses


(A) Insect control 22. It can be inferred from the passage that
(B) Food harvesting and storage (A) Agricultural facilities used in North
(C) Tropical climates America are not appropriate in all parts
(D) Grain loss of the world
(B) Drying food is easy in tropical climates
19. The word “addressed” in line 1 is closest (C) African storage facilities are superior to
in meaning to North American ones
(A) dealt with (D) Pest control is the biggest problem
(B) mailed to facing agricultural research today
(C) neglected
(D) marketed 23. The author implies in the passage that
agricultural research
20. According to the passage, one problem (A) disregards climatic conditions in its
leading to crop loss is studies
(A) Poor planting methods (B) is making insignificant contributions to
(B) Damage from vandals tropical agriculture
(C) Proper transportation of food products (C) will continue to investigate storage
(D) Harvesting procedures during rainy facilities for food
seasons (D) is primarily taking place in North
America
21. What did the paragraph preceding this
passage most probably discuss?
(A) Proper threshing methods
(B) Food preservation
(C) Insect control
(D) Agriculture in North America

Reading Section | 87
ELPT_Level 2

Questions 24 – 31

Noise is a given in our everyday lives. From the moment the alarm clock buzzes or the
garbage trucks rouse us, to the time we fall asleep despite the neighbor’s stereo, we accommodate
noisy instruction
Studies suggest that we pay a price for adapting to noise: higher blood pressure, heart rate,
and adrenaline secretion – even after the noise stops; heightened aggression; impaired resistance to
disease; a sense of helplessness. In term of stress, unpredictability is an important factor. Studies
suggest that when we can control noise, its effects are much less damaging
Although there are no studies on the effects of quiet in repairing the stress of noise, those who
have studied the physiological effect of noise believe that quiet provides an escape. Most people who
work in a busy and fairly noisy environment love quiet and need it desperately
We are so acclimated to noise that complete quiet is sometimes unsettling. You might have
trouble sleeping on vacation in the mountains, for example, without the background sounds of traffic.
But making the effort to find quiet gives us a chance to hear ourselves think, to become attuned to the
world around us, to find peacefulness and calm. It provides a serene antidote to the intrusively loud
world we live in the rest of the day.

24. This passage mainly discusses 28. The author indicates in the passage that
(A) Life in the city stress from noise occurs mainly
(B) The effect of noise on our lives (A) in the morning
(C) Diseases related to stress (B) when we can’t control it
(D) Why quiet is hard to find (C) in the mountains
(D) from traffic
25. We can infer from the passage that the
author is writing for which group of people? 29. The phrase “pay a price for” in line 4 could
(A) people who live in the country best be replaced by which of the following?
(B) vacationers (A) suffer from
(C) city-dwellers (B) lose money because of
(D) doctors (C) work hard
(D) indulge in
26. What is the author’s attitude toward noise in
the passage? 30. The word “unsettling” in line 11 could best
(A) Humorous be replaced by which of the following?
(B) Critical (A) rewarding
(C) Emotional (B) necessary
(D) Indifferent (C) unavoidable
(D) disturbing
27. According to the passage, noise causes all
of the following EXCEPT 31. The word “it” in line 14 refers to
(A) oversleeping (A) peacefulness and calm
(B) stress (B) the world around us
(C) higher blood pressure (C) quiet
(D) heightened aggression (D) thinking

Reading Section | 88
ELPT_Level 2

Questions 32 – 39

The killer sea waves known as tsunamis are so quiet in their approach from afar, so seemingly
harmless, that until recently their history has been one of surprise attack.
Out in the middle of the ocean, the distance between tsunami wave crests can be 100 miles
and the height of the waves no more than three feet: mariners can ride one and suspect nothing. At
the shoreline, the first sign is often an ebbing of the waters that leaves fish stranded and slapping on
the bottom. However, this is not a retreat but rather a gathering of forces. When the great waves finally
do strike, they rear up and batter harbor and coast, inflicting death and damage
These seismic sea waves – or tidal waves, as they are sometimes called – bear no relation to
the moon or tides. And the word “tsunami” Japanese for “harbor wave” relates to their destination
rather than their origin. The causes are various: undersea or coastal earthquakes, deep ocean
avalanches or volcanism. Whatever the cause, the wave motion starts with a sudden jolt like a whack
from a giant paddle that displaces the water. And the greater the undersea whack, the greater the
tsunami’s devastating power.
In 1883, Krakatoa volcano in the East Indies erupted, and the entire island collapsed in 820
feet of water. A tsunami of tremendous force ricocheted around Java and Sumatra, killing 36,000
people with walls of water that reached 115 feet in height.
In 1946 a tsunami struck first near Alaska and then, without warning, hit the Hawaiian Islands,
killing 159 people and inflicting millions of dollars of damage. This led to the creation of the Tsunami
Warning System, whose nerve center in Honolulu keeps a round-the-clock vigil with the aid of new
technology. If the seismic sea waves are confirmed by the Honolulu center, warnings are transmitted
within a few hours to all threatened Pacific points. While Tsunami damage remains unavoidable, lives
lost today are more likely to be in the tens than in the thousands. Tsunamis have been deprived of
their most deadly sting – surprise.

32. The author’s main point in this passage is 35. The word “devastating” in line 13 is closest
that in meaning to
(A) seismic sea waves today are carefully (A) shocking
monitored and cause less damage than (B) destructive
in less damage than in the past (C) relative
(B) tsunamis can do little damage when (D) accelerating
they strike
(C) there is little possibility of avoiding tidal 36. According to the passage, all of the
waves once they are in motion following are possible causes for seismic
(D) we need better equipment to track the sea waves EXCEPT
movements of tsunamis (A) earthquakes near coastline
(B) tides
33. According to the passage, seismic sea (C) avalanches
waves (D) volcanoes
(A) are easily detected by fisherman
(B) are named “tsunami” for the origin of the 37. The phrase “a round-the-clock vigil” in line
wave in the harbor 19 is closest in meaning to
(C) are called tidal waves because of their (A) a good count
relation to the moon (B) a constant watch
(D) originate far from the place where they (C) a careful record
strike (D) an open line

34. In line 12, why does the author mention “a


giant paddle”?
(A) to make a comparison
(B) to give a definition
(C) to draw a conclusion
(D) to make a suggestion

Reading Section | 89
ELPT_Level 2

38. The tone of this article can best be 39. According to the passage, the Tsunami
described as Warning System war created because of
(A) informative (A) The availability of new technology
(B) exaggerated (B) The nervous state of people in Honolulu
(C) indignant (C) The occurrence of the 1946 tsunami
(D) humorous (D) The loss of millions of lives

Questions 40 – 46

For several decades, psychologists have been doing extensive research on a subject that
affects millions of us: hobbies. According to their findings, a person’s choice of hobby can be almost
as revealing as his reaction to an inkblot.
Investigators found that a clearly distinguishable pattern exists between hobby preferences
and personality. Scientists now say that they are in a position to study your hobby and come out with a
fairly accurate estimate of your emotional maturity, level of intelligence, and distinguishing personality
traits.
This is because people generally pick a hobby of their own free will. As a parallel, a person
choosing a mate employs a method of selection that reflects his or her intellectual and emotional
maturity; the same process is at work in choosing a hobby.
A hobby is never a task, but a form of living expression that complements and augments one’s
own personality.

40. This passage mainly discusses 44. The word “this” in line 8 refers to
(A) Psychologists’ preferred hobbies (A) the ability of scientists to match hobbies
(B) Benefits of having a hobby with personal characteristics
(C) Hobbies and personality traits (B) the need for people to choose an
(D) Useful leisure time activities appropriate hobby
(C) extensive research of psychologists
41. The word “findings” in line 2 is closest in (D) the ability of scientists to give advice on
meaning to how free time is best used
(A) inventions
(B) decisions 45. According to the passage, a person’s
(C) interests choice of hobby can tell scientists about all
(D) results of the following
EXCEPT
42. It can be inferred from the passage that a (A) level of intelligence
reaction to an inkblot (B) probable choice of mate
(A) is used to reveal a person’s personality (C) emotional maturity
(B) is one kind of hobby (D) special personality traits
(C) is being extensively investigated today
(D) is the author’s favorite hobby 46. As used in line 11, the word “augments” is
closest in meaning to
43. The author implies in the passage that (A) reveals
hobbies are (B) misconstrues
(A) only for the emotionally mature (C) extends
(B) something most people have (D) affects
(C) as important as mate
(D) very time-consuming endeavors

Reading Section | 90
ELPT_Level 2

Practice Test 2
Directions :In this section you will read several passages. Each one is followed by a number of
questions about it. For question 1 – 50, you are to choose the one best answer, (A), (B), (C), or (D), to
each question. Then, on your answer sheet, find the number of the question and fill in the space that
corresponds to the letter of the answer you have chosen.

Questions 1-10
One of the greatest problems for those settlers in Nebraska in the last quarter of the previous
century was fuel. Little of the state was forested when the first settlers arrived and it is probable that by
1880, only about one-third of the originally forested area remained, down to a mere 1 percent of the
state’s 77,000 square miles. With wood and coal out of the question, and with fuel needed year round
for cooking, and during the harsh winter months for heating, some solution had to be found.
Somewhat improbably, the buffalo provided the answer. Buffalo chips were found to burn
evenly, hotly, and cleanly, with little smoke and, interestingly, no odor. Soon, collecting them became a
way of life for the settlers` children who would pick them up on their way to and from school, or take
part in competitions designed to counteract their natural reluctance. Even a young man, seeking to
impress the girl he wanted to marry, would arrive with a large bag of chips rather than with a box of
candy or a bunch of flowers.

1. What is the main topic of this passage?


(A) The solution to the Nebraska settlers fuel 5. Which of the following does the author not
problem. express surprise at?
(B) Life in Nebraska in the late nineteenth (A) The children needed competitions to
century. stimulate them.
(C) The importance of the American buffalo. (B) The buffalo chips gave off no smell.
(D) Deforestation in Nebraska in the late (C) Buffalo chips were the answer to the
nineteenth century. settlers’ fuel problem.
(D) Young men took bags of buffalo chips to
2. Which of the following statements is not true their girlfriends.
according to the passage?
(A) Nebraska was not a densely-forested 6. The word “previous” in line 1 could best be
state even before the settlers arrived. replaced by?
(B) The children enjoyed collecting the (A) separated.
buffalo chips. (B) simultaneous.
(C) The children spent a lot of time (C) preceded
collecting the chips. (D) installed remained.
(D) Buffalo chips were satisfactory as a fuel.
7. The word “remained“ in line 3 is closest in
3. According to the passage, how much of meaning to
Nebraska was forested when the first settler (A) sustained.
arrived? (B) endured.
(A) About 33 percent (C) ruined.
(B) About 1 percent (D) redeemed.
(C) About 66 percent
(D) About 3 percent 8. The word “them” in the phrase “collecting
them” in line 7 refers to
4. The passage implies that buffalo chips were (A) little smoke.
needed (B) those settlers.
(A) in greater amounts in summer. (C) buffalo chips.
(B) in greater amounts in winter. (D) the settlers’ children.
(C) only in summer.
(D) only in winter.

Reading Section | 91
ELPT_Level 2

9. The works “pick up” in line 8 means 10. The word “their” in the phrase “on their way
(A) take away from. to and from school” in line 8 refers to
(B) take off. (A) buffalo chips.
(C) take along with. (B) the settlers’ child.
(D) take out. (C) the settlers.
(D) the children of settlers.

Questions 11-15
One of the most extraordinary of the people discussed was Kansas City-born Ted Serios, who
was in his mid-forties when introduced to Jule Eisenbud, Professor of Psychiatry at the Denver
Medical School, in 1963.
Over the ensuing three years, Eisenbud proved that Serios was endowed with an
extraordinary ability to produce recognizable images on film of distant objects by merely staring with
intense concentration into a camera. These “thoughtgraphs”, eventually numbering several hundred,
involved people, buildings, landscapes, or machines and were produced under carefully controlled
conditions in the presence of scores of reputable witness, some of whom were hostile. The
precautions to eliminate the possibility of fraud included medical examinations, x-rays, and tying
Serios up in a strait jacket or stripping him naked.
In spite of the rigorous scrutiny to which Serios offered, but the tests have excluded all the
obvious kinds of electromagnetic radiation which had originally been viewed offering as the most likely
explanation.

11. Ted Serios was born in Kansas City in? 14. The paragraph preceding this one most
(A) 1945 probably discussed
(B) 1918 (A) a general account of a meeting devoted
(C) 1948 to people with unusual powers.
(D) 1963 (B) previous research done by Jule
Eisenbud.
12. In order to produce his pictures on film, (C) how Ted Serios developed his incredible
Serios needed ability.
(A) several hundred people. (D) Ted Serios Kansas City childhood.
(B) carefully-controlled conditions.
(C) a number of reputable witnesses. 15. It can be inferred from the passage that
(D) a camera. (A) later studies explained exactly how
thoughtgraphy works.
13. According to the passage, the medical (B) Ted serios has been able to teach
examinations several hundred people how to produce
(A) consisted of x-rays. pictures using his method.
(B) were to keep Serios from becoming ill. (C) strenuous efforts were made to see
(C) were part of tests to guarantee scientific whether Serios was using some kinds of
validity. tricks.
(D) required that Serios be stripped naked. (D) an obvious type of electromagnetism
was subsequently shown to have been
the method Serios used.

Reading Section | 92
ELPT_Level 2

Questions 16-20

In what now seems like the prehistoric times of computer history, the early post-war era, there
was a quite widespread concern that computers would take over the world from man one day. Already
today, less than forty years later, as computers are relieving us of more and more of the routine tasks
in business computers and are reluctant to challenge their authority. Indeed, they behave as if they
were hardly aware that wrong buttons may be pushed, or that a computer way simply malfunction.
Obviously, there would be no point in investing in a computer if you had to check all its
answers, but people should rely on their own internal computers and check the machine when they
have the feeling that something has gone away. Questioning and routine double checks must continue
to be as much a part of good business as they were in pre-computer days. Maybe each computer
should come with the following warning: for all the help this computer maybe provide, it should not be
seen as a substitute for fundamental thinking and reasoning skills.

16. What is the main purpose of this passage? 19. An “internal computer” is
(A) to look back to the early days of (A) a computer used exclusively by one
computers. company for its own problems.
(B) to explain what technical problems may (B) a person’s store of knowledge and the
occur with computers. ability to process it.
(C) to discourage unnecessary investment (C) the most up to date in house computer a
in computers. company can buy.
(D) to warn against a mentally lazy attitude (D) a computer from the post war era which
towards computers. is very reliable.

17. According to the passage, initial concerns 20. It can be inferred from the passage that the
about computers were that they might? author would disapprove of
(A) lead us into the post-war era. (A) computers science courses in high
(B) be quite widespread. schools
(C) take control. (B) businessmen and women who use
(D) take over routine tasks. pocket calculators
(C) maintenance checks on computers
18. The passage recommends those dealing (D) companies which depend exclusively on
with computers to computers for decision making.
(A) be reasonably skeptical about them.
(B) check all their answers.
(C) substitute them for basic thinking.
(D) use them for business purposes only.

Reading Section | 93
ELPT_Level 2

Question 21-30

It has long been known that when exposed to light under suitable conditions of temperature
and moisture, the green parts of plants use carbon dioxide from the atmosphere and release oxygen
to it. These exchanges are the opposite of those that occur in respiration. The process is called
photosynthesis, carbohydrates are synthesized from carbon dioxide and water by the chloroplasts of
plants cell in the presence of light. In most plants, the water used in photosynthesis is absorbed from
the soil by the roots and Tran located through the xylem of the root and stem to the leaves. Except for
the usually small percentage used in respiration, the oxygen released in the process diffuses out of
the leaf into the atmosphere through the stomachs. Oxygen is the product of the reaction. For each
molecule of carbon dioxide used, one molecule of oxygen is released. A summary chemical equation
of photosynthesis is: 6CO + 6H2O C6H12O6 = 6O2
As a result of this process, radiant energy from the sun is stored as chemical energy. In turn,
the chemical energy is used to decompose carbon dioxide and water. The products of their
decomposition are recombined into a new compound, which is successively built up into more and
more complex substances. After many intermediate steps, sugar is produced. At the same time, a
balance of gases is preserved in the atmosphere.

21. Which title best expresses the ideas in this


passage? 27. The word “their” in line 12 refers to
(A) A chemical Equation. (A) radiant energy and chemical energy.
(B) The Process of Photosynthesis. (B) carbon dioxide and water.
(C) The Parts of Vascular Plants. (C) products.
(D) The Production of Sugar. (D) complex substances.

22. In photosynthesis, water 28. The word “successively” in line 13 is closest


(A) must be present. in meaning to
(B) is produced in carbohydrates. (A) with effort.
(C) is stored as chemical energy. (B) in a sequence.
(D) interrupts the chemical reaction. (C) slowly.
(D) carefully.
23. Which process is the opposite of
photosynthesis? 29. Besides the manufacture of food for plants,
(A) Decomposition. what another benefit of photosynthesis?
(B) Systemization. (A) It produces solar energy.
(C) Diffusion. (B) It diffuses additional carbon dioxide into
(D) Respiration. the air.
(C) It maintains a balance of gases in the
24. The combination of carbon dioxide and atmosphere.
water to form sugar results in an excess of (D) It removes harmful gases in the
(A) water. atmosphere.
(B) oxygen.
(C) carbon. 30. Which of the following is NOT true of the
(D) chlorophyll. oxygen used in photosynthesis?
(A) Oxygen is absorbed by the roots.
25. The word “stored” in line 11 is closest in the (B) Oxygen is the product of
meaning to photosynthesis.
(A) retained. (C) Oxygen is used in respiration.
(B) converted. (D) Oxygen is released into the atmosphere
(C) discovered. through the leaves.
(D) specified.

26. In photosynthesis, energy from the sun is


(A) changed to chemical energy.
(B) conducted from the xylem to the leaves
of green plants.
(C) not necessary to the process.
(D) released one to one for each molecule
of carbon dioxide used.

Reading Section | 94
ELPT_Level 2

Question 31-40

Alfred Bernhard Nobel, a Swedish inventor and philanthropist, bequeathed most of his vast
fortune in trust as a fund from which annual prizes could be awarded to individuals and organizations
who had achieved the greatest benefit to humanity in a particular year. Originally, there were six
classifications for outstanding contributions designated in Nobel’s will including chemistry, physics,
physiology or medicine, literature and international peace.
The prizes are administered by the Nobel Foundation in Stockholm. In 1969, a prize for
economics endowed by The Central Bank of Sweden was added. Candidates for the prizes must be
nominated in writing by a qualified authority in the field of competition. Recipients in physics,
chemistry, and economics are selected by The Royal Swedish Academy of Sciences, in physiology or
medicine by the Caroline Institute; in literature by The Swedish Academy; and in peace by the
Norwegian Nobel Committee appointed by Norway’s parliament. The prizes are usually presented in
Stockholm on December 10, with the King of Sweden officiating, an appropriate tribute to Alfred Nobel
on the anniversary of his death. Each one includes a gold medal, a diploma, and a cash award of
about one million dollars.

31. What does this passage mainly discuss?


(A) Alfred Bernhard Nobel. 37. The word “appropriate” in line 12 is closest
(B) The Nobel Prizes. in meaning to
(C) Great Contributions to mankind. (A) prestigious.
(D) Swedish Philanthropy. (B) customary.
(C) suitable.
32. Why were the prizes named for Alfred (D) transitory.
Bernhard Nobel?
(A) He left money in his will to establish a 38. The word “one” in line 13 refers to
fund for the prizes. (A) tribute.
(B) He won the first Nobel prize for his work (B) anniversary.
in philanthropy. (C) prize.
(C) He is now living in Sweden. (D) candidate.
(D) He serves as chairman of the committee
to choose the recipients of the prizes. 39. Which individual or organization serves as
administrator for the trust?
33. How often are the Nobel prizes awarded? (A) The King of Sweden.
(A) Five times a year. (B) The Nobel Foundation.
(B) Once a year. (C) The Central Bank of Sweden.
(C) Twice a year. (D) Swedish and Norwegian academies and
(D) Once every two years. institutes.

34. The word “outstanding” in line 4 could best 40. Why are the awards presented on
be replaced by December 10?
(A) recent. (A) It is attribute to the King of Sweden
(B) unusual. (B) Alfred Bernhard Nobel died on that day
(C) established. (C) That day was established in Alfred
(D) exceptional. Nobel’s will
(D) The Central Bank of Sweden
35. The word “will” in line 4 refers to administers the trust.
(A) Nobel’s wishes.
(B) a legal document.
(C) a future intention.
(D) a free choice.

36. A Nobel Prize would NOT be given to


(A) an author who wrote a novel.
(B) a doctor who discovered a vaccine.
(C) a composer who wrote a symphony.
(D) a diplomat who negotiated a peace
settlement.

Reading Section | 95
ELPT_Level 2

Question 41-50

Although stage plays have been set to music since the area of the ancient Greek, when the
dramas of Sophocles and Aeschylus were accompanied by lyres and flute, the usually accepted date
for the beginning of opera as we know it is 1600. As part of the celebration of the marriage of King
Henry IV of France to the Italian aristocrat Maria de Medici, the Florentine composer Jacopo Peri
produced his famous Euridice, generally considered the first opera. Following his example, a group of
Italian musicians, poets, and noblemen called a Cammerata began to revive the style of musical story
that has been used in Greek Tragedy. The Cammerata took most of the plots for their operas from
Greek and Roman history and mythology or musical works. It is from the phrase that the word “opera’
is borrowed.
For several years, the center of opera was Florence, but gradually, during the baroque period,
spread throughout Italy. By the late 1600s, operas were being written and performed in Europe,
especially in England, France and Germany. But, for many years, the Italian opera was considered the
ideal, and many non-Italian composers continued to use Italian librettos. The European form de-
emphasized the dramatic aspect. New orchestral effects and even ballet were introduced under the
guise of opera. Composer gave in to the demands of singers, writing many operas that were nothing
more than a succession of brilliant tricks for the voice. Complicated areas, recitatives, and duets
evolved. The aria, which is a long solo, may be compared to a song in which the characters express
their thoughts and feelings. The recitative, which is also a solo, is a recitation set to music whose
purpose is to continue the story line. The duet is a musical piece written for two voices which may
serve the function of either an aria or a recitative.

41. This passage is a summary of 46. What was the Cammerata?


(A) Opera in Italy. (A) A group of Greek Musicians.
(B) The Cammerata. (B) Musicians who developed a new
(C) The development of opera. musical drama based upon Greek
(D) Euridice. drama.
(C) a style of music not known in Italy.
42. According to this passage, when did (D) the name given to the court of King
modern opera begin? Henry IV.
(A) In the time of the ancient Greeks.
47.The word “revive” in line 6 could best be
(B) In the fifteenth century.
replaced by
(C) At the beginning of the sixteenth
(A) appreciate.
century.
(B) resume.
(D) At the beginning of the seventeenth
(C) modify.
century.
(D) investigate.
43. The word “it” in line 3 refers to
48. The word “plots” in line 7 is closest in
(A) Opera.
meaning to
(B) Date.
(A) locations.
(C) Era.
(B) instruments.
(D) Music.
(C) stories.
(D) Inspiration.
44. According to the author, what did Jacopo
Peri write?
49. From what did the term “opera” derive?
(A) Greek tragedy.
(A) Greek and Roman history and
(B) The first opera.
mythology.
(C) The opera Maria de Medico.
(B) Non-Italian composers.
(D) The Cammerata.
(C) The Italian phrase that means “musical
works”.
45. The author suggest that Euridice was
(D) The ideas of composer Jacopo Peri.
produced
(A) In France.
50. Which of the following is an example of a
(B) Originally by Sophocles and Aeschylus.
solo?
(C) Without much success.
(A) A recitative.
(D) For the wedding of King Henry IV.
(B) A duet.
(C) An opera.
(D) A lyre

Reading Section | 96
ELPT_Level 2

Practice Test 3
Question 1-10
All mammals feed their young. Beluga whale mothers, for example, nurse their calves for
some twenty months, until they are about to give birth again and their young are able to find their own
food. The behavior of feeding of the young is built into the reproductive system. It is a nonelective part
of parental care and the defining feature of a mammal, the most important thing that mammals--
whether marsupials, platypuses, spiny anteaters, or placental mammals -- have in common.
But not all animal parents, even those that tend their offspring to the point of hatching or birth,
feed their young. Most egg-guarding fish do not, for the simple reason that their young are so much
smaller than the parents and eat food that is also much smaller than the food eaten by adults. In
reptiles, the crocodile mother protects her young after they have hatched and takes them down to the
water, where they will find food, but she does not actually feed them. Few insects feed their young
after hatching, but some make other arrangement, provisioning their cells and nests with caterpillars
and spiders that they have paralyzed with their venom and stored in a state of suspended animation
so that their larvae might have a supply of fresh food when they hatch.
For animals other than mammals, then, feeding is not intrinsic to parental care. Animals add it
to their reproductive strategies to give them an edge in their lifelong quest for descendants. The most
vulnerable moment in any animal's life is when it first finds itself completely on its own, when it must
forage and fend for itself. Feeding postpones that moment until a young animal has grown to such a
size that it is better able to cope. Young that are fed by their parents become nutritionally independent
at a much greater fraction of their full adult size. And in the meantime those young are shielded
against the vagaries of fluctuating of difficult-to-find supplies. Once a species does take the step of
feeding its young, the young become totally dependent on the extra effort. If both parents are
removed, the young generally do not survive.

1. What does the passage mainly discuss? 4. What can be inferred from the passage
(A) The care that various animals give to about the practice of animal parents feeding
their offspring. their young?
(B) The difficulties young animals face in (A) It is unknown among fish.
obtaining food. (B) It is unrelated to the size of the young.
(C) The methods that mammals use to (C) It is dangerous for the parents.
nurse their young. (D) It is most common among mammals.
(D) The importance among young mammals
of becoming independent. 5. The word "provisioning" in line 11 is closest
in meaning to
2. The author lists various animals in line 5 to (A) supplying
(A) contrast the feeding habits of different (B) preparing
types of mammals (C) building
(B) describe the process by which mammals (D) expanding
came to be defined
(C) emphasize the point that every type of 6. According to the passage, how do some
mammal feeds its own young insects make sure their young have food?
(D) explain why a particular feature of (A) By storing food near their young.
mammals is nonelective (B) By locating their nests or cells near
spiders and caterpillars.
3. The word "tend" in line 6 is closest in (C) By searching for food some distance
meaning to from their nest.
(A) sit on (D) By gathering food from a nearby water
(B) move source.
(C) notice
(D) care for

Reading Section | 97
ELPT_Level 2

7. The word "edge" in line 15 is closest in 9. According to the passage, animal young
meaning to are most defenseless when
(A) opportunity (A) their parents are away searching for
(B) advantage food
(C) purpose (B) their parents have many young to feed
(D) rest (C) they are only a few days old
(D) they first become independent
8. The word "it" in line 16 refers to
(A) feeding 10. The word "shielded" in line 19 is closest in
(B) moment meaning to
(C) young animal (A) raised
(D) size (B) protected
(C) hatched
(D) valued

Question 11-21:

Printmaking is the generic term for a number of processes, of which woodcut and engraving
are two prime examples. Prints are made by pressing a sheet of paper (or other material) against an
image-bearing surface to which ink has been applied. When the paper is removed, the image adheres
to it, but in reverse.
The woodcut had been used in China from the fifth century A.D. for applying patterns to
textiles. The process was not introduced into Europe until the fourteenth century, first for textile
decoration and then for printing on paper. Woodcuts are created by a relief process; first, the artist
takes a block of wood, which has been sawed parallel to the grain, covers it with a white ground, and
then draws the image in ink. The background is carved away, leaving the design area slightly raised.
The woodblock is inked, and the ink adheres to the raised image. It is then transferred to damp paper
either by hand or with a printing press.
Engraving, which grew out of the goldsmith's art, originated in Germany and northern Italy in
the middle of the fifteenth century. It is an intaglio process (from Italian intagliare, "to carve"). The
image is incised into a highly polished metal plate, usually copper, with a cutting instrument, or burin.
The artist inks the plate and wipes it clean so that some ink remains in the incised grooves. An
impression is made on damp paper in a printing press, with sufficient pressure being applied so that
the paper picks up the ink.
Both woodcut and engraving have distinctive characteristics. Engraving lends itself to subtle
modeling and shading through the use of fine lines. Hatching and cross-hatching determine the
degree of light and shade in a print. Woodcuts tend to be more linear, with sharper contrasts between
light and dark. Printmaking is well suited to the production of multiple images. A set of multiples is
called an edition. Both methods can yield several hundred good-quality prints before the original block
or plate begins to show signs of wear. Mass production of prints in the sixteenth century made images
available, at a lower cost, to a much broader public than before.

11. What does the passage mainly discuss? 13. The author's purposes in paragraph 2 is to
(A) The origins of textile decoration describe
(B) The characteristics of good-quality prints (A) the woodcuts found in China in the fifth
(C) Two types of printmaking century
(D) Types of paper used in printmaking (B) the use of woodcuts in the textile
industry
12. The word "prime" in line 2 is closest in (C) the process involved in creating a
meaning to woodcut
(A) principal (D) the introduction of woodcuts to Europe
(B) complex
(C) general
(D) recent

Reading Section | 98
ELPT_Level 2

14. The word "incised" in line 14 is closest in 19. According to the passage, what do woodcut
meaning to and engraving have in common?
(A) burned (A) Their designs are slightly raised.
(B) cut (B) They achieve contrast through hatching
(C) framed and cross-hatching.
(D) baked (C) They were first used in Europe.
(D) They allow multiple copies to be
15. Which of the following terms is defined in produced from one original.
the passage?
(A) "patterns"(line 5) 20. According to the author, what made it
(B) "grain"(line 8) possible for members of the general public
(C) "burin"(line 14) to own prints in the sixteenth century?
(D) "grooves"(line 15) (A) Prints could be made at low cost.
(B) The quality of paper and ink had
16. The word "distinctive" in line 18 is closest in improved.
meaning to (C) Many people became involved in the
(A) unique printmaking industry.
(B) accurate (D) Decreased demand for prints kept prices
(C) irregular affordable.
(D) similar
21. According to the passage, all of the
17. According to the passage, all of the following are true about prints EXCEPT that
following are true about engraving EXCEPT they
that it (A) can be reproduced on materials other
(A) developed from the art of the goldsmiths than paper
(B) requires that the paper be cut with a (B) are created from a reversed image
burin (C) show variations between light and dark
(C) originated in the fifteenth century shades
(D) involves carving into a metal plate (D) require a printing press

18. The word "yield" in line 22 is closest in


meaning to
(A) imitate
(B) produce
(C) revise
(D) contrast

Reading Section | 99
ELPT_Level 2

Questions 22-31

The first peoples to inhabit what today is the southeastern United States sustained themselves
as hunters and gathers. Sometimes early in the first millennium A.D., however, they began to cultivate
corn and other crops. Gradually, as they became more skilled at gardening, they settled into
permanent villages and developed a rich culture, characterized by the great earthen mounds they
erected as monuments to their gods and as tombs for their distinguished dead. Most of these early
mound builders were part of the Adena-Hopewell culture, which had its beginnings near the Ohio River
and takes its name from sites in Ohio. The culture spread southward into the present-day states of
Louisiana, Alabama, Georgia, and Florida. Its peoples became great traders, bartering jewellery,
pottery, animal pelts, tools, and other goods along extensive trading networks that stretched up and
down eastern North America and as far west as the Rocky Mountains.
About A.D. 400, the Hopewell culture fell into decay. Over the next centuries, it was
supplanted by another culture, the Mississippian, named after the river along which many of its earliest
villages were located. This complex civilization dominated the Southeast from about A.D. 700 until
shortly before the Europeans began arriving in the sixteenth century. At the peak of its strength, about
the year 1200, it was the most advanced culture in North America. Like their Hopewell predecessors,
the Mississippians became highly skilled at growing food, although on a grander scale. They
developed an improved strain of corn, which could survive in wet soil and a relatively cool climate, and
also learned to cultivate beans. Indeed, agriculture became so important to the Mississippians that it
became closely associated with the Sun --- the guarantor of good crops. Many tribes called
themselves "children of the Sun" and believed their omnipotent priest-chiefs were descendants of the
great sun god.
Although most Mississippians lived in small villages, many others inhabited large towns. Most
of these towns boasted at least one major flat-topped mound on which stood a temple that contained a
sacred flame. Only priests and those charged with guarding the flame could enter the temples. The
mounds also served as ceremonial and trading sites, and at times they were used as burial grounds.

22. What does the passage mainly discuss? 24. What does the term "Adena-Hopewell"(line
(A) The development of agriculture 6) designate?
(B) The locations of towns and villages (A) The early locations of the Adena-
(C) The early people and cultures of the Hopewell culture
United States (B) The two most important nations of the
(D) The construction of burial mounds Adena-Hopewell culture
(C) Two former leaders who were honored
23. Which of the following resulted from the rise with large burial mounds.
of agriculture in the South Eastern United (D) Two important trade routes in eastern
States? North America
(A) The development of trade in North
America 25. The word "bartering" in line 8 is closest in
(B) The establishment of permanent meaning to
settlements (A) producing
(C) Conflicts with other Native American (B) exchanging
groups over land (C) transporting
(D) A migration of these peoples to the (D) loading
Rocky Mountains.
26. The word "supplanted" in line 12 is closest
in meaning to
(A) conquered
(B) preceded
(C) replaced
(D) imitated

Reading Section | 100


ELPT_Level 2

27. According to the passage, when did the 29. Why does the author mention that many
Mississippian culture reach its highest point Mississippians tribes called themselves
of development? "children of the Sun"(line 15)?
(A) About A.D. 400 (A) To explain why they were obedient to
(B) Between A.D. 400 AND A.D. 700 their priest-chiefs.
(C) About A.D. 1200 (B) To argue about the importance of
(D) In the sixteenth century religion in their culture.
(C) To illustrate the great importance they
28. According to the passage, how did the placed on agriculture.
agriculture of the Mississippians differ from (D) To provide an example of their religious
that of their Hopewell predecessors? rituals.
(A) The Mississippians produced more
durable and larger crops of food. 30. The phrase "charged with" in line 24 is
(B) The Mississippians sold their food to closest in meaning to
other groups. (A) passed on
(C) The Mississippians could only grow (B) experienced at
plants in warm, dry climates. (C) interested in
(D) The Mississippians produced special (D) assigned to
foods for their religious leaders.
31. According to the passage, the flat-topped
mounds in Mississippian towns were used
for all of the following purposes EXCEPT
(A) religious ceremonies
(B) meeting places for the entire community
(C) sites for commerce
(D) burial sites

Reading Section | 101


ELPT_Level 2

Question 32-40

Overland transport in the United States was still extremely primitive in 1790. Roads were few
and short, usually extending from inland communities to the nearest river town or seaport. Nearly all
interstate commerce was carried out by sailing ships that served the bays and harbors of the
seaboard. Yet, in 1790 the nation was on the threshold of a new era of road development. Unable to
finance road construction, states turned for help to private companies, organized by merchants and
land speculators who had a personal interest in improved communications with the interior. The
pioneer in this move was the state of Pennsylvania, which chartered a company in 1792 to construct a
turnpike, a road for the use of which a toll, or payment, is collected, from Philadelphia to Lancaster.
The legislature gave the company the authority to erect tollgates at points along the road where
payment would be collected, though it carefully regulated the rates. (The states had unquestioned
authority to regulate private business in this period.)
The company built a gravel road within two years, and the success of the Lancaster Pike
encouraged imitation. Northern states generally relied on private companies to build their toll roads,
but Virginia constructed a network at public expense. Such was the road building fever that by 1810
New York alone had some 1,500 miles of turnpikes extending from the Atlantic to Lake Erie.
Transportation on these early turnpikes consisted of freight carrier wagons and passenger
stagecoaches. The most common road freight carrier was the Conestoga wagon, a vehicle developed
in the mid-eighteenth century by German immigrants in the area around Lancaster, Pennsylvania. It
featured large, broad wheels able to negotiate all but the deepest ruts and holes, and its round bottom
prevented the freight from shifting on a hill. Covered with canvas and drawn by four to six horses, the
Conestoga wagon rivaled the log cabin as the primary symbol of the frontier. Passengers traveled in a
variety of stagecoaches, the most common of which had four benches, each holding three persons. It
was only a platform on wheels, with no springs; slender poles held up the top, and leather curtains
kept out dust and rain.

32. Paragraph 1 discusses early road building 35. The phrase "on the threshold of" in line 4 is
in the United States mainly in terms of the closest in meaning to
(A) popularity of turnpikes (A) in need of
(B) financing of new roads (B) in place of
(C) development of the interior (C) at the start of
(D) laws governing road use (D) with the purpose of

33. The word "primitive" in line 1 is closest in 36. According to the passage, why did states
meaning to want private companies to help with road
(A) unsafe building?
(B) unknown (A) The states could not afford to build
(C) inexpensive roads themselves.
(D) undeveloped (B) The states were not as well equipped as
private companies.
34. In 1790 most roads connected towns in the (C) Private companies could complete roads
interior of the country with faster than the states.
(A) other inland communities (D) Private companies had greater
(B) towns in other states knowledge of the interior.
(C) river towns or seaports
(D) construction sites 37. The word "it" in line 10 refers to
(A) legislature
(B) company
(C) authority
(D) payment

Reading Section | 102


ELPT_Level 2

38. The word "imitation" in line 13 is closest in 40. The "large, broad wheels" of the Conestoga
meaning to wagon are mentioned in line 14 as an
(A) investment example of a feature of wagons that was
(B) suggestion (A) unusual in mid-eighteenth century
(C) increasing vehicles
(D) copying (B) first found in Germany
(C) effective on roads with uneven surfaces
39. Virginia is mentioned as an example of a (D) responsible for frequent damage to
state that freight
(A) built roads without tollgates
(B) built roads with government money
(C) completed 1,500 miles of turnpikes in
one year
(D) introduced new law restricting road use

Question 41- 50

In Death Valley, California, one of the hottest, most arid places in North America, there is
much salt, and salt can damage rocks impressively. Inhabitants of areas elsewhere, where streets and
highways are salted to control ice, are familiar with the resulting rust and deterioration on cars. That
attests to the chemically corrosive nature of salt, but it is not the way salt destroys rocks. Salt breaks
rocks apart principally by a process called crystal prying and wedging. This happens not by soaking
the rocks in salt water, but by moistening their bottoms with salt water. Such conditions exist in many
areas along the eastern edge of central Death Valley. There, salty water rises from the groundwater
table by capillary action through tiny spaces in sediment until it reaches the surface.
Most stones have capillary passages that suck salt water from the wet ground. Death Valley
provides an ultra-dry atmosphere and high daily temperatures, which promote evaporation and the
formation of salt crystals along the cracks or other openings within stones. These crystals grow as
long as salt water is available. Like tree roots breaking up a sidewalk, the growing crystals exert
pressure on the rock and eventually pry the rock apart along planes of weakness, such as banding in
metamorphic rocks, bedding in sedimentary rocks, or preexisting or incipient fractions, and along
boundaries between individual mineral crystals or grains. Besides crystal growth, the expansion of
halite crystals (the same as everyday table salt) by heating and of sulfates and similar salts by
hydration can contribute additional stresses. A rock durable enough to have withstood natural
conditions for a very long time in other areas could probably be shattered into small pieces by salt
weathering within a few generations.
The dominant salt in Death Valley is halite, or sodium chloride, but other salts, mostly
carbonates and sulfates, also cause prying and wedging, as does ordinary ice. Weathering by a
variety of salts, though often subtle, is a worldwide phenomenon. Not restricted to arid regions, intense
salt weathering occurs mostly in salt-rich places like the seashore, near the large saline lakes in the
Dry Valleys of Antarctica, and in desert sections of Australia, New Zealand, and central Asia.

41. What is the passage mainly about? 42. The word "it" in line 8 refers to
(A) The destructive effects of salt on rocks. (A) salty water
(B) The impressive salt rocks in Death (B) groundwater table
Valley. (C) capillary action
(C) The amount of salt produced in Death (D) sediment
Valley.
(D) The damaging effects of salt on roads 43. The word "exert" in line 12 is closest in
and highways. meaning to
(A) put
(B) reduce
(C) replace
(D) control

Reading Section | 103


ELPT_Level 2

44. In line 12, why does the author compare 48. The word "dominant" in line 20 is closest in
tree roots with growing salt crystals? meaning to
(A) They both force hard surfaces to crack. (A) most recent
(B) They both grow as long as water is (B) most common
available. (C) least available
(C) They both react quickly to a rise in (D) least damaging
temperature.
(D) They both cause salty water to rise from 49. According to the passage, which of the
the groundwater table. following is true about the effects of salts on
rocks?
45. In line 16, the author mentions the (A) Only two types of salts cause prying and
"expansion of halite crystals...by heating wedging.
and of sulfates and similar salts by (B) Salts usually cause damage only in
hydration" in order to combination with ice.
(A) present an alternative theory about (C) A variety of salts in all kinds of
crystal growth environments can cause weathering.
(B) explain how some rocks are not affected (D) Salt damage at the seashore is more
by salt severe than salt damage in Death
(C) simplify the explanation of crystal prying Valley.
and wedging
(D) introduce additional means by which 50. Which of the following can be inferred from
crystals destroy rocks the passage about rocks that are found in
areas where ice is common?
46. The word "durable" in line 17 is closest in (A) They are protected from weathering.
meaning to (B) They do not allow capillary action of
(A) large water.
(B) strong (C) They show similar kinds of damage as
(C) flexible rocks in Death Valley.
(D) pressured (D) They contain more carbonates than
sulfates
47. The word "shattered" in line 18 is closest in
meaning to
(A) arranged
(B) dissolved
(C) broken apart
(D) gathered together

Reading Section | 104


ELPT_Level 2

BIBLIOGRAPHY
Hinkel, Eli, PhD. (2004). TOEFL Test Strategies With Practice Test. New York: Barron’s Educational
Series, Inc.

J. Sharpe, Pamela, Ph.D. (1999). Barron's How to Prepare for the TOEFL - Test of English As a
Foreign Language - 9th edition. New York: Barron’s Educational Series, Inc.

Phillips, Deborah. (2001). Longman Complete course for the TOEFL ® Test. New York: Pearson
Education

Phillips, Deborah. (2004). Longman Introductory course for the TOEFL ® Test. New York: Pearson
Education

Mahnke, M. Kathleen and Duffy, Carolyn B. (1996). The Heinemann ELT TOEFL ® Preparation
Course, Oxford: Macmillan Publishers Limited.

Bibliography | 105

You might also like